CBSE previous Year Solved Papers Class 12 English Outside Delhi 2013

CBSE previous Year Solved  Papers  Class 12 English Outside Delhi 2013

Time allowed : 3 hours                                                                                           Maximum Marks: 100
General Instructions :

  1. This paper is divided into three sections : A, B and C. All the sections are compulsory.
  2. Separate instructions are given with each section and question, wherever necessary. Read these instructions very carefully and follow them faithfully.
  3. Do not exceed the prescribed word limit while answering the questions.

SET I

SECTION – A
(Reading)

Question.1. Read the passage given below and answer die questions
that follow:

  1.  Air pollution is an issue which concerns us all alike. One can willingly choose or reject a food, a drink or a life comfort, but unfortunately there is tittle choice for the air we breathe. All that is there in the air is inhaled by one and all living in those surroundings.
  2. Air pollutant is defined as a substance which is present in an amount exceeding the normal concentrations. It could either be gaseous or a particulate matter. The important and harmful polluting gases are carbon monoxide, carbon dioxide, ozone and oxides of sulphur and nitrogen. The common particulate pollutants are the dusts of various inorganic or organic origins. Although we often talk of the outdoor air pollution caused by industrial and vehicular exhausts, the indoor pollution may prove to be as or a more important cause of health problems.
  3.  Recognition of air pollution is relatively recent. It is not uncommon to experience a feeling of ‘suffocation’ in a closed environment. It is often ascribed to the lack of oxygen. Fortunately, however, the composition of air is remarkably constant all over the world. There is about 79 percent nitrogen and 21 per cent oxygen in the air – the other gases forming a very small fraction. It is true that carbon dioxide exhaled out of lungs may accumulate in a closed and over-crowded place. But such an increase is usually small and temporary unless the room is really air-tight. Exposure to poisonous gases such as carbon monoxide may occur in a closed room, heated by burning coal inside. This may also prove to be fatal.
  4. What is more common in a poorly ventilated home is a vague constellation of symptoms described as the sick-building syndrome. It is characterized by a general feeling of malaise, head-ache, dizziness and irritation of mucous membranes. It may also be accompanied by, nausea, itching, aches, pains and depression. Sick building syndrome is getting commoner in big cities with the small houses, which are generally over¬furnished. Some of the important pollutants whose indoor concentrations exceed to those of the outdoors include gases such as carbon monoxide, carbon dioxide, oxides of nitrogen and organic substances like spores, formaldehydes, hydrocarbon aerosols and allergens. The sources are attributed to a variety of construction materials, insulations, furnishings, adhesives, cosmetics, house dusts, fungi and other indoor products.
  5.  By-products of fuel combustion are important in houses with indoor kitchens. It is not only the burning of dried dung and foelwood which is responsible, but also kerosene and liquid petroleum gas. Oxides of both nitrogen and sulphur are released from their combustion.
  6.  Smoking of tobacco in a closed environment is an important source of indoor pollution. It may not be “high quantitatively, but significantly hazardous for health. It is because of the fact that there are over 3000 chemical constituents in tobacco smoke, which have been identified. These are harmful for human health.
  7.  Micro-organisms and allergens are of special significance in the causation and spread of diseases. Most of the infective illnesses may involve more persons of a family living in common indoor environment. These include viral and bacterial diseases like tuberculosis.
  8.  Besides infections, allergic and hypersensitivity disorders are spreading fast. Although asthma is the most common form of respiratory allergic disorders, pneumonias are common, but more persistent and serious. These are attributed to exposures to allergens from various fungi, molds, hay and other organic materials. Indoor air ventilation systems, coolers, air- conditioners, dampness, decay, pet animals, production or handling of the causative items are responsible for these hypersensitivity – diseases.
  9. Obviously, the spectrum of pollution is very wide and our options are limited. Indoor pollution may be handled relatively easily by an individual. Moreover, the good work must start from one’s own house.

(a) (i) What is an air pollutant ?
Answer : Air pollutant is defined as a substance which is present in the air while normally, it is not present in an amount exceeding the normal concentrations.
(ii) In what forms are the air pollutants present ?
Answer : Air pollutants are present in gaseous form such as carbon monoxide, carbon dioxide, ozone and oxides of sulphur and nitrogen. The pollutants are also in the form of dust of various inorganic or organic origins.
(iii) Why do we feel suffocated in a closed environment ?
Answer : We feel suffocated in a closed environment due to lack of oxygen. Accumulation of carbon dioxide exhaled out of lungs and exposure to carbon monoxide are also the reasons of suffocation.
(iv) What is sick building syndrome ? How is it increasing ?
Answer : Sick building syndrome is characterized by a general feeling of malaise, headache, dizziness and irritation of mucous membrane. It may be accompanied by nausea, itching, aches, pains and depression.
It is increasing due to poorly ventilated and over furnished houses in big cities in which there is high Concentration of carbon dioxide, carbon monoxide and oxides of nitrogen and other harmful substances.
(v) How is indoor smoking hazardous ?
Answer : Indoor smoking is hazardous in the way that there are 3,000 chemical constituents in tobacco smoke which are harmful for human health.
(vi) How can one overcome the dangers of indoor air pollution?
Answer : The houses should be properly ventilated with more and more greenery around them. Smoking of tobacco and burning of coal inside the houses should also be avoided to overcome the dangers of indoor air pollution.
(b) Find the words from the above passage which mean the same as the following:
(i) Giddiness (para 4) 
Answer: Dizziness.
(ii) Constant (para 8)
Answer: Persistent.
(iii) Humidity (para 8)
Answer: Dampness.

Question.2. Read the passage below and answer the questions that follow:
The term dietary fibres refers collectively to’ indigestible carbohydrates present in plant foods. The importance of the these dietary fibres came into the picture when it was observed that the people having diet rich in these fibres, had low incidence of coronary heart disease, irritable bowel syndrome,’dental caries and gall stones.The foodstuffs rich in these dietary fibres are cereals and grains, legumes, fruits with seeds, citrus fruits, carrots, cabbage, green
leafy vegetables, apples, melons, peaches, pears etc.
These dietary fibres are not digested by the enzymes of the stomach ‘and the small intestine whereas, most of the other carbohydrates like starch and sugar are digested and absorbed. The dietary fibres have the property of holding water and because of it, these get swollen and behave like a sponge as these pass through the gastrointestinal tract. The fibres add bulk to the diet and increase transit time in the gut. Some of these fibres may undergo fermentation in the colon.
In recent years, it has been considered essential to have some amount of fibres in the diet. Their beneficial effects help in preventing coronary heart disease, and decreasing cholesterol levels. The fibres like gums and pectin are reported to decrease’ postprandial (after meals) glucose level in blood. These types of dietary fibres ate recommended for the management of certain types of diabetes. Recent studies have shown that the fenugreek (Methi) seeds, which contain 40 per cent gum, are effective in decreasing blood glucose and cholesterol levels as i compared to other gum containing vegetables.
Some dietary fibres increase transit time and decrease the time of release of ingested food in colon. The diet having less fibres is associated with colon cancer and the dietary fibres may play a role in decreasing the risk of it.
Theidietary fibres hold water so that stools are soft, bulky and readily eliminated. Therefore high fibre intake prevents or relieves constipation.
The fibres increase motility of the small intestine and the colon and by decreasing the transit time there is less time’ for exposure of the mucosa to harmful toxic substances. Therefore, there is a less, desire to eat and the energy intake can be maintained within the range of requirement. This phenomenon helps in keeping a check on obesity Another reason in helping to decrease obesity is that the high-fibre diets have somewhat lower coefficients of digestibility.
The dietary fibres may have some adverse effects on nutrition by binding some trace metals like calcium, magnesium, phosphorus, zinc and others and therefore preventing their proper absorption. This may pose a possibility of nutritional deficiency especially when diets contain marginal levels of mineral elements. This may become an important constraint on increasing dietary fibres. It is suggested that an intake of 40 grams dietary fibres per day is desirable.
(a) On the basis of your reading of the above passage, make notes in points only, using abbreviations wherever necessary. Supply a suitable title.
Answer:
Tide: Dietary Fibres
1. Notes
(i) Importance: lowers
(a) hrt disease
(b) irrtble bowel syndrome
(c) dental caries
(d) gall stones
(ii) Rich dietary fibers
(a) cereals, grains, legumes (b) fruits
(c) vegetables
(iii) Adverse effects
(a) bind metals like ca, mg, ph, zn
(b) prevents their absoptn
(c) nutnl defcncy
cbse-previous-year-solved-papers-class-12-english-outside-delhi-2013-1
(b) Write a summary of the above in about 80 words.  Summary: Dietary fibers are indigestible carbohydrates present in plant foods. People having diet rich in fibers have low heart diseases, irritable bowel syndrome, dental caries and gall stones. Sources of dietary fibers are legumes, cereals, fruits, vegetables and grains. The fibers like gums present in fenugreek and pectin decrease glucose level. The diet which is less in fibers results in colon cancer. The dietary fibers hold water and functions like sponge, hence they prevent constipation.
But there are adverse effects also. They bind some trace metals resulting in nutritional deficiency.

SECTION – B
(Advanced Writing Skills)

Question.3. You are Vineeta/Vikram, School pupil leader of Rani Laxmi
Bai Senior Secondary School, Gwalior. Draft a notice for your school notice board in not more than 50 words inviting the names of the students who want to participate in the cultural programme organized in aid of the victims of the recent Assam floods.
OR
You have a three bedroom flat in Dwarka, which you want to let out on rent. Draft an advertisement in not more than 50 words to be published in “The Times of India” under classified columns, Contact 2758902.
Answer:
cbse-previous-year-solved-papers-class-12-english-outside-delhi-2013-2
cbse-previous-year-solved-papers-class-12-english-outside-delhi-2013-3

Question.4. Maxim Gorky School, Kodaikanal celebrated “Nutrition Week” from 1st September to 7th September by arranging inter-school cookery contests, oratorical, painting and poster competition, besides talks by eminent nutritionists and medical professionals. There was an overwhelming response from students and parents. Prepare a report in 100¬125 words to be published in your school magazine. You are Arjun/Anita, School pupil leader.
OR
You are Roshan/Rohini, School Pupil Leader of Meerut Public School, Meerut. Your school joined a campaign organized by various agencies of your city to create awareness among people to conserve water. Write a report in 100-125 words highlighting the activities such as rain water harvesting etc.
Answer:
NUTRITION-WEEK
REPORT BY: ANITA (School Pupil Leader)
Maxim Gorky School, Kodaikanal celebrated Nutrition
Week from 1st September to 7th September in the school auditorium. The celebration started with lightning of candles by our respected Principal Sir followed by his inspirational speech on the present day’s health issues and the impket of diet on it. Different schools participated in various competitions organized by the school. The competitions included inter¬school cookery contests, oratorical, painting and poster competition, besides talks by eminent nutritionists and medical professionals was also organized. The students as well as their parents gave an overwhelming response because nowadays everybody is conscious about their health and well-being. The speech on the nutritional value of food by a student, was praised by all. The programme was a combined effort of the teachers and the students and received a huge appreciation by the ones present there.
In the end, the results of the competitions organized were declared and the winners were awarded prizes by Principal Sir.
OR
WATER CONSERVATION
A Report By: Rohini (School Pupil Leader)
Water scarcity today has become a big issue which if not dealt with in time, will turn out to be a hazard.To make people aware to conserve water, Meerut Public School joined a campaign organised by various agencies of the city. People in large numbers attended it and were asked to conserve every drop of water and not waste it unnecessarily. Small skits presenting “* the ways of conserving water were presented by the children.
It was an easy approach as the venue of the campaign was the Public Park and the passers by also stopped to become a part of the campaign. Besides, importance of rain water harvesting and the method to do it was also explained. Without water there is no life. They were encouraged to improve water management practices that reduce the wastage of water and enhance the beneficial uses of water. Some ancient methods of water conservation should also be practiced. In case the water is not used wisely by anyone, then heavy fine should be _ imposed. Public was motivated by this, and they joined hands and took oath to use water properly and motivate others also.

Question.5. Write a letter to your cousin, Raj Prakash who is currently staying in Dubai explaining the process of CCE, being used by CBSE for the secondary school examination in its affiliated schools. Also mention how you and your classmate have reacted to this scheme. You are Narain/Nisha, 20 Fort Road, Mumbai.
OR
Recently you travelled from Bangalore city to Vasco Express. To your dismay, you found that the coach was infested with cockroaches. Write a letter to the editor “The Hindu” drawing the attention of the General Manager, Southern Railways, to the prevailing unhygienic conditions and asking for remedial action. You are Saroj/Saran, 5/31 Bangalore Cantt., Bangalore.
Answer:
20, Fort Road, Mumbai 20th April, 2014
Dear Raj Prakash,
Hope you will be good in your health and very well adjusted in the atmosphere of Dubai. How do you find your school ? What pattern of examination is followed there ? Here, in our schools, CBSE has started a new pattern-CCE (Continuous and Comprehensive Evaluation). This system evaluates the performance of the students in scholastic as well as co-scholastic areas. It has lessened the burden of the children and we feel that it will go a long way. It includes the formative and summative assessment. With this new system it has become possible for us to learn and do-it-yourself in a fun-filled way. Learning has become a pleasure as it has taken away the fear of examination. It not only excel us in academics but our holistic development also. We all are enjoying it and it is a positive approach towards learning.
Hope you will be doing fine in your studies. Convey my regards to uncle and aunty.
Yours affectionately,
Nisha .
OR
5/31, Bangalore Cantt.
Bangalore .
10th January, 2014 The Editor ,
The Hindu ,
Subject: Unhygienic conditions in train coaches Dear Sir,
Through the columns of your esteemed newspaper, I would like to draw the attention of the General Manager, Southern Railways towards the unhygienic conditions of train coaches. „ Recently, I was travelling from Bangalore city to Vasco in Vasco Express, I noticed that the coach was infested with cockroaches. The sheets and blankets given to the passengers were not properly washed; the food was also stale and uncovered, toilets were unclean, wash basins were dirty. For this reason also, the cockroaches are breeding and the whole place is infested with them. Therefore imrtiediate remedial action is required. There should be regular check on the cleanliness of the coaches and the staff members who are given the duty, should also be taken into charge. If they are not performing their duty, they should be fined so that the negligence is not repeated. And to be sure, this will bring a definite change.
I shall be grateful if this letter finds a place in your newspaper.
Yours Sincerely .
Saroj.

Question.6. Write an article in 150-200 words for your school magazine on the topic. “Obesity among School Children.” You are Mohini/Mohit.
OR
You are Rainesh/Ruchika. Write an article in 150-200 words for your school magazine on the topic, “Life without Modern Gadgets”.
Answer:
OBESITY AMONG SCHOOL CHILDREN . By: Mohini
The most common problem found among the children in todays scenario is the growing problem of obesity. This is the result of present day lifestyle where parents are much responsible for it. Even the tastes of the children have changed a lot and they are moving towards junk food like burgers, pizza, pasta and many more. These are not only harmful for their proper growth but also affect their metabolism. As the children are confined to their rooms playing on laptops, computer games and mobiles, the outdoor activities have come to an end. As a result more and more children are becoming obese and overweight which is alarming because it causes health problems which in result, leading to social issues. They are likely to be teased by their peers and they tend to become inactive. Once children become obese; it is very difficult to return to the original shape. Apart from this, the risk factors are very high. They are more prone to high . blood pressure, weak heart, diabetes, orthopedic disorders etc. So the only remedy for this is the awareness, less intake of junk foods, consumption of homemade food, avoiding aerated drinks and more and more physical activities. Then only this problem can be overcome.
OR
LIFE WITHOUT MODERN GADGETS
By: Ruchika
Science has made life easy, convenient, comfortable, better and more beautiful. Its achievements, discoveries and inventions are wonderful. Among them is the boon of gadgets. We cannot think of over life without gadgets as it Would just be miserable. Every device or appliance has a unique purpose. They have revolutionized our life so much that without them we feel handicapped. Beginning from kitchen, gadgets like mixer, microwave oven, refrigerator and water purifier have proved to be a boon. Cell phones are the center of our life. t I-phones and I-pad have huge potential, we can listen to our favorite music while travelling and need not carry heavy radios.
Our life starts with the sound of an alarm clock, so we can say that our day begins with gadgets. They work as our personal assistant. In offices, meetings, trainings and all, we are dependent on them only. Even at the shops, libraries, railways, banks, airports, a computer is required for booking tickets and keeping accounts. If for a day our internet is not working, we feel paralyzed. It feels as something important is missing from our life. Thus, these gadgets have left a remarkable impact on our lives and we cannot work or go without them.

Question.7. Read the extract given below and answer the questions that follow:
The stunted, unlucky heir Of twisted bones, reciting-a father’s gnarled disease, His lesson, from his desk. At the back of the dim class One unnoted, sweet and young.
OR
Driving from my parents Home to Cochin last Friday Morning, I saw my mother, beside me,
Doze, open mouthed, her face ashen like that Of a corpse and realized with pain That she was as old as she looked …
(i) Who is this ‘unlucky heir1 and what has he inherited ?
Answer : The boy who has twisted bones and is sitting in the slum classroom is the unlucky heir. He has inherited the gnarled disease of twisted bones from his father.
(ii) What is the stunted boy reciting ?
Answer : The stunted boy is reciting his father’s gnarled disease and his lessons.
(iii) Who is sitting at the back of the dim class ?
Answer : At the back of the dim class, there is an unnoted, sweet and young dreamer sitting, who is dreaming of squirrel’s game.
(i) Where was the poet driving to ?
Answer : The poet was driving to Cochin from her parents home.
(ii) Why was her mother’s face looking like that of a corpse ?
Answer : Her mother’s face was looking like a corpse because she was old, weak, pale and lifeless.
(iii) What did the poet notice about her mother ?
Answer: The poet noticed that her mother’s mouth was open and her face looked like that of a corpse.

Question.8. Answer any three of the following in 30-40 words each :
(a) What does Keats consider an endless fountain of immortal drink and why does he call its drink immortal ?
Answer : Beauty, according to Keats is immortal and is continuous source of motivation and inspiration and things of beauty are like endless mountains pouring from heaven.
It is an endless fountain of beautiful things so that man can enjoy happiness and peace.
(b) What are the different kinds of wars mentioned in the poem ? What is. Neruda’s attitude towards these wars ?
Answer:The poet talked of the chemical wars, war with fire, wars with ammunition, war with environment and green wars. According to him, these wars are the gate to destruction, therefore should be avoided, as there will be no survivors to enjoy the victory.
(c) What lies heavily on Aunt Jennifer’s hand ? How is it associated with her husband ?
Answer : The wedding band and the ring lie heavily on Aunt Jennifer’s hand. The ring symbolizes the weight of her marriage which she has to bear, dead or alive. It reminds her of the pressures of an unhappy and unpleasant marriage.

Question.9. Answer any three of the following in 30-40 words each:
(a) What was the order from Berlin and what changes did it cause in the school ?
Answer : There was an order from Berlin that French would not be taught in school anymore. Instead German would be taught by a new teacher. That was probably their last French lesson. It was not a Sunday morning yet, there was no noise or activity in the school. The French teacher Mr. Hamel, seemed to be calm and he was becoming emotional. He was wearing a special dress and all the villagers were sitting on the back benches of the class. The whole atmosphere of the school was silent and peaceful.
(b) How did Douglas make sure that he conquered the old terror ?
Answer : The old terror always returned whenever Douglas was alone in the pool. He covered the length of the pool but when he was not satisfied, he went to Lake Wentworth and ^ swam two miles to overcome his terror. The terror was now gone and he laughed at it.
(c) Why was Sophie jealous of Geoff’s silence ?
Answer : Sophie was jealous of Geoff’s silence because she wanted him to share everything as she used to do. Geoff was an introvert and spoke very little. She thought that there was something about his life that was unknown to her.

Question.10. Answer the following in 125-150 words :
OR
Why did Gandhiji agree to a setdement of 25 per cent refund to the farmers ? How did it influence the peasant- landlord relationship in Champaran ?
Answer : Gandhiji agreed to the settlement of 25 percent because the amount was not important for him but to release the peasants from the indigo raising agreement was much more important. The British planters wanted to prolong the agreement but Gandhiji proved to be wiser. He fought for the poor peasants a long battle for one year and managed to get justice for them. This was a moral victory for them. Even the Britishers had to compromise with their pride and prestige. Peasants now got the courage and became aware of their rights. Their money and time was saved and within few years British planters were forced to give up their estates and these were returned to farmers. They became the owners of their own land. It also ignited the feelings of self-dependence and patriotism in farmers.

Question.11. Answer the following in 30-40 words :
Give a character-sketch of the Governor of Oxford Prison based on your understanding of the story ‘Evans Tries an O-level’.
Answer : Governor of Oxford Prison was a generous man who allowed Evans to appear for exam. Evans was a criminal and had escaped from jail twice, so the governor took all the possible precautions for the smooth conduct of examination. The examination had been arranged in the cell itself. The door was locked one day before and the security was made very tight. Everything was checked thoroughly to ensure that there was no way that could help him escape. This shows that he was very wise, on the other hand he was so simple that he couldn’t understand the cunningness of Evans.

Question.12. Answer the following in about 100 words:
There is a saying ‘kindness pays, rudeness never’. In the story ‘The Rattrap’ Edla’s attitude towards men and matters is different from her father’s attitude. How are the values of concern and compassion brought out in the story, ‘The Rattrap’ ?
Answer : Edla, the ironman’s daughter was totally different from her father. In the beginning, the iron master seemed to be very sympathetic when he mistook the’stranger for one of his old acquaintance. He even sent his daughter to call for him when he did not agree to come to their house. But soon his attitude changed when he came to know the truth that the stranger was someone else. He called him dishonest and threatened to hand him over to the police. Whereas his daughter was entirely different from her father. She had different-ways of looking at things. She was affectionate and considerate. She acted more wisely than her father. She stopped her father from behaving badly towards the stranger. She proved to be a good host and it was her, who brought change in his behavior. It was her generosity and sympathy that converted him entirely. These values strengthen our hope in goodness.

Question.13. Answer the following in 30-40 words :
(a) What forced Dr. Sadao to be impatient and irritable with his patient ?
Answer : When Dr. Sadao saw the difficulties faced by his wife in the absence of maids and servants due to the presence of an enemy in his house, he felt impatient and irritable with him. Besides when this secret was out that he was sheltering an enemy, he feared of getting into serious trouble. It caused a great deal of tension to him.
(b) Why does Jo insist that her father should tell her the story with a different ending ?
Answer : Jo insists that her father should tell her the story with a different ending because-she wants that other animals should play with Roger Skunk and for that he must smell like roses. According to her, the mother is wrong as she made him smell foul again and the wizard should hit his mother. In fact, she wants to assert her views on her father.
(c) If you were to give a different ending to the story ‘On the face of it’ how would you end it ?
Answer : If I, were given a chance to end the story, surely it would be a happy ending. Mr. Lamb would not have died and Derry along with him would have started a new life. They would have been living happily in each other s company.
(d) What were the articles in the stalls and shops that fascinated Bama on her way back from school ?
Answer : It was a daily routine of the writer to walk home from school, which was a ten minute walk. But it always took her thirty minutes to reach home as the stalls and shops on the way fascinated Bama alot. These stalls and shops were of sweets, dried fish, fried snacks etc.

Going Places Important Questions CBSE Class 12 English

Going Places Important Questions CBSE Class 12 English

Short Answer Type Questions (3 Marks, 30-40 Words)

Question.1. Why is Sophie attracted to Danny Casey? (Compartment 2014)
Answer. Danny Casey is a young and successful football player from Ireland. He is also quite handsome, which adds to his popularity among his fans, especially young giris of Sophie’s age. Naturally, Sophie is attracted to him. Besides, her family members are also huge football fans and Casey is a hero for them.

Question.2. How are Jansie and-Sophie different from each other? (Compartment 2014)
Answer. Jansie and Sophie have contrasting characters and an altogether different approach towards life. Jansie is quite practical and her feet are grounded in reality, whereas Sophie is a daydreamer and lives in an imaginary world. Jansie is mature and accepts the truth that people of their stature can only become workers in a biscuit factory. On the other hand, Sophie tries to escape from reality and dreams of becoming an actress, a manager or a fashion designer.

Question.3. Why did Sophie long for her brother’s affection? (All India 2014)
or
Why did Sophie like her brother Geoff more than any other person? (Delhi 2013)
Answer. Sophie found in her brother, Geoff a patient listener to all her fantasies and also one she could confide in. The otjier members of her family and even her friend Jansie made fun of her fantastic stories. This made her quite fond of Geoff. She also looked upon Geoff as someone widely travelled. His world remained a fascination for her and she longed that someday he might take her thera

Question.4. Did Geoff keep up his promise? How do you know? (All India 2013)
or
Did Geoff keep his promise to Sophie? How do you know? (Delhi 2011)
Answer. Geoff didn’t keep his promise to Sophie. Sophie told Geoff about her meeting with Danny Casey at the Royce’s. Geoff told the story to Frank who in turn told his sister, Jansie, about it. However, Geoif never revealed the full story, as he did not tell anyone about Sophie’s supposed’date’with Danny Casey.

Question.5. Why was Sophie jealous of Geoff’s silence?
Answer. Geoff never spoke much. Sophie always pondered over the areas of his life which he never talked about. His world remained a fascination for her. She felt that when he was not speaking, his mind was away at some unknown place, and so she felt jealous of him.

Question.6. Why did Sophie not want Jansie to know anything about her meeting with Danny
Casey? (All India 2012)
or
Why didn’t Sophie want Jansie to know about her story with Danny? (Delhi 2011)
Answer. Sophie did not want Jansie to know anything about her meeting with Danny Casey because she knew that Jansie was a blabber mouth. The moment Jansie knew of someone’s secret, the whole neighbourhood would get to know of it. Sophie did not want to be ridiculed before others.

Question.7. How did Sophie’s father react when Geoff told him about her meeting with Danny
Casey?(Delhi 2011)
Answer. Sophie’s father is a realist to the core. He does not believe in Sophie’s story at all and dismisses it as another of her wild fantasies. When Geoff tells him about Sophie’s chance encounter with Danny Casey, the Irish prodigy, he expresses disdain and rubbishes her story. He changes the topic by saying that he once knew a man who knew another famous English footballer named Tom Finney. He warns Sophie that such concocted stories would land her intp trouble some day.

Question.8. What thoughts came to Sophie’s mind as she sat by the canal? (All India 2011)
Answer. Sophie felt doubts stirring inside her, as she sat by the canal waiting for Danny Casey. When she saw no sign of him, she remembered Geoff’s words that Casey would not come. She wonders what she would tell her family. She thinks that Geoff would be disappointed. She is saddened by the fact that she will never be able to show her family that they are wrong to cast doubts on her.

Question.9. Which was the only occasion when Sophie got to see Danny Casey in person?
(All India 2011)
Answer. The only occasion when Sophie got to see Danny Casey in person was when she went to watch the match with her family. Sitting amongst the spectators, Sophie saw Casey from a distance. The power of her imagination was such that she concocted a story of her brief encounter with him and almost got an autograph from him.

Question.10.Why did Jansie discourage Sophie from entertaining thoughts about the sports- star,
Danny Casey? (All India 2011)
Answer. Jansie was a realist, and not a daydreamer like Sophie. She discouraged Sophie from having such dreams because her dreams were wild and impossible. She had neither the means nor the skills to achieve them. She feels sad because she knows that both of them will have to work at the biscuit factory after they pass out of school. It was preposterous for her to live in such a dream world and it could lead to depression or low self-esteem.

Question.11.Why did Sophie wriggle when Geoff told her father that she had met Danny Casey?
(Foreign 2011)
Answer. When Geoff told her father about Sophie’s chance encounter with Danny Casey, Sophie wriggled because she knew that her father wouldn’t believe it. Her father was a practical, realistic person and Sophie was sure that he would be angry with her and reprimand her for concocting such stories. She feared that she would have to add more details to the story to make it sound authentic.

Question.12.”Damn that Geoff, this was Geoff thing, not a Jansie thing,” Why did Sophie say so?
(Delhi 2010)
Answer. Sophie knew that her classmate Jansie poked her nose into every matter, She was a rumour monger. Jansie came to know about the Sophie-Casey encounter through Frank, her brother, who was friends with Geoff. Sophie was annoyed with Geoff because this story was meant only for him. Sophie knew that Jansie would spread this information like wildfire and it could draw lots of people to her house to enquire about Casey.

Question.13.Does Geoff believe what Sophie says about her meeting with Danny Casey?
(All India 2010)
Answer. Geoff does not believe Sophie’s story about her meeting with Danny Casey, the Irish football star. Although he had a strong bond with his sister, as she shared all her secrets with him, he dismissed the whole story as a most unlikely thing. He does not hesitate to tell her that Casey would never keep his promise of meeting her again to give her his autograph.

Question.14.What did Sophie imagine about her meeting with Danny Casey? (All India 2008)
Answer. Sophie imagines that she has met Danny Casey, the Irish football prodigy, at Royce’s. She cooks up details about his appearance and says that she asked him for an autograph but could not get it as neither of them had paper or pen. They talked for a while about the clothes in the shop. Casey promised to meet her again next week.

Long Answer Type Questions (6 Marks, 120-150 words)

Question.1. Has Sophie met Danny Casey? What details of her meeting with Danny Casey did
she narrate to her brother? (Delhi 2014 Modified)
or
Did Sophie really meet Danny Casey? (NCERT)
Answer. No Sophie has never re&lly met Danny Casey. She has just seen Danny Casey on the field when she went to watch team United play a football match. There, she was one of the spectators and could see Casey playing only from a distance.
However, Sophie lives in an imaginary world. Casey was a young sportstar and Sophie worshipped him as her hero. Her meeting with Casey was just another of her wild fantasies. She was so engrossed in her daydreams that what she imagined seemed real to her.
Sophie told her brother Geoff that she met Casey near Royce’s. As she was looking at the clothes, Casey came and stood beside her. She wanted to take his autograph for her younger brother’Derek, but neither of them had a pen or paper. Casey then suggested that they could meet again next week, and then he would also give his autograph.

Question.2. What were Sophie’s plans for her future? Why would you call her dreams unrealistic?
(Delhi 2014; Modified)
Answer. Sophie planned to open a boutique after she passed out of school. When Jansie told her that it would require a lot of money, she said that she would become a manager and save money ! for her boutique. She later also added that she would either become an actress or a fashion designer. For a girl from a lower middle class family such dreams are certainly far from reality. Sophie’s friend Jansie, who is quite practical, knows that they will ultimately have to work in the biscuit factory. She even dissuades Sophie from indulging in such childish fantasies. Sophie wants to materialise all those things which were simply not possible practically. Never for once in the story does the girl think practically or come out of her dream world.

Question.3. Sophie was a dreamer. The lesson ‘Going Places’ reminds us that mere dreams will
not help us to accomplish anything. What qualities, do you think, would help Sophie to realise her dreams? (Foreign 2014)
Answer. Sophie lives in a world of dreams, which is far from reality. It is not unreasonable to have high hopes and ambitions for one’s future, but such dreaming can be justified only when one is prepared to work hard to realise one’s dreams.
Sophie too needs to work hard to achieve her dreams, instead of just imagining a bright and successful future for herself, or fantasising about her meeting with Danny Casey. She should accept the reality that she belongs to a middle class working family. Initially, she would face some difficulties because of her poor financial condition, but if she persists with sincere devotion, she could certainly raise her status to the level from where she would actually have the company of successful .people like Danny Casey. Besides, with better educational qualifications, increased opportunities, hard work and more savings, she could even own a boutique in futufe.

Question.4. Attempt a character sketch of Sophie as a woman who lives in her dreams. (Delhi 2012)
Answer. Sophie has been portrayed as the central character in the story ‘Going Places’. She perfectly represents the girls of her age who live in poor families.
Sophie always lives in a dream world, dreaming impossible things. The opening scene of the story clearly tells what sort of girl she is. She is not ready to accept the reality of her family’s condition and dreams of having a boutique of her own.
She makes up the story of meeting Danny Casey, a charming and upcoming footballer. Nobody believes her but she refuses to accept that it is her dream. Rather, she starts believing that she has met him and to prove that she is telling the truth, she makes up another story that she has fixed a date with him.
She is so lost in her dreams that she actually goes to the canal and waits for him. He does not show up. She knows that he will not come, but still she becomes sad. Such is the character of Sophie.

Question.5.Contrast Sophie’s real world with her fantasies. (Delhi 2009)
Answer. The story ‘Going Places’ is a journey into the dream world of the protagonist, a schoolgirl named Sophie who drifts from one dream to another. The world of dreams is so fascinating for her that even her wildest thoughts can find refuge it.
Sophie finds the reality of her life too harsh to accept. She belongs to a lower middle class family where nobody encourages her to dream of better prospects. Still she dreams of owning the best boutique, becoming a manager in some store or at other times becoming an actress or a fashion designer.
In her dream world, she visits the Royce’s, meets Danny Casey, the sports icon, and even fixes up a date with him. It’s all in her mind but she considers it true and actually waits for him to show up at a specified spot and time. The flight of her imagination is such that it flips from one dream to another, from one place to the other.

Important Questions for Class 12 English

CBSE previous Year Solved Papers Class 12 English Delhi 2013

CBSE previous Year Solved  Papers  Class 12 English Delhi 2013

Time allowed : 3 hours                                                                                           Maximum Marks: 100
General Instructions :

  1. This paper is divided into three sections : A, B and C. All the sections are compulsory.
  2. Separate instructions are given with each section and question, wherever necessary. Read these instructions very carefully and follow them faithfully.
  3. Do not exceed the prescribed word limit while answering the questions.

SET-I

SECTION- B
(ADVANCED WRITING SKILLS)

Question.3. You are Smitha/Sunil, Secretary AVM Housing Society. You going to organize a blood donation camp. Write a notice in not more than 50 words, urging the members of your society to come in large numbers for this noble cause. Invent all the necessary details.
OR
You are General Manager, Hotel Dosa, Gurgaon. You need a lady Front Office Assistant with sound‘knowledge of computers. She must be a graduate and good in communication skills with pleasing manners. Draft an advertisement in not more than 50 words to be published in Gurgaon Times. [10]
Answer:
Avm Housing Society
Notice
Blood Donation Camp 26th October, 2014
A blood donation camp is being organized by our society • on 1st November, 2014 in the front park of the society. All the residents are therefore, requested to donate their blood for a noble cause and save a life. For any details please contact the undersigned.
Smitha
Secretary
AVM Housing Society
OR
REQUIRED
A lady Front Office Assistant, having sound knowledge of computers, a graduate with good communication skills and pleasing manners. Salary negotiable. Interested may apply immediately
Contact: General Manger Hotel Dosa, Gurgaon

Question.4. Your school Sun Public School, Poona celebrated ‘Environment Day’ on 5th November. Write a report on the programme in 100-125 words for your school newsletter. You are Neeta/Naveen, Cultural Secretary of the school. Invent the necessary details.
OR
You are Deepak/Deepika, Secretary of Ahimsa Club, Parsva Public School, Delhi. On 2nd October your school observed ‘International Day for peace and Non-violence’, organizing various activities Such as visit to Rajghat, Charkha spinning, lectures by eminent Gandhians, etc. Write a report for the same in 100-125 words.
Answer:
Environment Day Celebration
Report By : Neeta (Cultural Secretary)
Sun Public School, Poona, celebrated ‘Environment Day’ on 5th November very enthusiastically. Everybody was excited and showed eagerness towards the celebration. The students wore eco friendly dresses. The corridors were decorated with the posters made by the children with the help of their art teacher. Many activties were organized on this occasion. A small play was presented by the students in connection with the environment friendly measures which should be adopted by the people. There was also an exhibition of the things made by the junior section of the school, which included paper bags, jute bags, and products made up of biodegradable matter. Our Principal gave an inspiring speech on how to preserve our environment. Student speakers also presented their views on the conservation of environment. After the function was over, tree plantation drive started. The first tree was planted by the Principal Madam followed by the faculty members and then the children. In the end students took the oath of planting a tree every year to make the environment neat and clean.
It was a great step towards the awareness about the environment.
OR
International Day For Peace And Non-Violence Observed,
REPORT BY : DEEPIKA (Secretary, Ahimsa Club)
2nd October was a memorable day for our school as it observed ‘International day for Peace and Non-violence.’ We all know that this day is celebrated to commemorate Mahatma Gandhi, who was the greatest messenger of peace and non¬violence. Various activities were organized by the school. Students were told the importance of ‘Charkha’ Spinning’ and they enjoyed doing it by themselves. At the back stage a group of students sang Bapu’s favourite bhajan ‘Vaishnav jan to tinhe kahiye…’ in a choir. There were speeches made on the way of living, that Gandhiji advocated all his life and the means to achieve them which according to Bapu should be noble and high. After that the children were taken to the Samadhi of Gandhiji known as ‘Rajghat’. The teachers as well as children paid homage to Bapu. The eminent Gandhians delivered speeches and told that he was a great example of selfless service, self-discipline, renunciation and deep love for suffering humanity.
In the evening, we returned back to the school to disperse to our homes but there was content and’ peace in our mind and soul.

Question.5. As a regular commuter by bus from Noida to Delhi, you have been witnessing rash driving by the bus drivers daily without an exception Write a letter to the editor “The Times of India” drawing the attention of the General Manager Delhi Transport Corporation to this problem. You are Priti/ Prakash, 15, UdyogVihar, Noida.
OR
Write a letter to Light ways Sport, Amrapalli, Thane,, placing an order for sports articles (Minimum 4) to be supplied to your school, ABC Matriculation School, Civil Lines, Poona, Sign as Ravi/Rayeena, Sports Secretary. 
Answer:
15, UdyogVihar
Noida
24th September, 2014 The Editor,
The Times of India Subject: Rash Bus Driving
Sir,
Through the columns of your esteemed newspaper, I would like to draw the attention of the General Manager, Delhi Transport Corporation towards the increasing cases of rash bus driving. This has created havoc in the city as it has become a routine habit of the drivers. Daily one or two cases of mishaps, due to the negligence of the drivers are coming into light. Its not an overheard thing, btit I have experienced it myself as I am a regular commuter by bus from Noida to Delhi. The number of accidents have been increasing daily which results in the loss of many lives. This should be checked as, drivers are indifferent towards the sufferings of the public and do not pay heed towards the traffic rules. The most common sufferers are the elders and the children. Immediate action is the need of the hour. Either the registration of their vehicle should be cancelled or their driving license, to bring such mishaps under control.
I expect a favorable response.
Thank you Yours Sincerely ,
Priti.
OR
ABC Matriculation School, Civil Lines, Poona 26th October, 2014 The Manager Light ways Sport,
Amrapali, Thane
Subject: Order for Sports Articles
Dear Sir/Madam,
We need to purchase some sports articles for our school. The list of the articles is as follows :
: Item
1. Basketball
2. Cricket bats
3. Volleyball
4. Football
Kindly send the articles after proper checking at the school sports complex between 10. a.m. to 2. p.m. Please make the delivery as soon as possible. As we have been a regular buyer of sports goods from your shop for past many years, a good amount of discount is expected on the above mentioned items. Payment will be done at the time of delivery through demand draft.
Thank You.
Yours Sincerely,
Raveena.
(Sports Secretary)

Question.6. Your family has recently shifted from Kota in Rajasthan to Ernakulam in Kerala, where your house is situated in the midst of beautiful flowering plants and fruit-yielding trees. Every minute and every second, you are experiencing the joy ofbeingin the lap of nature. Write an article in 150-200 words on the diversity of nature that you have experienced. You are Latha/Lalitha of class XII. 
OR
Write an article in 150-200 words on the topic, ‘Poverty is the cause of all evils’, to be published in the Young World of‘The Hindu’, Chennai.
Answer :
In The Lap of Nature
By : Latha Class XII
One can find huge diversity in nature while living in India. This, I experienced personally when we shifted to Ernakulam in Kerala from Kota in Rajasthan. It seemed as if we have shifted not places but nature. From the brown mountains to the green canopies, the change was completely welcomed by all the family members. For the first time in my whole life, I saw such greenery. The words used for Kerala as ‘God’s Own Country’, appeared to be true which I had heard or read till now. Living in the lap of nature, is a completely new experience. The greenery is extremely enchanting and to add to its beauty, is the location of my house amidst the beautiful flowering plants and fruit yielding trees. The fragrance of the blooming flowers fill the house and the aroma enters the soul. The view of fruit laden trees is equally fascinating: There is no sign of pollution here. Evergreen beaches, fishing nets spread all over the sea, richness of coconut and jackfruit, all make it a desired place to live in. The place has not lacked even in the preservation of its culture. The boat race in the backwaters is done during the festival of Onam. It is famous for its spices and ‘Ayurvedic Kerala Massage.’ In all, I must say that I enjoy the bracing environment, relaxed life, temples, backwaters and shaded woods of Kerala.
OR
Poverty is the Cause of All Evils
By: Xyz
India is a country where most of the population lives in villages and mostly below poverty line. Poverty prevails not only in villages but in big cities also. The metros are full of slums, where majority of people are living in inhuman conditions. Their basic needs for food, shelter and clothing are not fulfilled. Millions and millions of people are the victims of malnutrition, hunger, poor health and illiteracy Their condition is not better than stray dogs. The prices are rising at a very high rate and in the same way their condition is also deteriorating. The rich are becoming richer, and the poor # are becoming poorer. Circumstances and negligence force them to adopt evil means and criminal activities like robbery, murder, kidnapping, looting and so bn. Children are made to beg and women are compelled to become prostitutes. If it is not acceptable to anyone, then they commit mass suicide. The society, reformatory organizations, government, and non-profit organizations should take a step forward to improvise the condition of these people. There should be a check on the fund allocated, for poverty removal and whether it is properly utilized or not.

SECTION- C 

Question.7. Read the extract given below and answer the questions that follow:
Perhaps the Earth can teach us As when everything seems dead And later proves to be alive Now I’ll count up to twelve And you keep quiet and I will go.
OR
A flowery band to bind us to the Earth,
Spite of despondence, of the inhuman dearth of noble natures, of the gloomy days,
Of all the unhealthy and o’er-darkened ways made for our searching: ”
(i) What does the Earth teach us ?
Answer.The Earth teaches us to remain quiet even in the adverse situations but inside the change goes on non-stop.
(ii) What does the poet mean to achieve by counting up to. twelve ?
Answer.The poet, asks us to keep quiet while he counts up to twelve as he wants us to self-analyze for the welfare of the society.
(iii) What is the significance of ‘keeping quiet’?
Answer.lt is quite necessary to keep quiet for some time as it would help us retrospect and understand ourselves, then only we can solve our problems.
OR
(i) What are we doing every day ?
Answer : Every day we are weaving a beautiful string of flowers that would bind us to earth, and motivate us to live our life happily instead of pains and sufferings.
(ii) Which evil thing do we possess and suffer from ?
Answer : We possess evils of hatred and jealousy that do not let us live freely and happily. They steal away the joys of life.
(iii) What are the cirumstances that contribute towards making humans unhappy and disillusioned with life ?
Answer : There is competition everywhere and the world is full of selfish people. There is dearth of goodness and nobility in human nature. This makes us gloomy, unhappy and disillusioned with life.

Question.8. Answer any three of the following questions in 30-40 words each :
(a) Why has the poet’s mother been compared to the ‘Late winter’s moon’ ?
Answer : The poet’s mother has been compared to the ‘Late winters moon because she is too old and tired in the same way as the winters moon is dull and without shine.
(b) What message does Stephen Spender convey through the poem ‘An Elementary school Classroom ina Slum’?
Answer : The poet conveys the message that the life of the children living in slum is limited to their filthy dark room only. They have no future or hope and they don’t know the outer world. This problem can be solved only when a school inspector, educationist or a governor comes here.
(c) What was the plea of the folk who had put up„the roadside stand ?
Answer : The men who had put up the roadside stand, pleaded in a pitiable manner to customers as they wanted to earn some money. They wished the city folks would stop there to by something from them.
(d) What will happen to Aunt Jennifer’s tigers when she is dead ?
Answer : After Aunt Jennifer’s death, she will enter another world and her plight would remain same but her tigers would lead a fearless life and walk with grace, elegance and confidence.

Question.9. Answer the following questions in 30-40 words each :
(a) How did Franz react to the declaration that it was their last French lesson ?
Answer : Franz was shocked to hear that there would be no French teaching from now onwards and felt extremely sorry for neglecting his lessons, and regretted for not having the chance to learn grammar rules and participles. His French teacher M. Hamel would also be seen no more.
(b) How did the instructor turn douglas in to a swimmer ?
Answer : The instructor taught Douglas swimming step by step. He told him to exercise daily and practice making his limbs and hands stronger. He also taught him to exhale and inhale outside and inside water. He trained him perfectly by teaching all the strokes of swimming and thus turned Douglas into a swimmer.
(c) Why did Sophie like her brother Geoff more than any other person ?
Answer : Sophie’s brother Geoff was the only person in her life to whom she could confide. He was an introvert who used to speak very little and so she thought that he would never let any of her secret out. She felt, that she would be safe with him anywhere in this strange world.

Question.10. Answer the following in 125-150 words :
How did the peddler feel after robbing the crofter ? What course did he adopt and how did he react to the new situation ? What does his reaction reveal ?
Answer : The peddler robbed thrity kronor from crofter’s house, who gave him shelter for the whole night in his cottage showing him kindness and hospitality. The peddler felt very happy after that and believed, that it was not safe to walk along the public highway. S6 he went into the woods but lost his way and when he could not get out of it, he thought the whole world, with its lands and seas, its cities sand villages is nothing but a big ‘rattrap’ and he was the victim of if. He felt that the bait in his case was the money that he had stolen from crofter’s house. It was now his turn to be caught in the world’s rattrap. Whenever we become the victim to temptations’, we get trapped in this vicious circle.
Peddler could not help but think, and was trying to prove his act of robbery that crofter had put the money to trap him and given to temptation, he was also caught. He was not the only one who was tempted, but almost everyone; some or the other day gets trapped.

Question.11. Answer the following in about 100 words:
Exploitation is a universal phenomenon. The poor indigo farmers were exploited by the British landlords to which Gandhiji objected. Even after our independence we find exploitation of unorganized labor .
What values do we learn from Gandhiji’s campaign to counter the present day problems of exploitation ?
Answer : The conditions that prevailed during the time of Gandhiji are still existing. The weak and the poor are living in the stpne condition as they used to, prior the to independence, only the ways of exploitation have altered. The poor sharecroppers were exploited by the British landlords and their lands were snatched from them. Gandhiji, fought for them systematically by gathering information and presenting the facts courageously and managed to get justice for the fear stricken peasants. The striking feature of this was that he did not use any form of violence or unfair means.
These methods can be adopted by us also, as we must be aware of our strength and weaknesses and then proceed accordingly ‘ as one should not fall into the trap by any means. We should never compromise at the cost of our dignity or self-respect and thus sought out the solutions wisely.

Question. 12. Answer the following in 125-150 words :
What is the bond that unites the two-the old Mr. Lamb and Derry, the small boy ? How does the old man inspire the small boy ?
OR
Do you think Dr. Sadao’s final decision was the best possible one in the circumstances ? Why/why not ? Explain with reference to the story ‘The Enemy’.
Answer : Derry and Mr. Lamb had the same sense of loneliness and they both lived, a secluded life due to their physical inability. Derry experienced Mr. Lamb to be a peculiar person saying strange things which he was not able to understand, still he felt some kind of attraction towards him. He was differently-abled but was calm and maintained peace with people around him. He was not irritated by the children who called him ‘Lamey-Lamb!. Before coming in, contact with Mr. Lamb, Derry felt lonely due to the behavior of people but lamb inspired him and filled him with self¬confidence and determination.- He taught him to live life to the fullest without brooding over his burnt face. He told him to move ahead in life as his brain and senses were working completely. He imbibed positive thinking in him and showed him the way to live life in spite of his inability because he could do much better than others. This was the bond that united both and Derry wept over Mr. Lamb’s death.
OR
Answer : Yes, Dr. Sadao’s final decision was the best possible , one in the given circumstances because, he was hiding an enemy in his house and everybody was aware of it. Although he had no love for Americans and he was a true patriot still ‘ he saved him realizing his ‘sacred’ duty to save a dying man. This , brought an open defiance from his domestic staff and his wife had to do all the household work. Moreover, there was a high risk as he was sheltering an enemy. So when he felt, that the man was out of danger, he helped him to escape.
He got a boat ready with food, water and extra clothing and asked the soldier to row the boat to a little island that was not too far from the coast and from there he could take a Korean fishing boat. One evening, there was no signaling and Dr. “* Sadao understood that the man was off safely on a Korean fishing boat, and it was then that he had a sound sleep.

Question.13. Answer the following in 30-40 words :
(a) What sort of hunts did the Maharaja offer to organize for the high-ranking British officer ? What trait of the officer does it reveal ?
Answer : The Maharaja offered to organize every sort of hunt for the high ranking British Officer which also included boar hunt, mouse hunt and so on, but refused to entertain the tiger hunt.
The British officer himself was not interested in killing the tiger, he only wanted his photograph Beside the body of the dead tiger with a gun in his hand. It reveals that the British officer was a boastor and self-publicist.
(b) What did Jo want the wizard to do when Mommy Skunk approached him ?
Answer: Jo, wanted the wizard to hit Mommy Skunk because she wanted Roger Skunk to smell foul like before and that’s why she took him to the wizard. There she hit him with the umbrella and Jo disliked it as according to her, the wizard was right.

SET-II

SECTION- B
(ADVANCED WRITING SKILLS)

Question.4. You are General Kanpur Senior Secondary School, Kanpur celebrate ‘No Tobaco Day’ on 4th September Write a report in 100-125 words on the activities performed for your school newsletter. You Shobita/Sameer, cultural Secretary of your school. . Invent the necessary details.
OR
As a staff reporter of The Times of India, Delhi, you are asked to cover an incident of daylight robbery on the outskirts of Delhi, While the inmates were present in the house. Write a report in 100-125 words.
Answer:
NO TOBACCO DAY
REPORT BY: SAMEER (Cultural Secretary)
4th June 20xx was celebrated as ‘No Tobacco Day by our school. An interesting and educative programme was organised to highlight the evil effects of tobacco on human body. The programme was inaugurated by the Director of Education. Students had exhibited pictures, posters and cartoons depicting the harms of smoking. A demonstration was presented to highlight the miserable state of tobacco addicts. The messages ‘No Smoking’ and ‘Smoking is injurious to health’ were preached quiet overtly. Then, the school doctor gave an illuminating discourse, on the harms caused by tobacco. He used slides, graphs and pictures to illuminate his talk. A short film, was also screened to drive home the menace caused by tobacco smoking or chewing. The plight of the patients of cancer, caused by tobacco, made us shudder. We, resolved not to smoke or chew tobacco.
OR
A DAYLIGHT ROBBERY
REPORT : XYZ (TOI, CORRESPONDENT)
New Delhi : 2nd March, 20xx
A daring daylight robbery occured today in the house of Mr. William at 2.00 p.m. After lunch, the inmates were enjoying their afternoon siesta, when a group of marked young men entered the house, from the roof top. At first, they shut all the doors from inside and bolted the main gate from within.
They cut off the telephone connection and then pointed their gun at the chest of Mr. William and demanded the keys of the safe, in which ornaments, cash, valuables and other important documents were kept. The head of the family, was helpless and had to submit. According, to Mr. William the robbers decamped with all the cash and jewellery, they could, when a sudden knock, at the main gate by the Postman, made the robbers beat a hasty retreat, thank God, there was no loss of life. Personal rivalry is assumed to be one of the motives behind the crime.

Question.5. Your are Kamini/Kamal of 10 Civil Lines Extension, Chanakyapuri, Agra The main road leading to your locality has three open manholes causing frequent accidents at night, especially during the rainy season. Write a letter to the Editor, The Times of India drawing attention of the Municipal Commissioner to this problem of the residents . OR
You are Anoop/Akriti, of Baden Powel Senior Secondary School, Delhi write a letter to the Sales Manager, Aparna Publishing House, placing an order for the books (minimum four tides) for your school library. Invent the necessary details.
Answer:
Civil lines Extension Chanakyapuri Agra.
20th June, 20xx The Editor The Times of India,
Subject: Open Manholes in the locality Dear Sir,
Through the columns qf your esteemed daily, I wish to draw the attention of the concerned authority to the problem caused by three open man-holes, on the main road of our locality. The number of accidents have increased due to these, specially, during the rainy season. School students are vulnerable to such situations. All the complaint letters written to the concerned authorities till date has been in vain. The man-holes need to be cemented at the earliest to avoid future mishaps Thank You
Yours Sincerely,
Kamini/Kamal
OR
Badan Power Sr. Secondary School
20th July, 20xx Sales Manager,
Aparna Publishing House Delhi
Subject: Order for Books
Dear Sir,
We have been purchasing books from you for the past ten years and hence we are placing an order for the following books for our school library.
Name Quantity
1. CBSE Complete Course English
Core Class XHth.
2. CBSE Political Science Textbook
Class XHth.
3, CBSE Biology Part I and II Textbook
Class XHth.
4. CBSE Home science Textbook Class 12
XHth.
Kindly send these books and provide us the necessary discount. Payment will be done by cheque upon receipt of the books.
Thank You
Yours Sincerely,
Anoop/Akriti

Question.9. Answer any three of the following question in 30-40 words Delhi each:
(a) How did Franz react to the declaration that it was their last French lesson ?
Answer: Franz is overcome with a feeling of remorse and regret when he heard that henceforth only German was to be taught in the schools of Alsace and Torraine. He wishes that he had spent so much time in trivial pursuit like seeking birds eggs, or going sliding in the saar. The books that seemed such a nuisance earlier appeared like old friend now.
(b) What made the lawyer lose his job ? What was about it ? (Poets and Pancakes)
Answer: The lawyer was a part of the story department, so when the boss shut it down he lost his job. Perhaps the only instance in all human history where a lawyer lost his job because the part were asked to go home.
(c) What according to the author, Tishani Doshi are the issues engaging public attention and interest during our time ?
Answer: Acc. to Tishani Doshi climate change is one of the most heartily contested environmental debates of our time. Will the west Antartic ice sheet melt etirely ? Will it be the end of the world as we know it.

SET III

SECTION- B
(ADVANCED WRITING SKILLS)

Note : Except for the following questions, all the remaining questions have been asked in previous sets.
Question.3. You are Smitha/Sunil, Secretary AVM Housing Society. You are going to organize a yoga camp. Write a notice in not more than 50 words, urging the members of your society to come in large numbers to attend the camp. Invent all the necessary details.
Answer:
AVM HOUSING SOCIETY, DELHI
NOTICE
REJUVENATION YOGA CAMP
1st MARCH, 2013
All residents are hereby informed that a yoga camp is being
organized from March 5, 2013 to April 5, 2013, at 6:00
am, at the Central Park of the Society. Residents from all age groups can participate in the camp. Please get yourselves registered at the earliest by contacting the undersigned.
Smitha
Secretary,
AVM Housing Society

Question.5. You have noticed many stray animals on the road during the busy hours of the day. These animals cause traffic jams as well as accients. You have already written to the concerned authorieis but no action has been taken so far. Write a letter to the Editor, The Hindu, drawing attention of the Muncipal Commissioner, Chennai. You are Shantha/ Suresh, 12 M.G. Road, Chennai.
OR
You are Nalini/Vishal, Hostel Warden, Zenith Public School, Kosikalan. Write a letter to the Sales Manager, Bharat Electronics and Domestic appliances Limited, New Delhi, placing an order for a few fens, microwave ovens and geysers that you wish to purchase for the hostel. Also ask for the discount permissible on the purchase.
Answer :
12 M.G. Road,
Chennai
13th December, 2014 The Editor The Hindu
Subjects : Havoc created by Stray Animals Dear Sir,
Through the columns of your esteemed newspaper, I would like to draw the attention, of the concerned authorities towards the havoc created by the stray animals in the city. These animals are left to roam during the busy hours of the day, resulting in huge traffic jams, due to which the public has to face a lot of problem. Sometimes, accidents also take place, as they appear suddenly in front of moving vehicles. The concerned authorities have already been notified but no action has been taken yet. The issue needs to be addressed at the earliest, in order to avoid any such inconvenience and mishaps.
Thank you
Yours Sincerely,
Shantha
OR
Zenith Public School Kosi Kalan
25th May, 2014 The Sales Manager
Bharat Electronics and Domestic Appliances Ltd.
New Delhi
Subject: Placing an order for Electronic Appliances Dear Sir,
We need to purchase some electronic items for the school hostel. The list of items is as follows :
Item
1. Ceiling fans
2. Microwave ovens 5
3. Geysers 6
All the items should be properly checked and delivered at the school hostel between 10 a.m. to 2 p.m. Please make sure to give the necessary discount permissible on every item. The payment will be done through cheque only after the delivery of the items.
Thank You Yours Sincerely,
Nalini
(Hostel Warden)

SECTION-C

Question.8. Answer the following in 30-40 words each:
(b) The poet says, ‘And yet, for these children, these windows, not this map, their world’. Which world do these children belong to? Which world is inaccessible to them ?
Answer:
Through these words, the poet is conveying the message that the children dwelling in the slums have no idea of the outer world. They are living a dark, gloomy life, with no future or hope. The world map hanging in their classroom is meaningless to them. They have the view of the outer world only though these windows of the classroom. So, these windows should be much bigger, so that the children may be able to come out of them.

Question.9. Answer the following in 30-40 words each:
(a) WTtat did M. Hamel tell them about the French language?
What did he ask them to do and why ?
Answer:
It was, the last day of M. Hamel in the school and he told them, the importance of French language. He praised the language as the most beautiful and sweetest language of the world. If one keeps his language with him, he cah liberate himself from any type of slavery.

Solved CBSE Sample Papers for Class 12 Political Science Set 2

Solved CBSE Sample Papers for Class 12 Political Science Set 2

[Time Allowed : 3 hrs.]                                                                                              [Maximum Marks] : 100

General Instruction:

  1. All Questions are compulsory.
  2. Question numbers 1-5 are of 1 mark each. The answers to these questions should not exceed 20 words each.
  3. Question numbers 6-10 are of 2 marks each. The answers to these questions should not exceed 40 words each.
  4. Question numbers 11-16 are of 4 marks each. The answers to these questions should not exceed 100 words each.
  5. Question numbers 17-21 are of 5 marks each. The answers to this question should not exceed 150 words.
  6. Question numbers 22-27 are of 6 marks each. The answers to this question should not exceed 150 words.

Questions 1-5 (1 Mark), 6-10 (2 Marks), 11-16 (4 Marks), 17-21 (5 Marks), 22-27 (6 Marks)

Question .1. Fill in the blanks:
Cuban missiles crisis was on account of………placed in Cuba by………
Answer. Cuban missiles crisis was on account of nuclear missiles placed in Cuba by USSR.

Question .2. As a result of ‘shock-therapy’, to which economic system, each state of the Soviet bloc was gradually to be absorbed ?
Answer. As a result of ‘Shock-therapy’ each state of the Soviet bloc adopted the ‘Democratic capitalist system’. Shock-therapy was the model of transition from Authoritarian socialist system to a Democratic capitalist system.

Question .3. Correct the following statement and rewrite :
In the UN Security Council, the five permanent members are : China, USA, Russia, Australia, UK.
Answer. In the UN Security Council, the five permanent members are: USA, France, Britain, China and Russia.

Question .4. Give the names of two provinces which were also divided during the partition of India.
Answer. Punjab and Bengal were the two provinces which were also divided during the partition of India. This caused the deepest trauma of partition.

Question .5. Name one pact of eastern alliances and three pacts of western alliances during cold war era.
Answer. One pact of eastern alliances was – Warsaw Pact. It was led by the Soviet Union and was created in 1955.
Three pacts of western alliances are :

  • SEATO – South East Asian Treaty Organisation, signed in 1954.
  • CENTO – Central Treaty Organisation, signed in 1955.
  • NATO – North Atlantic Treaty Organisation, formed in 1949.

Question .6. How did the election of sixteenth Lok Sabha 2014 show the landmark to the Indian Electoral Politics?
Answer. The sixteenth Lok Sabha Election of 2014 showed a historic landmark as the BJP led NDA claimed a landslide victory making huge gains across the country. Out of 543 seats NDA took to win 336 seats and incredibly the BJP crossed 282 seats on its own without any allies.
This is the biggest victory since 1984 elections when Rajiv Gandhi won with 415 Lok Sabha seats. Even it is also the first time ever in the 67 years history of independent India that a non-congress party had won a simple majority of its own.

Question .7. What do ASEAN and FTA stand for ?
Answer.

  • ASEAN stands for Association.of South East Asian Nations. It was established in 1967 by five countries of this region— Indonesia, Malaysia, The Philippines, Singapore and Thailand, with the objective of accelerating economic growth followed by social progress and cultural development.
  • FTA stands for Free Trade Area for investment, labour, and services. It was created by ASEAN.

Question .8. Define cultural heterogenisation.
Answer. Cultural Heterogenisation is a concept which signifies cultural differences and distinctive nature of culture. This phenomenon is generated by the process of globalisation.
This is the negative cultural implication of globalisation which leads to fear and poses a threat to cultures in the world.

Question .9. Mahatma Gandhi said on 14th August, 1947, “Tomorrow will be a day of rejoicing as well as mourning”.
According to Mahatma Gandhi, why would 15th August, 1947 (tomorrow) be a day of rejoicing as well as mourning ?
Answer. According to Mahatma Gandhi 15th August, 1947 would be a day of rejoicing as well as mourning because the much awaited freedom, /.e., the matter of rejoicing came with the partition of the country, a cause for mourning.
The year 1947 was a year of unprecedented violence and trauma of displacement. All these signified mourning at the time of independence.

Question .10. State any two Directive Principles of State Policy relating to foreign affairs.
Answer. In the Indian Constitution Article 51 deals with the Directive Principles of State Policy on
promotion of international peace and security. The two principles are as follows:

  1. Maintain just and honourable relations between nations.
  2. Foster respect for international law and treaty obligations in the dealings of organised people with one another.

Question .11. Why were most of the non-aligned countries categorised as LDC-Least Developed Countries?
Answer. Most of the non-aligned countries were categorised as Least Developed Countries because of the following reasons :
Low Economic Development: The economic development of these countries was very low and poverty level was very high. So their main challenge was to develop economically and lift the people out of poverty.
Limited Role in International Economic Institutions: The non-aligned countries had a very minimal role in International Economic Institutions which also became a factor responsible for being Least Developed Countries.
Absence of Sustained Economic Development: Most of the newly independent non-aligned countries remained dependent on the richer countries including the colonial powers from which political freedom had been achieved. Thus, without sustained development, a country could not be truly free.
Exploitation of Natural Resources by Developed Countries: Natural resources of most of the Non-aligned countries were exploited by the developed western countries. Thus, NAM countries remained the Least Developed Countries.

Question .12. Mention any four common features of European Union.
Answer. European Union is a group of European capitalist countries established in 1992. It was founded for a common foreign and security policy, cooperation injustice and home affairs.

  1. European Union functions as an important bloc in international organisations such as the World Trade Organisation. In this way, it is able to intervene in economic areas.
  2. The European Union has tried to expand areas of cooperation while acquiring new members especially from the erstwhile Soviet Bloc.
  3. At the same time European Union has a great influence in the world arena, and on some of the UN policies because its two members, Britain and France, hold permanent seats in the UN Security Council.
  4. European Union’s another important feature is its very effective influence in the areas of diplomacy, economic investments and negotiations.

Question .13. Describe any four criteria that have been proposed for new permanent and non¬permanent members of the Security Council.
Answer. After the 1992 resolution over the reform in UN the following are some of the criteria that have been proposed for new permanent and non-permanent members of the Security Council. A new member should be :

  1. A major economic and military power. It must also be a big nation in terms of population.
  2. A substantial contributor to the UN budget.
  3. A promoter of democracy and human rights.
  4. Above all, a country that would help the council to become more representative of the world’s diversity in terms of geography, economic systems and culture.

Question .14. Describe briefly any four problems faced in the process of partition of India.
Answer. In the Indian context the word ‘partition’ signifies the division of British India and creation of
Pakistan. The process of partition started in 1940 when the Muslim League propounded the
Two Nation Theory. Various problems and difficulties were involved in the process of partition.

  1. Problem of Religious Majority : Principle of religious majority was followed for the division, i.e., the arias where the Muslims were in majority would make up the territory of Pakistan and the rest was to stay with India. This created the feeling of communalism.
  2. Problem of East and West: There was no single belt of Muslim majority areas in British
    India. There were two areas of concentration, one in the East and other in the West. These regions were the Muslim majority provinces but there was no way to join these two parts. So it was decided that the new country, Pakistan, will comprise two territories, west and east Pakistan separated by a long expanse of Indian territory.
  3. Merger of NWFP : On the partition move not all Muslim majority areas wanted to be
    in Pakistan. Even Khan Abdul Ghaffar Khan, the undisputed leader of the North Western Frontier Province staunchly opposed the Two Nation Theory. But ultimately the NWFP was made to merge with Pakistan.
  4. Problem of Minorities : Another difference was the problem of minorities on both sides of border (East and West). Lakhs of Hindus and Sikhs in the areas that were now in Pakistan and an equally large number of Muslims on the Indian side of Punjab and Bengal found themselves trapped. The minorities on both sides of the border were left with no option except to leave their homes.

Question .15. What do the following relate to ?
(a) Jai Jawan, Jai Kissan (b) Garibi Hatao
(c) Indira Hatao                  (d) Grand Alliance
Answer.
(a) Jai Jawan, Jai Kissan : This slogan was given by the then Prime Minister Lai Bahadur Shastri in 1965. .
This slogan symbolised the country’s resolve to face the challenges of food crisis and external threat.
(b) Garibi Hatao : This slogan was given by the then Prime Minister Mrs. Indira Gandhi in 1970. It symbolized removal of poverty.
Through ‘Garibi Hatao’ Mrs. Gandhi tried to generate a support base among the disadvantaged, especially the landless labourers, dalits and adivasis, minorities, women and the unemployed youth.
(c) Indira Hatao: This slogan was given by the Grand Alliance which had only one common programme, i.e., Remove Indira Gandhi from the political arena.
(d) Grand Alliance: The grand alliance was formed by Non-Communist and Non-Congress parties. It was an electoral alliance formed against the Congress (R).
It did not have a coherent political programme rather it had only one common programme, i.e., Indira Hatao.

Question .16. Who were Dalit Panthers ? What did they stand for ?
Answer. Dalit Panthers were the Dalit communities who had experienced caste injustices for a long time. They formed a militant organisation by the name, Dalit Panthers in 1972 in Maharashtra.

  1. Dalit Panthers were mainly fighting against the perpetual caste-based inequalities and material injustices that the Dalits faced in spite of constitutional guarantees of equality and justice.
  2. The Dalits faced atrocities over minor symbolic issues of caste pride. So effective
    implementation of reservations and other such policies of social justice was their main demand.
  3. The Dalit Panthers resorted to mass action and their activities were mostly centred around fighting increasing atrocities on Dajits in various parts of the state. Legal mechanisms proved inadequate to stop the economic and social oppression of the Dalits.

Question .17. ‘Although India-Pakistan relations seem to be a of endemic conflict and violence both countries are trying to improve the relations, in a better way. “Analyse the passage.
Answer. In spite of sharing a bitter past based on endemic conflict and violence both India and Pakistan are trying to normalize the situation in the following ways:

  • The two countries have agreed to undertake confidence building measures (C8M) to reduce
    the risk.
  • Socialist/Social activists and prominent personalities have collaborated to create an atmosphere of friendship among the people of both countries.
  • Bus routes have been opened up between the two countries trade between the two parts of Punjab has increased substantially in the last five years.
    Thus the above passage justifies that there have been a series of efforts to manage tension and build peace, between the two countries.

Question .18. “The independent country must consist fundamentally and basically of foreign relation that is the test of independence. All else is local autonomy. Once foreign relations go out of you hands into the charge of somebody else to that extent and in that measure you are not independent”.
Read the passage and answer the following questions;

  1. Who gave the following statement and in what context ?
  2. What does the extract reveal ?

Answer.

  1. The above statement given by Pt. Jawarharlal Nehru in the context of Independent India and its foreign policy.
  • The above extract of Pt. Jawaharlal Nehru reveals the significance of non-alignment policy of India. It focusses on the power politics initiated by the two power blocs.
  • When India achieved its freedom and started forming its foreign policy, it followed non-aligned policy to pursue its national interest within the prevailing international context.

Question .19. Read the following passage and answer the questions below:
“Patel, the organisational man of the Congress, wanted to purge the Congress of other political groups and sought to make of it a cohesive and disciplined political party. He… sought to take the Congress away form its all-embracing character and turn it into a close-knit party of disciplined cadres. Being a ‘realist’ he looked more for discipline than for comprehension. While Gandhi took too romantic a view of “Carrying on the movement,” Patel’s idea of transforming the Congress into strictly political party with a single ideology and tight discipline showed an equal lack of understanding of the eclectic role that the Congress, as a government, was to be called upon to perform in the decades to follow.”
(a) Why does the author think that Congress should not have been a cohesive and disciplined party ?
(b) Give some examples of the eclectic role of the Congress party in the early years.
(c) Why does the author say that Gandhi’s view about Congress futures was romantic ?
Answer.
(a) The author thinks that Congress should not have been a cohesive and disciplined party
because he wanted to take the Congress away from its all embracing character and turn it into a close knit party of disciplined cadres.
(b) There are some examples of the “Electic role” of the Congress party in the early years.
(i) The congress party provided a ‘Platform’ for numerous groups, interests and even political parties to take part in the national movement.
(ii)The Congress party also presented a “Rainbow-like social coalition” broadly representing India’s diversity in terms of class and castes, religions and languages and various interests.
(c) The author says that Gandhiji’s view about Congress future was romantic because Gandhiji’s believed in the inclusive character of the National Movement led by the Congress which in turn enabled it to attract different sections, groups and interests.
And all these made the Congress a “broad based social and ideological coalition”.

Question .20. Identify the four princely states marked in the map given below as A, B, C and D. Mention
the major problem faced in the integration of any one of these states into the Indian Union.
solved-cbse-sample-papers-for-class-12-political-science-set-2-1
Answer.
A : Hyderabad B : Manipur
C : Junagarh   D : Jammu and Kashmir
A. Hyderabad: It was the largest princely state. The Nizam signed a ‘Standstill Agreement’ with India in Nov. 1947.
A movement was started against the Nizam supported by the communists and Hyderabad Congress but Nizam ordered a para-military force (Razakars) to suppress the people. The atrocities and brutal nature of Razakars led to the intervention of the Indian Army to control and liberate the people from Nizam’s control.
B. Manipur : The Maharaja of Manipur (Bodhachandra Singh) signed ‘The Instrument of Accession’ with the Government on the assurance that the internal autonomy of Manipur would be maintained.
The Manipur Congress wanted merger of Manipur with India but there were sharp differences in the legislative Assembly of Manipur.
C. Junagarh : It was a small state in Maharashtra (presently in Gujarat territory) where the ruler demanded an independent state but the people desired to join India.
D. Kashmir :

  1. It was ruled.by a Hindu ruler ‘Hari Singh’ who did not want to merge with
    India and tried to negotiate with both India and Pakistan to recognize Kashmir as an Independent State,
  2. Pakistan claimed Kashmir based on ‘Muslim’ majority and sent tribal infiltrators to capture Kashmir in 1947. This forced the Maharaja to ask/seek help from India. (Any one)

Question .21. What according to you is the message of the following cartoon? What do the two wheels in this bicycle represent ?
solved-cbse-sample-papers-for-class-12-political-science-set-2-2
Answer. The cartoon shows the economic transformation of communist China, i.e., before and after 1978.
China earlier restricted foreign trade and investment but now China extended her role in this aspect also. For example, China exports a large number of low-cost products to the capitalists (Western countries and Japan).
China is the largest user of bicycles which symbolise the dual economic policies of China. The front wheel represents the symbol of hammer and sickle adopted by Mao (Socialist policy) and the rear wheel represents the capitalist symbol of dollar (adopted by HU). Both the socialists and the capitalists co-exist in China now.

Question .22. What was India’s response to on-going cold war? What interest of India did non-alignment serve ?
Or
India’s relations with Russia are an important aspect of India’s foreign policy. How was it of great benefit to India ?
Answer. As a leader of Non-aligned Movement, India’s response to the ongoing cold war was two fold.

  1. At one level, it took particular care in staying away from two alliances.
  2. Secondly, it raised its voice against the newly decolonised countries becoming part of these alliances.

Non-alignment emerged from India’s initiative for formulating an independent foreign policy which was non-partisan.
So, non-alignment served India’s interest directly in two ways :

  1. Firstly, non-alignment allowed India to take international decisions and stances that served its interests. .
  2. Secondly, India was often able to balance one super power against the other. If India felt ignored or unduly pressurized by one super power, it could tilt towards the other. Neither alliance system could take India for granted or bully it.
    Or
    Indo-Russian relations are embedded in a history of trust and common interests and are matched by popular perceptions.
    Common view on the multipolar world order: Russia and India share a vision of multipolar world order. For both these countries a multipolar world order is the co-existence of several powers in the international system, collective security, greater regionalism, negotiated settlements of international conflicts, an independent foreign policy for all countries and decision making through bodies like the UN that should be strengthened, democratised and empowered.
    India’s stand towards Russia : India gets meaningful benefits for having healthy relations with Russia on the issu.es like Kashmir, energy supplies, sharing information on international terrorism, access to central Asia, and balancing its relations with China.
    Russia’s stand towards India : Like India, Russia stands to benefit from this relationship because India is the second largest arms market for Russia.
    Besides, Indian military gets most of its hardware from Russia. Since India is an oil importing nation, so Russia is important to India and has repeatedly come to the assistance of India during its oil crisis.
    In order to meet the demands of energy, India is trying to increase its energy imports from Russia and the republics of Kazakhistan and Turkmenistan. This also broadened the scope for partnership and investment in oilfields.
    India has also strengthened its relations with Russia for her nuclear energy plans and space industry. India gets the cryogenic rocket from Russia whenever needed.
    Thus, we may safely conclude that India has maintained good relations with all the post-communist countries. But the strongest relations are still those between Russia and India.

Question .23. Examine India-U.S. relations.
Or
Analyse the basis of projection of China to overtake the US as the world’s largest economy by 2040.
Answer. In the Post-Cold War Era, India and US share very harmonious relations based on mutual cooperation and understanding. During the cold war years India’s closest friendship was with the Soviet Union. But after the collapse of the Soviet Union India suddenly found itself friendless in an increasing hostile international environment.
During these years, India introduced a New Economic Policy to liberate its economy and integrate it with the global economy. The liberal economic policy and India’s impressive economic growth rate in recent years have made the country an attractive economic partner for a number of countries including the US. Other features of Indo-US relationship are :

  1. Technological dimension and role of Indo-American Diaspora is so interrelated that it gives interdependency in Indo-US relations.
  2. The US absorbs about 65 per cent of India’s total exports in the software sector.
  3. On the other side 35 per cent of the technical staff of Boeing is estimated to be Indian.
  4. More than 3,00,000 Indians work in Silicon Valley.

But in the phase of global hegemony India has yet to decide exactly what type of relationship it should have with the US. Three possible strategies have been suggested through debates by Indian analysts.

  1. Aloofness from US.
  2. Take advantage of US Hegemony.
  3. India should join hands to challenge US hegemony.
    By observing the above strategies we can conclude that Indo-US relations are perhaps too complex to be managed by a single strategy. Actually India needs to develop a mixed strategy in its foreign policy to deal with the US by maintaining its own identity in the global hegemony.
    Or

China emerged as the third alternative since its Economic Reforms of 1978. China introduced an open door policy and economic reforms in 1978. The open door policy, thus tried to generate higher productivity by investments in capital and technology from abroad. Hence, China has been the fastest growing economy since the reforms.
Now, it is projected to overtake the US as the world’s largest economy by 2040 on the following grounds:

  1. China’s economic integration into the region makes it the driver of East Asian growth, thereby giving it enormous influence in regional affairs.
  2. Besides, the strength of its economy, together with other factors such as population, land, mass resources, regional location and political influence have added to its power in significant ways and made it the third alternative power in the world.
  3. In this way, integration of China’s economy and the inter-dependencies have enabled it to have considerable influence with its trade partners. China is now moving towards global economy.

Question .24. What were the main considerations for bringing princely states with Indian Union? Who played the historic role in this task ?
Or
What was Green Revolution ? Examine any two positive and two negative consequences of Green Revolution.
Answer. The main considerations for bringing princely states with Indian Union were as follows:

  1. To shape India as a nation. India is a land of continental size. Hence, the first and the foremost consideration was the political unification and integration of the territory.
  2. The people of most of the princely states wanted to become a part of the Indian Union.
  3. The government was prepared to accommodate plurality and adopt a flexible approach in dealing with the demands of the regions.
  4. Another consideration was the concern towards the integration and consolidation of the territorial boundaries of the Indian nation.

Sardar Vallabhbhai Patel played a historic role in negotiating with the rulers of princely states. Sardar Vallabhbhai Patel was India’s Deputy Prime Minister and the Home Minister during the integration of Indian States. It was a very complicated task which required skilful persuasion.
For example, there were 26 small states in today’s Odisha, Saurashtra region of Gujarat had 14 big states, 119 small states and numerous other administrations. He played a very firm and diplomatic role in bringing rrjost of them into Indian Union.
Or
The term ‘Green Revolution’ refers to the new method adopted by the government in Agriculture in order to increase food grains production during 1960s.
The modern method includes the use of high yielding variety of seeds, use of fertilisers and pesticides, better irrigation methods, use of modern tools and highly subsidised prices.
Positive Consequences

  1. The region where Green Revolution was introduced like Punjab, Haryana and Western U.P. became prosperous. The use of modern technology made peasants skilled to operate in maximum areas of cultivation.
  2. Green Revolution led to easy availability of latest technology, good equipment, loans and other facilities like Crop Insurance Scheme, etc. Thus, the food grains production increased manifold and food security was ensured.

Negative Consequences

  1. Green Revolution increased polarisation between classes and regions. Some regions of North and Northwestern India like Punjab and Haryana became prosperous while rest of India remained backward.
  2. Green Revolution also created a sharp contrast between the rich and poor farmers. The illiterate peasants could not understand the system behind the Green Revolution.

Question .25. What does the term ‘syndicate’ mean in the context of the Congress Party of the sixties ? What role did the syndicate play in the Congress Party ?
Or
‘Governments that are perceived to be anti-democratic are severely punished by the voters’. Explain this statement with reference to emergency period 1975-77.
Answer. Syndicate was the informal name given to a group of powerful and influential leaders from within the Congress.

  • It was led by K. Kamraj, fdtmer Chief Minister of Tamil Nadu and the then President of the Congress Party. It included powerful leaders like S.K. Patil, S. Nijalingappa, N. Sanjeeva Reddy and Atulaya Ghosh.

Role of Syndicate in the Congress Party :

  1. In the sixties, Syndicate played a decisive role in the installation of both Lal Bahadur Shastri and Indira Gandhi as the Prime Ministers.
  2. This group had a decisive say in Indira Gandhi’s first Council of Ministers and also in policy formulation and implementation.
  3. But, the Syndicate lost its importance and prestige in 1971 after the Congress split. After the split the leaders of the syndicate and those owing allegiance to them stayed with the Congress (O) and other members joined Congress (R) led by Mrs. Indira Gandhi.
    Or

The given statement is justified because the emergency of 1975 at once brought out both weaknesses and the strength of India’s democracy. Many observers think that India ceased to be democratic during the emergency.

  1. It was a period of political crisis with changes in the party system. The party in power had absolute majority and yet its leadership decided to suspend the democratic process.
  2. And, the result of the 1977 election took everyone by surprise. For the first time since independence the Corfgress Party was defeated. It brought to end the one-party dominance. It opened the way for the opposition and the coalition type of governments.
  3. Basically, the most valid reasons for the defeat of the Congress Party was the people’s verdict which was decisively against Emergency. The opposition fought the election on the slogan of ‘Save Democracy’.
  4. The Janta Party made this election a referendum on the Emergency. Its campaign was focussed on the non-democratic character of the rule and on the various excesses that took place during this period.
  5. Thus, the 1977 elections turned into a referendum on the experience of the emergency and proved that ‘Governments that are perceived to be anti-democratic are severely punished by the voters.

Question .26. How do public or social movements in India strengthen democracy ? What are their limitations ?
Or
‘Assam movement was a combination of cultural pride and economic backwardness.’ Justify the statement.
Answer. To some extent public movements and protests in a country strengthen democracy. The history of movements and prbtests help us to understand better the nature of democratic politics.

  1. We have observed that these non-party movements like Anti-arrack Movement, Chipko Movement, NBA are neither sporadic in nature nor are these a problem.
  2. These movements came to rectify some problems in the functioning of party politics and should be seen as an integral part of our democratic politics.
  3. Popular Movements ensured effective representation of diverse groups and their demands. This reduced the possibility of deep social conflict and disaffection of these groups from democracy.
  4. Besides, popular movements suggested new forms of active participation in Indian democracy, for example, Anti-Arrack Movement and Dalit Panthers Movement.

Limitations:

  1. Yet the real impact of these movements on the nature of public policies seems to be very limited. This is partly because most of the contemporary movements focus on a single issue and represent the interest of one section of society, thus it is possible to ignore their demands.
  2. The collective actions of the movement like strikes, sit-ins, and rallies disrupt the functioning of the government, delay decision-making and destabilise democracy.

Conclusion : Keeping in view both limitations and importance, we can safely conclude that public movements are not only rallies or protests, they involve a gradual process of coming together of people with similar problems, similar demands and similar expectations. Social or public movements are also about making people aware of their rights and the expectations that they can have from democratic institutions.
Or
The Assam Movement was a combination of cultural pride and economic backwardness because it was against outsiders to maintain the cultural integration of Assam. The issue of – outsiders continues to be a live issue in the politics in Assam and many other places in the North-East.
Side by side, there was widespread poverty and unemployment in Assam, despite the existence of natural resources like oil, tea and coal. It was felt that these were drained out of the state without any commensurate benefit to the people.
Ultimately an Accord was signed between Rajiv Gandhi led government and AASU leaders over the issue of outsiders in 1985. According to this agreement those foreigners who migrated to Assam during and after Bangladesh war and since, were to be identified and deported.

Question .27. Bring out briefly the stages of major developments in international politics which prepared and shaped the non-aligned movement.
Or
‘Trace the growth and promotion of NAM through various conferences.
Answer. Following major developments in international politics shaped the non-aligned movement.

  1. Termination of second World War (1945-53): The first stage witnessed the termination of the war time alliances and blocs were formed by entering into military alliances and later Cold War politics started.
  2.  ” Principles of Panchsheel” (1953-61): The second staged found the world polarised, increase in nuclear arms race and increased possibility of mutually assured destruction. It was the beginning of the non-aligend movement and Non-Aligned countries were guided by the principles of “Panchsheel”.
  3. Liberation Movement (1961-65) : The third stage was a crucial period. There were Sino-Indian, Sino-Soviet differences, regional wars in Asia, rise of military regimes in the developing world, liberation of African colonies, tremendous increase in the membership of the UN. All these promoted a conducive environment for the existence of NAM.
  4. Conflict among Power Blocs (1965-71) : The fourth stage saw the rise of conflict among the bloc member and the decline of NATO, rise in the European community. The socialist world became more consolidated and there was increase in the role of the NAM.
  5. Beginning of Detente (1971-76) : Next stage saw the beginning of detente, Soviet rapprochement with the western world, normalisation of the Sino-US ties.
  6. Helsinki Accord: The sixth stage (1976-84) saw the alternative process of cooperation and confrontation between the east and the west, arms race and efforts at disarmament e.g. Helsinki accord.
  7. Process of Liberalisation and Democratisation (1985-89) : The seventh stage
    signifies the start of peaceful co-existence among the countries. The NAM at this time became more and more popular. Also, the USSR started the process of liberalisation and democratisation.
  8. Disintegration of USSR and end of the Cold War (since 1989): The eighth stage saw a new world scenario. The USSR was dismantled and 15 new republics were formed; both the parts of Germany were unified, Nambia became independent, aparthied ended in South Africa, peace efforts in west Asia increased, North south Dialogue started, and European Union was established, ALI these developments shaped the NAM in a every dignified manner.
    Or

Answer. Although the non-aligned movement formally come into being in 1961, its beginning can be traced back to the early post-World War II years. The important conferences through which the growth and promotion of NAM can be traced are as follows:
solved-cbse-sample-papers-for-class-12-political-science-set-2-3

  1. Asian Relations Conference— Asian Relations Conference was held in 1947 in Delhi. It was presided our by Jawahalal Nehru and was attended by 250 delegates from 25 countries.
    They talked about the linkage between peace, freedom, and development.
    This conference laid two foundation stone for NAM.
  2. The Bandung Conference—The Bandung Conference was the conference of Afro-Asian leaders held at Bandung in 1955. This conference gave an opportunity for common understanding of international problems.
  3. The Belgrade Conference— The Belgrade Summit was the first summit of Non-aligned movement which was held at Belgrade (Yugoslavia) in 1961. Twenty five countries participated in it. They discussed about disarmament, decolonisation and national liberation movements in Asia, Africa and Latin America.
  4. UN Declaration— In 1960, the UN adopted the historical Declaration on granting Independence to colonial countries and peoples. This session was attended by the our founder member/leaders of the NAM in which Jawaharlal Nehru said that the coming together of the leader of Asian and african states was an event of great importance.

In this declaration the leaders subscribed to the five principles of coexistence i.e., Panchsheel which was first enumerated in 1954 by Indian and china in their agreement. Henceforth of the non-aligend movement.
Other important Summits/Conferences 

  • The Conference held at Colombo in 1976 confronted with the problem of new applications for membership. It also emphasised that all nations should have full control over their national resources. It called for declaration of Indian Ocean as Zone of peace.
  • Another important conference was held at New Delhi in 1983 which was attended by a lot members. This conference stressed the need for a New international Economic order based on equity and sovereignty.
  • The eighth summit was held at Harare in 1986. In this summit Africa fund for frontline states was established.
  • The conference held at Belgrade in 1989 was equally important. ‘
  • In this conference emphasis was on a new international political order based on equality and justice.
  • Three more NAM summits Were held in the post-Cold War period. These were held at
  • Jakarta (Indonesia) 1992
  • Cartagena (Colombia) 1995
  • Durban (South Africa) 1998
  • Kuala Lumpur 2003
    In all these conferences and summits emphasis has shifted form political issues to economic questions such as poverty alleviation, New International Economic Order based on equality, equity and justice.
  1. Asian Relations Conference— Asian Relations Conference was held in 1947 in Delhi. It was presided over by Jawaharlal Nehru and was attended by 250 delegates from 25
    countries
    They talked about the linkage between peace, freedom, and development.
    This conference laid two foundation stone for NAM.
  2. Bandung Conference— The Bandung Conference was the conference of Afro-Asian leaders held at Bandung in 1955. This conference gave an opportunity for common understanding of international problems.
  3. Belgrade Summit— The Belgrade Summit was the first summit of Non-aligned movement which was held at Belgrade (Yugoslavia) in 1961. Twenty five countries participated in it. They discussed about disarmament, decolonisation and national liberation movements in Asia africa and Latin America.

 

CBSE previous Year Solved Papers Class 12 English Delhi 2010

CBSE previous Year Solved  Papers  Class 12 English  Delhi 2010

Time allowed : 3 hours                                                                                           Maximum Marks: 100
General Instructions :

  1. This paper is divided into three sections : A, B and C. All the sections are compulsory.
  2. Separate instructions are given with each section and question, wherever necessary. Read these instructions very carefully and follow them faithfully.
  3. Do not exceed the prescribed word limit while answering the questions.

SET I

SECTION —A
(READING)

Question.1. Read the passage given below and answer the questions that follow:

  1.  Today’s woman is a highly self-directed person, alive to the sense of her dignity and the importance of her functions in the private domestic domain and the public domain of the world of work. Women are rational in approach, careful in handling situations and want to do things as best as possible. The Fourth World Conference of Women, held in Beijing in September 1995, had emphasized that no enduring solution of society’s most threatening social, economic and political problems could be found without the participation and empowerment of the women. The 1995, World Summit for Social Development had also emphasized the pivotal role of women in eradicating poverty and mending the social fabric.
  2.  The Constitution of India had conferred on women equal rights and opportunities – political, social, educational and of employment – with men. Because of the oppressive traditions, superstitions, exploitation and corruption, a majority of women are not allowed to enjoy the rights and opportunities, bestowed on them. One of the major reasons for this state of affairs is the lack of literacy and awareness among women. Education is the main instrument through which we can narrow down, the prevailing inequality and accelerate the process of economic and political change in the status of women. ‘
  3.  The role of women in a society is very important. Women’s education is the key to a better life in the future. A recent World Bank study says that educating girls is not a charity, it is good economics and if developing nations are to eradicate poverty, they must educate the girls. The report says, that the economic and social returns on investments in education of the girls considerably affect the Human Development Index of the nation. Society would progress only if the status of women is respected and the presence of an educated woman in the family would ensure education of the family itself. Education and empowerment of women are closely related.
  4. Women’s education has not received due care and attention from the planners and policy makers. The National Commission for women has rightly pointed out that even after 50 years of Independence, women continue to be treated as .the single-largest group of backward citizens of India. The role of women in overall development has not been fully understood nor has it been given its full weight
    in the struggle to eliminate poverty, hunger, injustice and inequality at the national level. Even, when we are at the threshold of the 21st century, our society still discriminates against women in matters of their rights and privileges and prevents them from participating in the process of national and societal progress. Various committees and commissions have been constituted before and after Independence to evaluate the progress in women’s education and to suggest ways and means to enhance the status of women. The female literacy rate has gone up in the 20th century from 0.6 percent in 1901 to 39.29 percent in 1991 but India still possesses the largest number of illiterate women in the world. The Female Literacy Index for the year 1991 shows that there are eight states which fall below the national average. The most populous states of the country, UP, MP, Bihar and Rajasthan fall in the category of most backward states as far as female literacy is concerned. .
  5.  The prevailing cultural norms of gender behaviour and . the perceived domestic and reproductive roles of women tend to affect the education of the.girls. Negative attitude towards sending girls to schools, restrictions on their mobility, early marriage, poverty and illiteracy of parents affect the girls’ participation in education.
  6.  Women’s political empowerment got a big boost with the Panchayati Raj Act of 1993 which gave them 30 percent reservation in Village Panchayats, Block samities and Zila Parishads throughout the country. The National Commission for women was also set up in 1992, to act as a lobby for women’s issues.
  7.  The educational system is the only institution which can counteract the deep foundations of inequality of sexes that are built in the minds of people through the socialisation process. Education, is the most important instrument of human resource development. Educational system should be used to revolution is  the traditional attitudes and inculcate new values of equality.

(a) (i) Mention any two attributes of a modem woman.
Answer : (i) Women are careful in handling situations.
(ii) They are rational in their approach.
(ii) Why is women’s participation and empowerment
considered necessary ?
Answer : Society’s most threatening social, economic and political problem’s solution could not be found without the participation and empowerment of women.
(iii) Which factors adversely affect the education of
girls ?
Answer : The prevailing cultural norms of gender bias ness and the perceived domestic and reproductive roles of women tend to affect the education of the girls.
(iv) What benefits did the women get with the enactment of the Panchayat Raj Act of 1993 ?
Answer: The women got 30% reservation in village Panchayats, Block samities and Zila parishads, throughout the country with the enactment of the Panchayat Raj Act of 1993.
(v) By what process can we remove the sense of inequality of sexes from the minds of the people ?
Answer : The educational system is the only institution which can remove the sense of inequality of sexes from the minds of the people through the socialisation process.
(b) Pick out words from the passage which mean the same as each of the following:
(i) Gruel and unfair (para 2)
Answer: oppressive
(ii) Remove (para 3)
Answer: eradicate
(iii) full of people (para 4)
Answer: populous

Question.2. Read the passage given below and answer the questions that follow:
Despite all the research, each one of us catches cold and most of us catch it frequently. Our failure to control one of the commonest of all ailments, sometimes seems ridiculous. Medical Science regularly practices transplant surgery and has rid many a countries of such killing diseases as Typhus and the Plague. But the problem of common cold, is unusually difficult and much is yet to be done to solve it.
It is known that a cold is caused by one of a number of viral infections that affect the lining of the nose and other passage leading to the lungs but the confusing variety of viruses makes study and remedy very difficult. It was shown in 1960 that many typical colds in adults are caused by one or the other of a family of viruses known as rhinoviruses, yet there still remain many colds for which no virus has yet been isolated. There is also the difficulty that because they are so much smaller, than the bacteria, which causes many other infections, viruses cannot be seen with ordinary microscopes. Nor can they be cultivated easily in the bacteriologists laboratory, since they only grow within the living cells of animals or plants. An important recent step forward, however, is the development of the technique of tissue culture, in which bits of animal tissue are enabled and to multiply independently of the body. This has greatly aided virus research and has led to the discovery of a large number of viruses. Their ^existence had previously been not only unknown but even unsuspected. .
The fact that we can catch cold repeatedly creates another difficulty. Usually a virus strikes only once and leaves the victim immune to further attacks. Still we do not gain immunity from colds. Why ? It may possibly be due to the
fact that while other viruses get into the bloodstream where antibodies can oppose them — the viruses causing cold attack cells only on the surface. Or it may be that immunity from one of the many different viruses does not guarantee protection from all others. It seems, therefore, that we are likely to have to suffer colds for some time yet.
(a) On the basis of your reading of the above passage, make notes on it in points only, using abbreviations wherever necessary. Also suggest a suitable title.
Answer:
Title: Difficulties of Common Cold.
1. Notes
(i) Common cold
(a) everyone catches
(b) Med Sc. failed to control
(c) much has to be done in order to cure it.
(ii)causes
(a) viral infections
(b) variety of viruses
(c) family virus known as Rhino.
(iii)other problems
(a) too small to be seen under microscp
(b) leaves victim immune to further attacks
(c) can’t be cured by anti bods.
cbse-previous-year-solved-papers-class-12-english-outside-delhi-2010-1
(b) Write a summary of the above passage in about 80 words using the notes made.

SUMMARY

Everyone catches cold and the cure of it has not been found yet. Medical Science has failed to control it. It is caused by viral infections that affect the lining of the nose and other passages. The confusing variety of viruses, makes the study difficult. Family virus rhinovirus is also responsible for it. The viruses are so small that they can neither be seen with microscope nor cultivated in laboratory. This virus leaves the. victim immune to further attacks. They do not get into the bloodstream where antibodies can oppose them. Therefore, we have to suffer from cold for some time yet.

SECTION- B
(ADVANCED WRITING SKILLS)

Question.3. You are the Secretary of your School Literary Association. Write a notice in not more than 50 words for your school notice board, giving details of the proposed inauguration of the Literary Association of your school. You are ‘XYZ’ of Jain Vidyashram, Cuddalore. 
OR
You are the General Manager of E.V.L. Company which requires posh bungalows on company lease, as guest houses. Draft an advertisement in not more than 50 words under classified columns to be published in “The New Indian Express”1.
Answer:
cbse-previous-year-solved-papers-class-12-english-outside-delhi-2010-2
OR
cbse-previous-year-solved-papers-class-12-english-outside-delhi-2010-3

Question.4. You are Sweety/ Suresh of L.M. Jain School, Ajmer. As Secretary of your school’s Co-Curricular Activities Club, you visited a slum area in your city where the people suffered a great loss of life and property in a massive fire. The students of your school rendered their services and material help to the victims, Write a report in 100-125 words for your school magazine.
OR
Recently your school held a seminar on ‘Conservation of Water’ as a part of World Day celebrations. As the school Pupil Leader of Maryland School, Gurgaon, write a report in 100-125 words for a local daily. Sign as Preetham/ Preeti.
Answer:
A Fire Accident
Report By: Sweety
(Secretary, Co-Curricular Activities)
12th May, 20XX
A massive fire broke out in a nearby slum area of our city. It was caused by a short circuit in one of the lose wires passing through the area. The fire was uncontrollable and the situation became extremely serious as it gulped many houses and vehicles. People suffered fatal burns and grievous
injuries. It was a pitiable and horrifying scene. It took an hour to bring the fire under control. NGO’s, government aid, local residents, volunteers all came forward to carry out the relief operations. Food packets, medical aid and shelter were provided to the affected victims. Students from our school also offered help in the form of money and material. They donated clothes and reached the area to render services to the victims.
They also helped the victims by taking them to the nearby hospitals and stayed with them till their relatives came.
OR
Answer:
Need to conserve water
Report By : Preeti (School Pupil Leader)
Maryland School, Gurgaon
5thjune, 20XX
As a part of World Day Celebration, Maryland School held a seminar on the topic ‘Conservation of Water’ on 31st May, 20XX. It was headed by the eminent environmentalist’ Mr. Jain, who laid emphasis on the wise use of water. The main aim of this seminar was to highlight the importance of water conservation. It was stated that, with alarming increase of the population, we are experiencing an acute water shortage. Our water table is shrinking and it’s time to pledge to save water. Certain measures like use of water as per our requirement, turn off the tap after use, don’t leave tap running while brushing or shaving, install small shower heads to reduce the excess flow of water, were suggested. Water used for cleaning
the vegetables and fruits can be reused to water the plants. In the concluding speech, the appeal was made to start conserving and harvesting rain water for the present as well as the future generations. The organizers deserve appreciation and congratulations for holding a seminar on such topic.

Question.5. You are Sruthi/ Shiraj staying at R.K. Puram, Secunderabad. Your locality being away from the main city, the poor bus service adversely affects the life of residents. Write a letter to the editor, ‘Deccan Chronicle’, highlighting the problems faced and also giving a few possible solutions. [10]
OR
You are Varsha/ Varan, Incharge of Excursion Club of B.V.P. School, Mathura Road, Delhi. Write a letter to the General Manager, Northern Railways, requesting reservation of a bogie for 80 students from New Delhi to Chennai and back by G.T. Express. .
Answer:
R.K. Puram
Secunderabad.
18th August, 20XX
The Editor
Deccan Chronicle
Dear Sir,
Subject: Problems faced due to poor bus service
I would like to draw your attention towards, the problems faced by the people of R.K. Puram, in the absence of proper bus service. As my locality is far away from the main city, we have no other transportation facility than bus service. At times, the residents of the locality have to cover a long distance on foot to reach the place which not only wastes their time but also proves to be a tiresome journey. Even the time duration between the commutations of two buses is 4 to 5 hours and in case of urgency, people have to hire a taxi at a very high fare. The taxi drivers also take the undue advantage of the emergency. It is becoming a serious problem for the locals of the area, therefore I request you to start a new bus service from R.K. Puram to the main city to avoid any further inconvenience to the residents and if that is not possible, kindly start a tempo or mini bus service.
I hope my request would be considered favourably.
Thanking You Yours Sincerely,
Sruthi (Resident, R.K. Puram)
OR
B.V.P. School
Mathura Road Delhi
12th January, 20XX The General Manager Northern Railways Dear Sir,
Subject: Request for reserving a bogie for 80 students This is to inform your that an excursion tour has been planned by our school for 80 students from New Delhi to Chennai. This is an educational cum entertainment tour and I would like to make their train journey a great fun and full of experience. As G.T. Express is a fast and comfortable train and our priority is to see students’ comfort. I request you to make reservation for 80 students plus 1 teacher from G. T. Express on 5th-Feburary, 20xx. New Delhi to Chennai and return by the same train on 20th February, 20XX in first class A.C. Coach.
Kindly forward me the schedued time and the tickets so that students can reach the station in time. The payment of the tickets will be done online immediately after the reservation is confirmed.
Thanking You Yours Sincerely,
Varsha (Incarge, Excursion Club)

Question.6. Presently the prices of essential commodities are skyrocketing causing much hardship to the common man. Write an article in 150-200 words expressing your views and suggesting measures to curb this problem.
OR
The number of women in the police force seems insufficient specially when we see the increasing involvement of women in terrorist activities. Write an article in 150-200 words for ‘The Hindustan Chronicle’ on the need of having more women in the police force.
Answer:
Rising Prices : A Menace for
the Common Man
Majority of the population is the victim of this acute problem known as price hike. Most of the people live below poverty line and they are unable to fulfil even their basic requirements. The prices of essential commodities have touched the sky and therefore are beyond common man’s reach. The prices of vegetables, oil, sugar, wheat etc. have risen so much that the purchase of these has become difficult for common man. Money has lost its value. The rich are becoming richer whereas the poor are getting poorer. It has now become a fashion for the shopkeepers to increase, the rate of commodities within „ a day. Once the rates are increased, they cannot be expected to come down. This all, is due to black marketing and the dealers’ profit marking nonopoly. Once they raise the price, it cannot be reduced on the pretext of increase in petrol and diesel prices. Now it becomes the duty of the Government to take action against undue price hike and stop black marketing to control the rising prices. There should be regular checks on the import and export of commodities and the rates of food items must be under control so that the poor can also afford them.
OR
Need of Women in Police Force
By: Xyz
Having more women in the Police Force means proper law enforcement and better response to community. We see nowadays that women are becoming more and more active in terrorist activities as compared to the number of women in any of the forces. Their number is abysmally low in the Police Forces when they are the major portion of our work force. To combat this situation we need more women in Police Force. They not only play critical role in forming a police organization but also in addressing gender based violence. It affects women as well as children sometimes as they do not feel free to open up in front of male police officers. Therefore the women force is required to understand the grievances of the victims properly specifically women and teaching the security challenges faced by women. It is a local custom to recruit men in such forces but we cannot deny the fact that the number of women is increasing in criminal activities. Hence, recruitment of women ifi Police Forces should take place. Despite the hidden barriers, women should be given a chance to inter the force through proper relevant trainings.

SECTION-C
(Text Book)

Question.7. Read the extract given below and answer the question the follow:
It would be an exotic moment without rush, with engines, we would all be together in a sudden strangeness.
OR
Aunt Jennifer’s tigers prance across a screen,
Bright topaz denizens of a world of green.
They do no fear the men beneath the tiger;
They pace in sleek chivalric certainty.
(a) What does ‘It’ refer to?
Answer : ‘It’ refers to keeping quiet for a moment.
(b) Who is the poet speaking to?
Answer: The poet is speaking to the common man.
(c) What would be the moment like?
Answer : The moments would be without any rush or noise when all would be clam and serene will be the most desirable moment for the human beings.
OR
(a) How are Aunt Jennifer’s tigers described?
Answer : They are chivalric, brave and fearless. They are the lord of the jungle and have a respectable place in the animal kingdom.
(b) Why are they described as denizens of world of green?
Answer : The tigers are free to roam anywhere in the green forests and they are known for their strength and fearlessness.
(c) Why are they not afraid of men?
Answer : They are not afraid of men because they are not afraid of the men below the tree. They are strong, brave and fearless and known for their pride.

Question.8. Answer any three of the following in 30-40 words :
(a) Why are the young trees described as ‘sprinting’?
Answer : The young trees are symbol of young age, life and energy, so they seem to be sprinting. They are compared to the poet’s mother who is old and weak. She is in the last stage of her life whereas the trees are full of life.
(b) How does the poet describe the classroom walls?
Answer: The poet describes the walls of the classroom as dull and damp that smelled of sour cream. They are discoloured and faded and thus need paint. These dull walls have a Similarity with the faces of the children who are also pale.
(c) What image does Keats use to describe the beautiful bounty of the earth?
Answer : The poet emphasizes, the intransient nature of beautiful things that leave a permanent impression on our mind. Beauty is showered on us from heavens by God. This endless fountain of eternal beauty is God’s biggest gift to the mankind.

Question.9. Answer the following in 3040 words each :
(a) What had narrator counted on to enter the school
unnoticed?
Answer : The sounds that were heard daily like the banging of the desks, repetition of the lessons and rapping of the ruler, were so loud that the narrator thought he could enter the school without anybody’s notice.
(b) When Douglas realized that he was sinking, how did ^ he plan to save himself?
Answer : Douglas had death like experience while sinking but he gathered his courage and thought of conquering, the fear. As soon as he touched the bottom of the pool, he pushed himself up and came gradually to the surface.
(c) Why did the servants think Gandhiji to be another peasant ?
Answer : Gandhiji used to wear dhoti which was generally worn by the peasants, moreover he was a simple man, so servants mistook him as another peasant.
(d) *
(e)“Damn that Geoff, this was a Geoff thing not a Jansie thing” .Why did Sophie say so ?
Answer : Sophie wanted to reveal her truths, so she thought Geoff to be the right person rather than Jansie because Geoff could keep her secrets as he was an introvert and rarely talked to anyone else.

Question.10. Answer the following in 125-150 words:
The bangle makers of Ferozabad make beautiful bangles and make everyone happy but they live and die in squalor. Elaborate.
OR
The story ‘The Rattrap’ focuses on human loneliness and the need to bond with others. Explain.
Answer : The life of the bangle
makers of Ferozabad is miserable although they bring smiles to everyone’s face by making colorful and beautiful bangles. In their opinion, they are born to live in poverty for generation after generation and are destined to work in bangle making factories because they are born to the family of bangle makers. All through day and night, they have to work in bright furnaces to make beautiful bangles for the women but their own life is spent in utter darkness. They are forced to work in hot temperatures in dirty rooms without proper ventilation. They have to sit near burning lamps as a result of which they lose the brightness of their eyes. Their eyes become so accustomed to that light that they cannot see the natural daylight outside. Sometimes they become blind also. They are exploited by middlemen, policemen, sahukars, bureaucrats and politicians and it is impossible for them to come out of this vicious trap.
OR
The story ‘The Rattrap’ deals with the human psychology which reveals the need of human beings to share their joys and sorrows with others. Everyone wants other’s company to live in this vast world and its general human nature. Here in the story, the peddler is lonely and a homeless wanderer. He sells rattraps. On the other hand, Crofter lives alone so when the peddler came to his house at odd hours, he shows kindness and hospitality and allows him to stay for the night so that he can start a conversation with him. Other instance in the story is that, the ironmaster and his daughter are also lonely. They are very rich and have no problem, they live in a big house but they do not have company to have conversation with therefore, they insist peddler to spend Christmas Eve with them but when he doesn’t agree the ironmaster’s daughter requests him to stay which he could not deny. Thus we can say that the story focuses on human need to bond with others.

Question.11. Answer the following in 125-150 words :
When did the Tiger king stand in danger losing his kingdom? How was he able to avert the danger ? 
OR
How did the question paper and the correction slip help the prisoner and the Governor !
Answer : Tiger King was told by the astrologer that he would be killed by a tiger, so he started hunting and killing tigers. If anybody disobeyed him, he would be punished severely. A high ranking British official wanted to kill the tigers but the Maharaja did not give him the permission and told him that he could kill any animal except tigers. The officer wished to get a photograph of himself holding a gun beside tiger’s body. But the Maharaja refused that too. As he had not allowed him to fulfill his desire, he was in the danger of losing his kingdom.
He then discussed the issue with his deewan. So he offered a bribe of 50 diamond rings of the value of three lakhs to the wife of the British officer. She became happy and kept all of them. This way he was able to retain his kingdom.
OR
The real purpose of the question paper and the correction slip was to provide Evans the plan of escaping from the cell. The sheet was cleverly superimposed. The correction slip also made the prison officers believe that the injured person was actually McLeery and he wanted to help them in tracing the criminal, Evans. ”
On the other hand the question paper and the correction slip helped both the criminals as well as the Governor. The assistant secretary made a call to the Governor after the examination had started that he had forgotten to place a correction slip in the package. In fact, there was a clue in the slip for Evans that after escaping from the prison he should go to Hotel Golden Lions. At the same time this information helped the Governor to locate the Hotel where Evans was staying because Evans had written index number as 313 and the centre number as 271 which came out to be six figures 313/271 and with the help of ‘Ordance Survey Map for Oxfordshire’, the Governor reached the hotel. This was also a part of their plan and the Governor was made to trace Evans deliberately and eventually he was able to dupe the Governor and ran away.

Question.12. Answer the following in 30-40 words :
(a) *
(b) *
(c) Why did to think Roger Skunk was better off with the new smell ?
Answer : Roger Skunk had a very bad smell due to which other animals did not play with him. But a Wizard made him smell like roses and little animals started liking him and played with him. Thus Jo thought him to be better off with the new smell.
(d) Why did the landloard’s man ask Bama’s brother, on which street he lived ? What was the significance ?
Answer : To the landlord’s man, Bama’s brother was a new face so they asked him the name of the street on which he lived to know his caste as in those days the Dalit community lived in separate streets.

SET I

Note : Except for the following questions, all the remaining questions have been asked in previous sets.

SECTION-B
(ADVANCED WRITING SKILLS)

Question.4. Your school has recently organized a medical camp for screening students and staff for swineflu on the occasion of Independence Day Celebrations. The parents were also invited far screening. You are Rohit/Rashmi, Cultural Secretary of Vivekananda Kendra School, Bangalore. Write a report of the event in 100-125 words for ‘The Times of India.’
OR
You watched a special Independence Day Programme, “India’s Talent” on television. In the programme disabled children presented their latest talents in a marvelous manner. You were greatly touched and admired by organizers for their well thought out programme. Write a report in 100-125 words for your school magazine. You are Anjali/Ankur, School Pupil leader, Asha Memorial Sr. Sec. School, Agra.
Answer:
Medical Camp for Screening Students
and Staff for Swine Flu
Report By : Rashmi (Cultural Secretary)
‘The Times of India’
15th August, 20XX
On the occasion of Independence Day Celebrations, Vivekananda Kendra School, Bangalore came forward with an innovative idea of organizing a Medical Camp, for screening students and staff for swine flu. The main objective of organizing this camp was to emphasize the need of creating awareness of the spread, prevention and treatment of swine flu.: It was told by the senior Medical Officer that although the vaccines for Swine flu are available in medical stores but if not cured in time, it may prove to be fatal. It was also told that it spreads very fast and had started initially in pigs. The parents were also invited to join this event so that they would also be made aware of the causes and cures of this disease. A team of medical experts told that it spreads in the same way as seasonal flu, so we must follow the rules of health and hygiene, then only, the outbreak of this fatal disease can be controlled and prevented.
OR
India’s Talent
A report by : Anjali (School Pupil Leader)
Indian Television took a noble initiative of observing the independence Day by broadcasting a special programme
‘India’s Talent’ presented by people with disabilities. It was a social change campaign to encourage and support a fundamental shift in attitudes towards differently abled people. The performance done by them was mesmerizing and left an indelible impact’on viewer’s mind. They showcased their talent that would help them get respectable place in a society, where they are valued and included. The courage and confidence displayed by them was really fantastic. The objective of the program was to make the students aware of the fact that disability should not be treated as something to be ashamed off. The main credit goes to the organizers of the programme, who put their foot forward to give these special children a platform.

Question.6. In many parts of our country girls are discouraged from going to school. Consequendy a sizeable section of the population is deprived of education. Schemes like Sarva Shiksha Abhiyan, CBSE scholarship to single girl child and the government’s policy of giving free education to girls have come as a boon to our society. Write an article in 150¬200 words on education of the girl child in the country.
OR
Advertisements have become a big business. They are promoted by celebrities drawn from various fields like films, sports, etc., leaving their influence on people specially the young. Write an article in 150-200 words on ‘The impact of advertisements on the younger generation’.
Answer:
Education of the Girl Child
Since Independence time has changed a lot. The status of the women has seen a revolutionary change. They are competing – men in every walk of life. Even though its a male dominant society. There are places where girls are deprived of necessary education, and are not sent to schools. This shows the height of selfishness of men who think that acquiring good -» education is their right. It is generally said that girls have to learn their daily household chores which would be affected in case they go to school or colleges. It is also assumed that if they acquire education, they may surpass men. In the rural areas or villages, the situation is still worse. There is a gender biasness and girls are regarded as inferior to boys and are far more backward, illiterate, weak- and vulnerable than boys. They have to remain confined and satisfied with routine, monotonous domestic roles. In cities, towns and urban localities also, the percentage of educated girls is far less than boys. But the schemes started by Government like ‘ Sarva Shiksha Abhiyan’ which provides 8 years of quality education to all children between 6 to 14 years is a CBSE scholarship
to single girl child and the Government policy of giving free education to girls, are highly appreciable and in future they can prove to be productive also for the welfare of the country as well as their own family.
OR
‘The Impact of Advertisements on
the Younger Generation’
This is the age of advertisements and it is impossible to escape from them. Hoardings staring at us, magazine covers and neon signs on the shops catch our attention automatically. All these are one of the developments that have taken place with technological advances. T.V. Channels, films and magazines are full of advertisements as they are the lifeline of media. They directly reach out to the public through these commercial advertisements. But nobody has ever thought the impact they are leaving on our younger generation. They are not only colorful and attractive but are promoted by the film actors, sports players, T.V. artists etc. which lures the youth to a greater extent. They blindly follow them without giving a second thought whether they would be useful to them or not. The creativity packed in few seconds attracts youth so much that many of them can recall the advertisements better than their study material. The number of commercial advertisements is much more than the number of news channels, information media or entertainment channels. The innocent youth tries to adopt everything that is shown in ads without giving an ear to the adults. We can say that our youth is spell bpund by the magic of the ad world.

SECTION – C
(TEXT BOOKS)

Question.8. Answer any three of the following in 30-40words each :
(a) The poet says, “and yet for these children, these windows, not this map, their world”. Which world do these children belong to ? Which world is inaccessible to them?
Answer : The world of children living in slum is confined to their dirty and damp classroom only. The narrow street is their world. So the map and the world shown to them has no meaning for them. The world of freedom, education, equal rights and bright future is inaccessible to them.

Question.9. Answer the following in 30-40 words each :
(a) “This is your last French lesson”. How did Franz react to this declaration of M. Hamel?
Answer : Franz was shocked to hear that there would be no French teaching from now onwards. He felt extremely sorry for neglecting his lessons and regretted for not getting chance to learn grammar rules and participles and his French teacher M. Hamel would also be seen no more. 1
(b) What did Douglas experience as he went down to the bottom of the pool for the first time ?
Answer : As Douglas went down to the bottom of the pool for the first time, he felt afraid and paralyzed with fear. He shouted but no one came for his rescue and a sort of terror captured his soul.

Question.12.Answer the following in 30-40 words each:
(a) Why was Roger Skunk’s mommy angry with him ? What did she finally tell him?
Answer: Roger Skunks mommy was angry with him because he smelled like roses and little animals started playing with him but mommy wanted him to smell like before. Finally, she took him back to the wizard and he made Roger smell as awful as ever.

SET III

Note : Except for the following questions, all the remaining questions have been asked in previous sets.

SECTION B
(ADVANCED WRITING SKILLS)

Question.4. Parks are not only useful for children, adults and the old, but are environment friendly too. Write a factual description in 100-125 words of the neighborhood park you often visit.
OR
The Green Club of your school celebrated ‘Van Mahotsava and Environment Protection Week’ involving students of your school. Eminent environmentalists, journalists and celebrities were also invited. Write a report in 100-125 words on the activities performed for a focal daily. You are Amrit/Amrita, Secretary of the Club.
Answer:
My Neighbourhood Park
The nearest park to my locality is hardly at 100 meters of distance. I often visit this park with my father for morning and evening walks. It is full of beautiful flowers and plants. On the one side there are swings for children, painted in rainbow colours. The other corner has benches for the people to sit. The inner circumference of the park is surrounded by walk-way. People can be seen jogging and doing exercises in morning hours. Throughout the day it remains occupied . with people. Children always come in the evening as they are busy at school during day hours. One can hear chirping of birds from the branches of Neem and Gulmohur trees that surrounds the area. Old folks usually spend their evenings on the benches of the park, gossiping with each other. The whole environment inside the park is lively with the laughter of children and gathering of people in large numbers. It is not only a place for relaxation and entertainment but the green trees also provide fresh air and oxygen which is soothing to our eyes also.
OR
‘Van Mahotsava And Environment
Protection Week Celebration
Reported By : Amrita (Secretary, Green club)
A Local Daily
30th August, 20XX
The Green Club of our school celebrated a week long ‘Van Mahotsava and Environment Protection’ from 16th to 23rd August, 20XX in the school premises. The theme of the year was Reuse… Reduce…Recycle. The programme was inaugurated by eminent environmentalist, Mrs. Meneka Gandhi. Children up to the age of 12 years were seen in green dress. They made models of solar cooker, save water, utilizing garbage as fuel etc. They also gave the message through the plays ‘Death due to polythene’ and ‘Where are the jungles’? Famous Journalists and Celebrities from film industry and sports were also invited to view the programme. Cards and handmade paper bags made by the Eco Club members were distributed to everybody. The highlight of the program was the botde gardening by classes X and XII. The activities during the whole week included sapling plantation, awareness plays to save forests and wildlife, techniques to reuse discarded materials like cans, pen refills, plastic bottles etc.
On the final day, noted Journalist, Mr. Rajat Sharma appreciated the efforts of staff, students and Principal for spreading awareness through such programmes.

Question.6. Indian Culture has traditionally sustained a joint family system in which the old have always had a very respectable place. There has been an erosion of these values in the present times. Write an article in 150-200 words on the need for the young to take care of their old instead of sending them to the old age homes.
OR
The present day youth are greatly stressed due to cut throat competition and consumerist culture. Write an article in 150-200 words on the causes of the stress on the modem generation suggesting suitable solutions.
Answer:
Benefits of Joint Family „ over Nuclear Family
The concept of joint family where grandparents live together with their son, daughter-in-law and grandchildren, is very old in India. But the concept of nuclear families is catching up fast which consist of husband, wife and their children. We cannot deny the fact that there are many advantages of
joint families. The elder people have their own value in a family. We get complete support of them when needed. We don’t have to ask for help from others and children will not be left alone at home when parents are job. Burglary cases also decrease. One can share his joys, sorrows and opinions with them. They are considered to be the pillar of the family. But today’s culture is so spoiled that the elders of the family are considered as unwanted burden. Their lives in many houses have become miserable because of materialistic thought procedure. In some families, they are even sent to old age homes and do not get the love, security, sense of belongingness which they deserve. They need our affection, love and attention and it is our responsibility to take proper care of the elders who once had taught us to walk in this world. One must think again that they are more experienced and can still guide us. So, keeping an open mind, accept your faults and move ahead with them.
OR
Answer:
Stress and Problems Faced
By: Teenagers
The modern generation is facing unbearable pressure and problems due to cut throat competition in the field of education, employment and relationships. Educational problems are related to scoring high marks among students so as to get admission in a good college or university or getting a high degree in University. The pressure is so much that they don’t have time for recreation. They do not get time to relax or enjoy with their peers. As a result, this stress comes out in the form of aggression and violence. They display this sort of behavior at home, in school or among friends. Due to certain reasons and government policies, youth is not getting desired employment which results in depression and unbearable pressure. Another problem faced by the youth is the ups and downs in their relations. Either it’s their relationship with parents, siblings or friends; they are quite emotionally attached at this age.
All these problems can be cured by doing yoga and practicing meditation. They can also be relaxed, by involving in any of , their hobbies like painting, reading books, listening music. They should face these problems bravely instead of becoming its victim.

SECTION-C
(Text Book)

8. Answer any three of the following in 30-40 words each :
(b) “So blot their maps with slums as big as doom”, says Stephen Spender. What does the poet want to convey ?
Answer: The poet wants to convey that the children living in slums have miserable lives. The map in their classroom has no meaning for them, and the slums are like blot on the modern civilized world.

9. Answer the following in 30-40 words each :
(a) “What a thunder clap these words were to me!” Which were the words that shocked and surprised little Franz ?
Answer : The words that shocked and surprised little Franz were spoken by M. Hamel, he told them that it was their last French lesson and a new teacher would arrive the next day. There would be no German teaching from henceforth.
(b) What sort of terror seized Douglas as he went down the water with a yellow glow ? How could he feel that he was still alive ?
Answer : Extreme terror and fear seized Douglas when he went down the water with a yellow glow. He tried to scream but his voice remained choked in his throat. His heart beat and pounding in his head reminded him that he was alive.

12. Answer the following in 30-40 words each :
(c) How did Joe want the wizard to behave when Mommy Skunk approached him ?
Answer : Joe wanted wizard to take his magic wand and punish Rogers mommy by hitting over her head and cut her arms as she made little Roger smell foul again.

CBSE Previous Year Solved Papers Class 12 Computer Science Delhi 2012

CBSE Previous Year Solved  Papers  Class 12 Computer Science Delhi 2012

Time allowed : 3 hours                                                                                           Maximum Marks: 70

General Instructions :

  1.  There are a total of 26 questions and five sections in the question paper, All questions are compulsory.
  2. Section A contains question number 1 to 5, Very Short Answer type questions of one mark each.
  3.  Section B contains question number 6 to 10, Short Answer type I questions of two marks each.
  4.  Section C contains question number 11 to 22, Short Answer type II questions of three marks each.
  5.  Section D contains question number 23, Value Based Question of four marks.
  6. Section E contains question number 24 to 26, Long Answer type questions of five marks each.
  7. There is no overall choice in the question paper, however, an internal choice is provided in one question of two marks, one question of three marks and all three questions of five marks. An examined is to attempt any one of the questions out of two given in the question paper with the same question number.

Question.1. (a) Differentiate between the formal and actual parameters. Also, give a suitable C++ code to illustrate both.
Answer:
cbse-previous-year-solved-papers-class-12-computer-science-delhi-2012-1
(b) Which C++ header file(s) are essentially required to be
included to run/execute following C++ source code (Note : Do not include any header file, which is/are not required) :
cbse-previous-year-solved-papers-class-12-computer-science-delhi-2012-2
Answer : iostream.h and string.h
(c) Rewrite the following program after removing the syntactical error(s) (if any). Underline each correction.
cbse-previous-year-solved-papers-class-12-computer-science-delhi-2012-3
cbse-previous-year-solved-papers-class-12-computer-science-delhi-2012-4
Answer :
cbse-previous-year-solved-papers-class-12-computer-science-delhi-2012-5
cbse-previous-year-solved-papers-class-12-computer-science-delhi-2012-6
(d) Find the output of the following program :
cbse-previous-year-solved-papers-class-12-computer-science-delhi-2012-7
cbse-previous-year-solved-papers-class-12-computer-science-delhi-2012-9
(e) Find the output of the following program :
cbse-previous-year-solved-papers-class-12-computer-science-delhi-2012-10
Answer :
cbse-previous-year-solved-papers-class-12-computer-science-delhi-2012-11
(f) Observe the following program and find out, which output (s) out of (i) to (iv) will not be expected from the program? What will be the minimum and the maximum value assigned to the variable Chance ?
cbse-previous-year-solved-papers-class-12-computer-science-delhi-2012-12
(i) 9#6# (ii) 19#17# (iii) J9#16# (iv) 20#16
Answer:
(iii)19# 16#
Minimum Value : 16
Maximum Value : 20

Question.2.(a) What is the difference between the members in private visibility mode and the members in public visibility mode inside a class? Also, give a suitable C++ code to illustrate both.
Answer:
cbse-previous-year-solved-papers-class-12-computer-science-delhi-2012-13
cbse-previous-year-solved-papers-class-12-computer-science-delhi-2012-14
(b) Answer the question (i) and (ii) after going through the following class:
cbse-previous-year-solved-papers-class-12-computer-science-delhi-2012-15
cbse-previous-year-solved-papers-class-12-computer-science-delhi-2012-16

  1. In Object Oriented Programming, what are Function 1 and Function 3 combined together referred as?
  2.  In Object Oriented Programming, which concept is illustrated by Function 4 ? When is this function called/ invoked ?

Answer:

  1.  Function 1 and Function 3 combined together referred as Constructor Overloading.
  2. Function 4 is called Destructor. Whenever an object goes out of scope Function 4 is called.

(c) Define a class SUPPLY in C++ with following description: 4 Private Members:

  •  Code of type int
  •  FoodName of type string
  •  Sticker of type string
  •  FoodType of type string
  •  A member function GetType( ) to assign the following values for FoodType as per the given Sticker:
    cbse-previous-year-solved-papers-class-12-computer-science-delhi-2012-17

Public Members:

  • A function FoodIn() to allow user to enter values for
    Code, FoodName, Sticker and call function GetType() to assign respective FoodType.
  • A function FoodOut]) to allow user to view the concept of all the data members.

Answer:
cbse-previous-year-solved-papers-class-12-computer-science-delhi-2012-18
cbse-previous-year-solved-papers-class-12-computer-science-delhi-2012-19
(d)Answer the questions (i) to (iv) based on the following:
cbse-previous-year-solved-papers-class-12-computer-science-delhi-2012-20
cbse-previous-year-solved-papers-class-12-computer-science-delhi-2012-21
(i) Name the type of inheritance illustrated in the above C++ code.
(ii)Write the names of data members, which are accessible from member functions of class SHOWROOM.
(iii) Write the names of all the member functions, which are accessible from objects belonging to class WORKAREA.
(iv)Write the names of all the members, which are accessible from objects of class SHOWROOM.
Answer.
cbse-previous-year-solved-papers-class-12-computer-science-delhi-2012-22

Question.3.(a) Write a ftmction SWAP2CHANGE (int p[ ],int N) in C++ i to modify the content of the array in such a way that the elements, which are multiples of 10 swap with the value present in the very next position in the array.
For example: Let us take an array namely P.
cbse-previous-year-solved-papers-class-12-computer-science-delhi-2012-23
Answer:
cbse-previous-year-solved-papers-class-12-computer-science-delhi-2012-24
(b) An array S[10][30] is stored in the memory along the column with each of the element occupying 2 bytes, find out the memory location of S[5][10], if an element S[2] [15] is stored at location 8200.
Answer:
cbse-previous-year-solved-papers-class-12-computer-science-delhi-2012-25
(c) Write a function in C++ to perform insert operation in a Dynamic Queue containing DVD’s information (represented with the help of an array of structure DVD).
cbse-previous-year-solved-papers-class-12-computer-science-delhi-2012-26
cbse-previous-year-solved-papers-class-12-computer-science-delhi-2012-27
(d) Write a function SKIPEACH (int H[ ][3],int C, int R) in C++ to display all alternate elements from two-dimensional array H (starting from H[0] [0]).
For example:
cbse-previous-year-solved-papers-class-12-computer-science-delhi-2012-28
Answer:
cbse-previous-year-solved-papers-class-12-computer-science-delhi-2012-29
(e) Evaluate the following POSTFIX notation. Show status of Stack after every step of evaluation (i.e. after each operator) :
False, NOT, True, AND, True, False, OR, AND
Answer:
cbse-previous-year-solved-papers-class-12-computer-science-delhi-2012-30

Question.4(a) Observe the program segment given below carefully and the question that fallow:
cbse-previous-year-solved-papers-class-12-computer-science-delhi-2012-31
cbse-previous-year-solved-papers-class-12-computer-science-delhi-2012-32
(i) Write statement 1 to position the file pointer to the appropriate place, so that the data updation is done for the required Article.
(ii) Write statement 2 to perform the write operation so that the updation is done in the binary file. Answer:
(i) File.Seekp(-l * sizeof(I), ios :: (ur));
(ii) File.Writte((char*) &I, sizeof(I));
(b) Write a function in C++ to read the content of a text file “PLACES.TXT” and display all those lines on screen, which are either starting with ‘P’ or starting with ‘S’.
Answer:
cbse-previous-year-solved-papers-class-12-computer-science-delhi-2012-33
cbse-previous-year-solved-papers-class-12-computer-science-delhi-2012-34
(c) Write a function in C++ to search for the details (Number and Calls) of those Mobile phones, which have more than 1000 calls from a binary file “mobile.dat”. Assuming that this binary file contains records/objects of class Mobile, which is defined below:
cbse-previous-year-solved-papers-class-12-computer-science-delhi-2012-35
Answer: void Display
cbse-previous-year-solved-papers-class-12-computer-science-delhi-2012-36

Question.5(a) Give a suitable example of a table with sample data and illustrate Primary and Candidate Keys in it.
Answer:
Primary Key— It’s a column or set of columns that helps in to identify records uniquely.
HERE, Adm.no., Roll no. both can identify records uniquely. So we can make Asmno as primary key.
Candidate Key—It’s a column or set of columns that can act as a primary key but is not selected as a primary key.
Since, we have created Adm.no. as Primary key, so Roll no. is the candidate key.
Consider the following tables CARHUB and CUSTOMER and answer (b) and (c) parts of question :
cbse-previous-year-solved-papers-class-12-computer-science-delhi-2012-37
(b) Write the SQL command for the following statements :

  1.  To display the names of all the white colored vehicles.
  2. To display the name of vehicle, make and capacity of vehicles in ascending order of their sitting capacity.
  3. To display the highest charges at which a vehicle can be hired from CABHUB.
  4. To display the customer name and the corresponding name of the vehicle hired by them.

Answer:

  1.  select VehicleName from CABHUB where Color = ‘WHITE’;
  2. select VehicleName, Make, Capacity from CABHUB order by Capacity;
  3. select MAX(Charges) from CABHUB;
  4. select CName, VehicleName from CABHUB CA,
    CUSTOMER CU where CA.VCode= CU.VCode;

(c) Give the output of the following SQL queries :
(I) SELECT COUNT (DISTINCT Make) FROM CABHUB;
(II)SELECT MAX(CHARGES), MIN (CHARGES) FROM CABHUB;
(III)SELECT COUNTCO, Make, FROM CABHUB;
(IV)SELECT VEHICLE FROM CABHUB WHERE CAPACITY=4;
Answer:
cbse-previous-year-solved-papers-class-12-computer-science-delhi-2012-38

Question.6.(a) Verify the following using truth table
cbse-previous-year-solved-papers-class-12-computer-science-delhi-2012-39
Answer.
cbse-previous-year-solved-papers-class-12-computer-science-delhi-2012-40
(b) Write the equivalent Boolean Expression for the following Logic Circuit:
cbse-previous-year-solved-papers-class-12-computer-science-delhi-2012-41
Answer:
cbse-previous-year-solved-papers-class-12-computer-science-delhi-2012-42
(c) Write POS form of Boolean function G, which is represented in a truth table as follows:
cbse-previous-year-solved-papers-class-12-computer-science-delhi-2012-43
(d) Reduce the following Boolean Expression using K-Map: 3 F(P, Q, R, S) = £ (1, 2, 3, 4, 5, 6, 7, 8, 10)
Answer:
cbse-previous-year-solved-papers-class-12-computer-science-delhi-2012-44

Question.7(a) What out of the following, will you use to have an audio-visual chat with an expert sitting in a far-away place to fix¬up a technical issue?

  1.  Email
  2. VoIP
  3. FTP

Answer: VoIP
(b) Name one client side scripting language and one server side scripting language.
Answer : Server side script— ASP
Client side script— Javascript
(c) Which out of the following comes under Cyber Crime?

  1.  Stealing a Mouse from someone’s computer.
  2. Operating someone’s internet banking account, without his knowledge.
  3. Entering in someone’s computer remotely and copying data, without seeking his permission.

Answer :

  1.  Operating someone’s internet banking account, without his knowledge.
  2.  Entering in someone’s computer remotely and copying data, without seeking his permission.

(d) Write one advantage of Star Topology of network. Also, illustrate how 5 computers can be connected with each other using Star topology of network.
Answer : One advantage of Star Topology of network is that no disruption to the network while connecting or removing devices. 5 computers can be connected with each other using server.
cbse-previous-year-solved-papers-class-12-computer-science-delhi-2012-45
(e) Granuda Consultants are setting up a secured network for their office campus at Faridabad for their day-to-day office and web-based activities. They are planning to have connectivity between 3 buildings and the head office situated in Kolkata. Answer the questions (el) to (e4) after going through the building positions in the campus and other details, which are given below :
cbse-previous-year-solved-papers-class-12-computer-science-delhi-2012-46
cbse-previous-year-solved-papers-class-12-computer-science-delhi-2012-47
(1) Suggest the most suitable place (i.e. building) to house the server of this organization. Also give a reason to justify your suggested location.
(2) Suggest a cable layout of connection between the buildings
inside the campus.
Suggest the placement of the following device with justification:
(i) Switch
(ii)Repeater
(3) The organization is planning to provide a high speed link with its head office situated in KOLKATTA using a wired connection. Which of the following cables will be most suitable for this job?
(i) Optical Fibre
(ii)Co-axial Cable
(iii)Ethernet
Cable
Answer:
(1) “JAMUNA” building is most suitable place to install the server because maximum number of computers are there.
(2)
cbse-previous-year-solved-papers-class-12-computer-science-delhi-2012-48
(3) (ii) Co-axial Cable
(f) Give one suitable example of each URL and Domain Name.
Answer:
URL—http://www. face b ook. co m / login.
Domain Name—www. facebook.eom
(g) Name two Open Source software along with its application.
Answer:
(i) XAMPP — a package of web applications including Apache and MySQL
(ii)Mozilla Thunderbird — cross-platform email, news, and chat client developed by the Mozilla Foundation.

CBSE Previous Year Solved Papers Class 12 Physics Outside Delhi 2012

CBSE Previous Year Solved  Papers  Class 12 Physics Outside Delhi 2012

Time allowed : 3 hours                                                                                           Maximum Marks: 70

General Instructions:

  1.  All questions are compulsory. There are 26 questions in all.
  2.  This question paper has five sections : Section A, Section B, Section C, Section D and Section E.
  3. Section A contains five questions of one mark each, Section B contains five questions of two marks each, Section C contains twelve questions of three marks each, Section D contains one value based question of four marks and Section E contains three questions of five marks each.
  4.  There is no overall choice. However, an internal choice has been provided in one question of two marks, one question of three marks and all the three questions of five marks weightage. You have to attempt only one of the choices in such questions.
  5. You may use the following values of physical constants wherever necessary:

SET I

Question.1.Two wires of equal length, one of copper and the other of manganin have the same resistance. Which wire is thicker?
Answer : Since the resistivity of alloy is greater than the resistivity of its constituents. We have resistivity of manganin greater than resistivity of copper metal.
cbse-previous-year-solved-papers-class-12-physics-outside-delhi-2012-1
cbse-previous-year-solved-papers-class-12-physics-outside-delhi-2012-2

Question.2. What are the directions of electric and magnetic field vectors relative to each other and relative to the direction of propagation of electromagnetic waves?
Answer: Since electromagnetic waves are transverse in nature. We have electric and magnetic fields associated with an electromagnetic wave perpendicular to each other and perpendicular to the direction of propagation of electromagnetic waves.
cbse-previous-year-solved-papers-class-12-physics-outside-delhi-2012-3
Let the direction of electric field and magnetic field is along Y and Z-axis then the direction of propagation of EM waves will be along positive X-axis.

Question.3.How does the angular separation between fringes in single¬slit diffraction experiment change when the distance of separation between the slit and screen is doubled?
Answer : We know angular separation is given as
cbse-previous-year-solved-papers-class-12-physics-outside-delhi-2012-4
Since 9 is independent of D i.e., the distance of separation between the screen and the slit, so when D is doubled, angular separation would remain same.

Question.4.A bar magnet is moved in the direction indicated by the arrow between two coils PQand CD. Predict the directions of induced current in each coil
cbse-previous-year-solved-papers-class-12-physics-outside-delhi-2012-5
Answer : According to Lenz’s law the polarity of the induced emf is such that it opposes the change in magnetic flux responsible for its production.
Since North pole of bar magnet is receding away from the coil so the right end of the coil will develop South pole i.e., induced current as seen from the left end will be anticlockwise.
cbse-previous-year-solved-papers-class-12-physics-outside-delhi-2012-6

Question.5. For the same value of angle of incidence, the angles of refraction in three media A, B and C are 15°, 25° and 35° respectively. In which medium would the velocity of light be minimum?
Answer : As light travels from a rarer to denser medium it ‘ bends towards the normal as its speed decreases. So, if the
bending is more, the speed of the light would be less in that medium, compared to other media. As the angle of refraction is measured with respect to the normal, the ray making the
least angle of refraction would bend more and the speed oflight would be minimum in that case. So, the correct option is medium A where refracting angle is 15°.

Question.6. A proton and an electron have same kinetic energy. Which one has greater de-Broglie wavelength and why?
Answer : Since de-Broglie wavelength X in terms of kinetic energy is given as
cbse-previous-year-solved-papers-class-12-physics-outside-delhi-2012-7

Question.7. Mention the two characteristic properties of the material suitable for making core of a transformer.
Answer : Two characteristic properties of material:
(i) Low hysteresis loss.
(ii) Low coercivity.

Question.8.A charge ‘q’ is placed at the centre of a cube of side /. What is the electric flux passing through each face of the cube?
Answer : By using Gauss’s Law,
cbse-previous-year-solved-papers-class-12-physics-outside-delhi-2012-8

Question.9. A test charge ‘q’ is moved without acceleration from A to C along the path from A to B and then from B to C in electric field E as shown in the figure, (i) Calculate the potential difference between A and C. (ii) At which point (of the two) is the electric potential more and why?
cbse-previous-year-solved-papers-class-12-physics-outside-delhi-2012-9
Answer : (i) Since work done is independent of the path therefore, we may directly move from A to C.
Potential difference between A and C is given by,
cbse-previous-year-solved-papers-class-12-physics-outside-delhi-2012-10

Question.10. An electric dipole is held in a uniform electric field.
(i) Show that the net force acting on it is zero.
(ii) The dipole is aligned parallel to the field. Find the work
done in rotating it through the angle of 180°.
Answer: (i) Consider an electric dipole consisting of two equal and opposite point charges, -q at A and +q at B, separated by a small distance 2a.
cbse-previous-year-solved-papers-class-12-physics-outside-delhi-2012-11
cbse-previous-year-solved-papers-class-12-physics-outside-delhi-2012-12

Question.11. State the underlying principle of a transformer. How is the large scale transmission of electric energy over long distances done with the use of transformers?
Answer : Transformer Principle : It is a device which converts high voltage AC into low voltage AC and vice-versa. It is based upon the principle of mutual induction. When alternating current is passed through a coil, an induced emf is set up in the neighbouring coil.
Transformers are used for transmission of electrical energy over long distances.
It step up the output voltage of power plant using step up transformer which reduce the current through cables and hence reduce resistive power loss. Then a step down transformer is used at consumer end to step down the voltage.

Question.12. A capacitor of capacitance ‘C’ is being charged by connecting it across a dc source along with an ammeter. Will the ammeter show a momentary deflection during the process of charging? If so, how would you explain this momentary deflection and the resulting continuity of current in the circuit? Write the expression for the current inside the capacitor.
Answer: Yes, the ammeter shows a momentary deflection during the process of charging because changing electric field produce displacement current between the plates of the capacitor.
The resulting continuity of current in the circuit is because inside the capacitor, displacement current exist and in the wire conduction current flow.
cbse-previous-year-solved-papers-class-12-physics-outside-delhi-2012-13

Question.13. An object AB is kept in front of a concave mirror as shown in the figure.
cbse-previous-year-solved-papers-class-12-physics-outside-delhi-2012-14
(i) Complete the ray diagram showing the image formation of the object.
(ii)How will the position and intensity of the image be affected if the lower half of the mirrors reflecting surface is
painted black?
cbse-previous-year-solved-papers-class-12-physics-outside-delhi-2012-15
Image formed will be inverted, between focus and center of curvature and small in size.
(ii) If the lower half of the mirror’s reflecting surface is painted black, the position of image will be same but its intensity gets reduced.

Question.14. Draw a labelled ray diagram of a reflecting telescope. Mention its two advantages over the refracting telescope.
Answer : Reflecting Telescope :
cbse-previous-year-solved-papers-class-12-physics-outside-delhi-2012-16
Advantages over refracting telescope :

  1.  It reduces the spherical aberration by using parabolic mirror and forms a clear focussed image.
  2. The resolving power of a large aperture mirror is high and hence minute details of distant stars can be obtained.

Question.15. Describe briefly with the help of a circuit diagram, how the flow of current carriers in a p-n-p transistor is regulated with emitter-base junction forward biased and base-collector junction reverse biased.
Answer : In p-n-p transistor, the emitter base junction is always forward biased with voltage VEE and the collector base junction is always reverse biased with voltage Vcc
cbse-previous-year-solved-papers-class-12-physics-outside-delhi-2012-17
The holes in the emitter are pushed into the base by the positive terminal of battery of voltage VEE- Since, base is thin and lighdy doped so only few holes combine with electrons in the base. Thus, base current IB is small. Since Vcc is quite large almost 99% of holes coming from emitter are collected by collector. For each hole reaching the collector, an electron is released from the negative terminal of the collector base battery to neutralize thf hole. For each hole consumed in collector, a bond breaks in emitter and electron is released that enters positive terminal of emitter-base battery. Thus, we can say the current is carried by holes inside the transistor and by electrons in external circuit.

Question.16. In the given block diagram of a receiver, identify the boxes labelled as X and Y and write their functions.
cbse-previous-year-solved-papers-class-12-physics-outside-delhi-2012-18
X → IF stage → Intermediate frequency stage to change the , carrier frequency to lower frequency.
Y → Amplifier → It amplifies the signal i.e., it increase the amplitude of the detected signal to compensate the attenuation of the signal. The detected signal may not be strong enough to use.

Question.17. A light bulb is rated 100 W for 220 V ac supply of 50 Hz. Calculate:
(i) The resistance of the bulb;
(ii) The rms current through the bulb.
OR
An alternative voltage given by V = 140 sin 314t is connected across a pure resistor of 50Ω. Find
(i) the frequency of the source.
(ii) the rms current through the resistor.
cbse-previous-year-solved-papers-class-12-physics-outside-delhi-2012-19

Question.18. A circular coil of‘N’ turns and radius ‘R’ carries a current ‘I’. It is unwound and rewound to make another coil of radius ‘R/2’, current T remaining the same. Calculate the ratio of the magnetic moments of the new coil and original coil.
Answer:The magnetic moment m of a current loop,
cbse-previous-year-solved-papers-class-12-physics-outside-delhi-2012-20

Question.19. Deduce the expression for the electrostatic energy stored in ( a capacitor of capacitance ‘C’ and having charge ‘Q’. How will the (i) energy stored and (ii) the electric field inside the capacitor be affected when it is completely filled with a dielectric material of dielectric constant ‘K’?
Answer: Energy stored in a charged capacitor: The energy of a charged capacitor is measured by the total work done in charging the capacitor to a given potential. We know that,
cbse-previous-year-solved-papers-class-12-physics-outside-delhi-2012-21
cbse-previous-year-solved-papers-class-12-physics-outside-delhi-2012-22
When dielectric material of dielectric constant ‘K’ is introduced inside the capacitor then
cbse-previous-year-solved-papers-class-12-physics-outside-delhi-2012-23
cbse-previous-year-solved-papers-class-12-physics-outside-delhi-2012-24

Question.20. Calculate the value of the resistance R in the circuit shown in the figure so that the current in the circuit is 0.2 A. What would be the potential difference between points B and E?.
cbse-previous-year-solved-papers-class-12-physics-outside-delhi-2012-25
cbse-previous-year-solved-papers-class-12-physics-outside-delhi-2012-26
cbse-previous-year-solved-papers-class-12-physics-outside-delhi-2012-27

Question.21. You are given three lenses L1, L2 and L3 each of focal length 20 cm. An object is kept at 40 cm in front of L1, as shown. The final real image is formed at the focus T of L3. Find the separations between L1, L2 and L3
cbse-previous-year-solved-papers-class-12-physics-outside-delhi-2012-28
cbse-previous-year-solved-papers-class-12-physics-outside-delhi-2012-29
But we have seen above that image by L1 is formed at 40 cm on the right of L1 which is at 20 cm left of L2 (focus of L2). So, x1= distance between L1 and L2 = (40 + 20) cm = 60 cm Again distance between L2 and L3 does not matter as the image by L2 is formed at infinity.
Hence, the distance between L2 and L3 can have any value.

Question.22. Define the terms (i) ‘cut-off voltage’ and (ii) ‘threshold frequency’ in relation to the phenomenon of photoelectric effect.
Using Einstein’s photoelectric equation show how the cut-off voltage and threshold frequency for a given photosensitive material can be determined with the help of a suitable plot/ graph.
Answer : When light of suitable frequency is incident on a metal surface, electrons are ejected from the metal. This phenomenon is called the photoelectric effect.
cbse-previous-year-solved-papers-class-12-physics-outside-delhi-2012-30
(i) The cathode is illuminated with light of some fixed frequency v and fixed intensity lj. A small photoelectric current is observed due to few electrons that reach anode just because they have sufficiently large velocity of emission. If we make the potential of the anode negative with respect to cathode then the electrons emitted by cathode are repelled. Some electrons even go back to the cathode so that the current decreases. At a certain value of this negative potential, the current is completely stopped. The least value of this anode potential which just stops the photocurrent is called cut off potential or stopping potential.
(ii) For a given material, there is a certain minimum frequency that if the incident radiation has a frequency below this threshold, no photoelectric emission will take place, howsoever intense the radiation may be falling.
According to Einstein’s photoelectric equation, maximum K.E. is given as
cbse-previous-year-solved-papers-class-12-physics-outside-delhi-2012-31
cbse-previous-year-solved-papers-class-12-physics-outside-delhi-2012-32
We can read the value of threshold frequency from graph. From equation (1), we can find the value of stopping potential (V0).

Question.23. A series LCR circuit is connected to an ac source. Using the phasor diagram, derive the expression for the impedance of the circuit. Plot a graph to show the variation of current with frequency of the source, explaining the nature of its variation.
Answer : Let an alternating emf E = E0  sin ωt is applied to a series combination of inductor L, capacitor C and resistance R. Since all three of them are connected in series the current through them is same. But the voltage across each element has a different phase relation with current.
cbse-previous-year-solved-papers-class-12-physics-outside-delhi-2012-33
cbse-previous-year-solved-papers-class-12-physics-outside-delhi-2012-34
cbse-previous-year-solved-papers-class-12-physics-outside-delhi-2012-35
cbse-previous-year-solved-papers-class-12-physics-outside-delhi-2012-36
cbse-previous-year-solved-papers-class-12-physics-outside-delhi-2012-37
With increase in frequency, current first increases and then decreases. At resonant frequency, the current amplitude is maximum.

Question.24. Mention three different modes of propagation used in communication system. Explain with the help of a diagram how long distance communication can be achieved by ionospheric reflection of radiowaves.
Answer: Three modes of propagation of electromagnetic waves,
(a) Ground waves,
(b) Sky waves
(c) Space waves.
Sky wave propagation is used for long distance communication by ionospheric reflection of radio waves.
cbse-previous-year-solved-papers-class-12-physics-outside-delhi-2012-38
In the ionosphere of the Earth’s atmosphere, there are a large number of charged particles (ions). The ionosphere is situated about 65 km — 400 km above the surface of the Earth. The ionization of molecules occurs due to the absorption of the ultraviolet rays and high energy radiation from the sun. The ionosphere acts as a reflecting layer for certain range of frequencies (3 MHz — 30 MHz)!’The transmitting antenna sends the EM signals of this frequency range towards the ionosphere. When the EM waves strikes the ionosphere, it is reflected back to the Earth. A receiving antenna at a remote location on the Earth receives these reflected signals.

Question.25. Draw a plot of potential energy of a pair of nucleons as a functions of their separations. Mark the regions where the nuclear force is

  1. attractive and
  2.  repulsive. Write any two characteristic features of nuclear forces.

Answer : Potential energy of a pair of nucleons as a function of their separation :
cbse-previous-year-solved-papers-class-12-physics-outside-delhi-2012-39
Here, Part AB represents repulsive force and part BCD , represents attractive force.
ro is the distance at which potential energy is minimum.
For a separation greater than ro, the force is attractive and for separations less than ro the force is strongly repulsive. Characteristic features of nuclear forces are :

  1. Nuclear forces are much stronger then Coulomb forces acting between charges or the gravitational forces between masses.
  2. The nuclear force between neutron-neutron, proton- neutron and proton-proton is approximately the same. The nuclear force does not depend on the electric charge.

Question.26. In a Geiger-Marsden experiment, calculate the distance of closest approach to the nucleus of Z = 80, when a-particle of 8 MeV energy impinges on it before it comes momentarily to rest and reverses its direction.
How will the distance of closest approach be affected when the kinetic energy of the a-particle is doubled?
OR
The ground state energy of hydrogen atom is -13.6 eV. If an electron makes a transition from an energy level —0.85 eV to -3.4 eV, calculate the wavelength of the spectral line emitted. To which series of hydrogen spectrum does the wavelength belong?
Answer : Let r0 be the centre to centre distance between the alpha-particle and nucleus.
cbse-previous-year-solved-papers-class-12-physics-outside-delhi-2012-40
cbse-previous-year-solved-papers-class-12-physics-outside-delhi-2012-41
cbse-previous-year-solved-papers-class-12-physics-outside-delhi-2012-42

Question.27. Define relaxation time of the free electrons drifting in a conductor. How is it related to the drift velocity of free electrons? Use this relation to deduce the expression for the electrical resistivity of the material.
Answer : Relaxation time (τ), is the time for which a free electron accelerates before it undergoes a collision with the positive ion in the conductor. Or, we can say it is the average time elapsed between two successive collisions. It is of the order 10-14 seconds. It decreases with increase of temperature and is given as
cbse-previous-year-solved-papers-class-12-physics-outside-delhi-2012-43
cbse-previous-year-solved-papers-class-12-physics-outside-delhi-2012-44
cbse-previous-year-solved-papers-class-12-physics-outside-delhi-2012-45

Question.28. (a) In Youngs double slit experiment, derive the condition for
(i) constructive interference and
(ii) destructive interference at a point on the screen.
(b) A beam of light consisting of two wavelengths, 800 nm and 600 nm is used to obtain the interference fringes in a Youngs double slit experiment on a screen placed 1.4 m away. If the two slits are separated by 0.28 mm, calculate the least distance from the central bright maximum where the bright fringes of the two wavelengths coincide. OR
(a) How does an unpolarized light incident on a polaroid get polarized? Describe briefly, with the help of a necessary diagram, the polarization of light by reflection from a transparent medium.
(b) Two polaroids ‘A’ and ‘B’ are kept in crossed position.
How should a third polaroid ‘C’ be placed between them so that the intensity of polarized light transmitted by polaroid B reduces to l/8th of the intensity of unpolarized light incident on A?
Answer : (a) Condition of constructive and destructive interference :
cbse-previous-year-solved-papers-class-12-physics-outside-delhi-2012-46
In the given figure, S is a monochromatic source of light. S1 and S2 are two narrow pin holes equidistant from S and they act as coherent sources. Cofasider a point P on the screen XY placed parallel to S1 and S2.
Let a1 be the amplitude of the waves from S1 and S2 that from S2. Let ø be the phase difference between the two waves reaching the point P. Let y1 and y2 be the displacements of the two wavews, arriving at P.
cbse-previous-year-solved-papers-class-12-physics-outside-delhi-2012-47
cbse-previous-year-solved-papers-class-12-physics-outside-delhi-2012-48
cbse-previous-year-solved-papers-class-12-physics-outside-delhi-2012-49
Clearly the maximum intensity is obtained in the region of superposition at those points where waves meet in the same phase or the phase difference between the waves is even multiple of π or path difference between them is the integral multiple of λ and maximum intensity is (a1 + a2)2 which is greater than the sum intensities of individual waves by an amount 2a1 + a2.
cbse-previous-year-solved-papers-class-12-physics-outside-delhi-2012-50
cbse-previous-year-solved-papers-class-12-physics-outside-delhi-2012-51
(a) Polaroid is made up of a special material which blocks one of the two planes of vibration of an electromagnetic wave. Because of its chemical composition it allows only those vibrations of the electromagnetic wave which are parallel to its crystallographic axis.
cbse-previous-year-solved-papers-class-12-physics-outside-delhi-2012-52
An ordinary beam of light on reflection from a transparent medium becomes partially polarised. The degree of polarization increases as the angle of incidence is increased. At a particular value of angle of incidence, the reflected beam becomes completely polarised. This angle of incidence is called the polarizing angle (ip).
cbse-previous-year-solved-papers-class-12-physics-outside-delhi-2012-53

Question.29. (a) Describe briefly, with the help of a diagram, the role of the two important processes involved in the formation of a. p-n junction.
(b) Name the device which is used as a voltage regulator. Draw the necessary circuit diagram and explain its working.
OR
(a) Explain briefly the principle on which a transistor- amplifier works as an oscillator. Draw the necessary circuit diagram and explain its working.
(b) Identify the equivalent gate for the following circuit
and write its truth table.
cbse-previous-year-solved-papers-class-12-physics-outside-delhi-2012-54
Answer: (a) Two important process involved in the formation of a p-n junction are :
Diffusion and Drift: In n-type semiconductor, electrons are the majority carriers and holes are minority carriers. In the same way, in p-type semiconductor holes are majority and electrons are minority carriers. During the formation of p-n junction, due to concentration gradient, the holes diffuse from p side to n side and electrons diffuse from n side to p side. This motion gives rise to diffusion current across the junction.
When an electron diffuses from n to p side, it leaves behind a positive charge. In such a manner a positively charged layer forms on «-side of the junction.
Similarly, when a hole diffuses from p to n side, it leaves behind a negative charge and a negatively charged layer forms on p side of the junction.
This space charge region is known as depletion region. An electric field directed from positive charge towards negative charge develops. Due to this field, electrons on p side of the junction move to w-side and holes on n side of the junction move to p side. This motion of charge carriers due to the electric field is called drift.
Drift current is opposite in direction to the diffusion current. Electron drift E Electron diffusion
cbse-previous-year-solved-papers-class-12-physics-outside-delhi-2012-55
Initially, diffusion current is large and‘drift current is small. Space-charge region on either side increases as the diffusion process continues. This increases the electric field and hence the drift current. This process continues until the diffusion current equals the drift current. Thus, a p-n junction is formed.
cbse-previous-year-solved-papers-class-12-physics-outside-delhi-2012-56
Voltage regulator converts an unregulated dc voltage into a constant regulated dc voltage using zener diode. The un regulated voltage is connected to the zener diode through a series resistance Rs such that the zener diode is reverse biased. If the input voltage increases, the current through Rs and zener diode also increases. This increases the voltage drop across Rs without any change in voltage drop across zener diode. This is because in the breakdown region, zener voltage remain constant even though the current through zener diode changes.
Similarly, if the input voltage decreases, the current through Rs and zener diode decrease. The voltage drop across Rs decrease without any change in the voltage across the zener diode. Thus any change in input voltage results the change in voltage drop across Rs without any change in voltage across the zener diode.
Thus, zener diode acts as a voltage regulator.
OR
(a) Transistor amplifier as an oscillator: In an oscillator, the output at a desired frequency is obtained without applying any external input voltage.
The common emitter n-p-n transistor as an oscillator is shown in the following figure.
A variable capacitor C of suitable range is connected in parallel to coil T2 to give the variation in frequency.
cbse-previous-year-solved-papers-class-12-physics-outside-delhi-2012-57
Oscillator action: As in an amplifier, the base-emitter junction is forward biased while the base collector junction is reverse biased. When the switch S is put on, charge of collector flows in the coil T2. The inductive coupling between coil T2 and T1 cause a current to flow in the emitter circuit i.e. feedback from input to output. As a result of positive feedback, the collector current reaches at maximum. When there is no further feedback from T2 to T1, the emitter current begins to fall and collector current decreases. Therefore, the transistor has reverted back to its original state. The whole process now repeats itself. The resonance frequency (f) of the oscillator is given by:
cbse-previous-year-solved-papers-class-12-physics-outside-delhi-2012-58
cbse-previous-year-solved-papers-class-12-physics-outside-delhi-2012-59

Question.30.
cbse-previous-year-solved-papers-class-12-physics-outside-delhi-2012-60
(a) Explain giving reasons, the basic*difference in converting a galvanometer into (i) a voltmeter and (ii) an ammeter.
(b) Two long straight parallel conductors carrying steady currents Ii and I2 are separated by a distance ‘tP. Explain briefly, with the help of a suitable diagram, how the magnetic field due to one conductor acts on the other. Hence deduce the expression for the force acting between the two conductors. Mention the nature of this force.
cbse-previous-year-solved-papers-class-12-physics-outside-delhi-2012-80
The direction of electric and magnetic forces are in opposite direction. Their magnitudes are in such a way that they cancel out each other to give net force zero and so the charge particle does not deflect.
(b) When an electric current flows in closed loop of wire, placed in a uniform magnetic field, the magnetic forces produce a torque which tends to rotate the loop so that area of the loop is perpendicular to the direction of the magnetic field.
cbse-previous-year-solved-papers-class-12-physics-outside-delhi-2012-62
cbse-previous-year-solved-papers-class-12-physics-outside-delhi-2012-63
cbse-previous-year-solved-papers-class-12-physics-outside-delhi-2012-64
cbse-previous-year-solved-papers-class-12-physics-outside-delhi-2012-65
(a) (i) In converting a galvanometer into a voltmeter, a very high suitable resistance is connected is series to its coil. So, r that galvanometer gives foil scale deflection.
(ii) In converting a galvanometer itjto an ammeter, a very small suitable resistance is connected in parallel to its coil. The remaining pair of the current i.e. (I – Ig) flows through the resistance.
(b) Assumption : Current flows in the same direction.
Using Right hand thumb rule, the direction of the magnetic field at point P due to current I2 is perpendicular to the plane of paper and inwards.
Similarly, at point Q on x2Y2, the direction of magnetic field due to current I1 is perpendicularly outward. ‘
Using Flemings left hand rule we can find the direction of forces F12 and F21 which are in opposite directions thus,
By Amperes circuital law, we have,
cbse-previous-year-solved-papers-class-12-physics-outside-delhi-2012-66
cbse-previous-year-solved-papers-class-12-physics-outside-delhi-2012-67
From above we get the magnitude of forces F12 and F21 are equal but in opposite direction. So,F12 = -F21 Therefore, two parallel straight conducting carrying current in the same direction attract each other. Similarly, we can prove if two parallel straight conductors carry currents in opposite direction, they repel each other with the same magnitude as equation (1).

SET II

Note : Except for the following questions, all the remaining questions have been asked in Set-I 2012.

Question.8. In a single-slit diffraction experiment, the width of the slit is made double the original width. How does this affect the size and intensity of the central diffraction band?
Answer : In single-slit diffraction experiment fringe width is given as
cbse-previous-year-solved-papers-class-12-physics-outside-delhi-2012-68
If d is doubled the width of the central maximum is halved. Thus there is a reduction in the size of the central diffraction band. Intensity of central band of the diffraction pattern varies square of the slit width so as the slit gets double, intensity will get four times.

Question.19. You are given three lens L1, L2, L3  each of focal length 15 cm. An object is kept at 20 cm in front of L1, as shown. The final real image is formed at the focus ‘I’ of L3. Find the separations between L1, L2 and L3

cbse-previous-year-solved-papers-class-12-physics-outside-delhi-2012-69
Here, image by L3 is formed at focus. So the object should lie at infinity of L3. Hence, L2 will produce image at infinity. So, we can conclude that object for L2 should be at its focus.
But, we have seen above the image by L1 is formed at 60 cm right of L1 which is at 15 cm left of L2 (focus of L2).
So, x1 = distance between Land L2 = (60 +15) cm = 75 cm
Again distance between Land L3 does not matter as the image by L2 is formed at infini ty so X2 can take any value.

Question.27. In a Geiger Marsden experiment, calculate the distance of closest approach to the nucleus of Z = 75, when an oc-particle of 5 MeV energy impinges on it before it comes momentarily to rest and re verse its direction.
How will the distance of closest approach be affected when the kinetic energy of the oc-particle is doubled ?
OR
The ground state energy of hydrogen atom is -13.6 eV. If an electron make a transition from an energy level -0.85 eV to -1.51 eV, calculate the wavelength of the spectral line emitted. To which series of hydrogen spectrum does this wavelength belong ?
Answer : Let ro be the centre to center distance between the alpha-particle and nucleus when the a-particle is at its stopping point.
cbse-previous-year-solved-papers-class-12-physics-outside-delhi-2012-70
cbse-previous-year-solved-papers-class-12-physics-outside-delhi-2012-71
cbse-previous-year-solved-papers-class-12-physics-outside-delhi-2012-72
cbse-previous-year-solved-papers-class-12-physics-outside-delhi-2012-73

SET III

Note : Except for the following questions, all the remaining questions have been asked in Set-I and Set-II 2012.

Question.1. How does the fringe width, in Young’s double-slit experiment change when the distance of separation between the slits and screen is doubled?
Answer: The fringe width becomes double when the distance of separation between the slits and screen is doubled.

Question.4.
cbse-previous-year-solved-papers-class-12-physics-outside-delhi-2012-74
Answer : The frequencies of electromagnetic waves have its inherent characteristics. When an electromagnetic wave travels from one medium to another, its wavelength changes but frequency remains unchanged.

Question.7. A proton and an electron have same kinetic energy. Which one has smaller de-Broglie wavelength and why ?
Answer : In terms of kinetic energy, wavelength is given by
cbse-previous-year-solved-papers-class-12-physics-outside-delhi-2012-75
an electron, thus a proton has smaller de-Broglie wavelength than a electron for the same kinetic energy.

Question.9.A circular coil of closely wound N turns and radius r carries a current I. Write the expression for the following:
(i) the magnetic field at its centre
(ii) the magnetic moment of this coil
cbse-previous-year-solved-papers-class-12-physics-outside-delhi-2012-76

Question.12.A light bulb is rated 150 W for 220 V AC supply of 60 Hz. Calculate
(i) the resistance of the bulb
(ii) the rms current through the bulb
OR
An alternating voltage given by V = 70 sin 100π is connected across a pure resistor of 25Ω. Find
(i) the frequency of the source.  (ii) the rms current through the resistor.
cbse-previous-year-solved-papers-class-12-physics-outside-delhi-2012-77
cbse-previous-year-solved-papers-class-12-physics-outside-delhi-2012-78

Question.20. Explain briefly the following terms used in communication system:

  1. Transducer
  2.  Repeater
  3. Amplification

Answer :

  1.  Transducer : Any device that converts one from of energy to another can be termed as transducer. An electrical transducer may be defined as a device that coverts some physical variable in the electrical signals.
  2. Repeater : A repeater is a combination of a receiver and a transmitter. A repeater picks up the signal from the transmitter, amplifies and retransmits it to the receiver sometimes with a,change in carrier frequency.
  3.  Amplification : It is a process of increasing the amplitude of a signal using an electronic circuit called the amplifier. Amplification is done at a place between the source and the destination.

Question.22. You are given three lens L1, L2 and L3  each of focal length 10 cm. An object is kept at 15 cm in front of L1, as shown. The final real image is formed at the focus ‘I’ of L3. Find the separations between L1, L2 and L3  .
cbse-previous-year-solved-papers-class-12-physics-outside-delhi-2012-79
Here, image by L3 is formed at focus. So the object should lie at infinity for L3. Hence, L2 will produce image at infinity. So, we can conclude that object for L2 should be at its focus. But, we have seen above that image by L1is formed at 30 cm right of L1which is at 10 cm left of L2 (focus of L2 ).
So, x1= distance between L1 and L2 = (30 +10) cm = 40 cm Again distance between L2 and L3 does not matter as the image by L2 is formed at infinity so X2 can take any value.

CBSE previous Year Solved Papers Class 12 Maths Delhi 2009

CBSE previous Year Solved  Papers  Class 12 Maths Delhi 2009

Time allowed : 3 hours                                                                                           Maximum Marks: 100

General Instructions:

  1.  All questions are compulsory.
  2.  Please check that this question paper contains 26 questions.
  3.  Questions 1-6 in Section A are very short-answer type questions carrying 1 mark each.
  4.  Questions 7-19 in Section B are long-answer I type questions carrying 4 marks each.
  5. Questions 20-26 in Section C are long-answer II type questions carrying 6 marks each.
  6.  Please write down the serial number of the question before attempting it.

SET I

Note: Except for the following questions, all the remaining questions have been asked in previous sets.

SECTION – A

Question.1.
cbse-previous-year-solved-papers-class-12-maths-delhi-2009-2-1
Solution.
cbse-previous-year-solved-papers-class-12-maths-delhi-2009-2-2
cbse-previous-year-solved-papers-class-12-maths-delhi-2009-2-3

Question.2. Write a unit vector in the direction of
cbse-previous-year-solved-papers-class-12-maths-delhi-2009-2-4
Solution.
cbse-previous-year-solved-papers-class-12-maths-delhi-2009-2-5

Question.3.
cbse-previous-year-solved-papers-class-12-maths-delhi-2009-2-6
Solution.
cbse-previous-year-solved-papers-class-12-maths-delhi-2009-2-7

Question.4. If matrix A = [12 3], write AA’, where A’ is transpose of matrix X
Solution.
cbse-previous-year-solved-papers-class-12-maths-delhi-2009-2-8

Question.5. 
cbse-previous-year-solved-papers-class-12-maths-delhi-2009-2-9
Solution.
cbse-previous-year-solved-papers-class-12-maths-delhi-2009-2-10

Question.6. Using principal value, evaluate the following
cbse-previous-year-solved-papers-class-12-maths-delhi-2009-2-11
Solution.
cbse-previous-year-solved-papers-class-12-maths-delhi-2009-2-12

Question.7.
cbse-previous-year-solved-papers-class-12-maths-delhi-2009-2-13
Solution.
cbse-previous-year-solved-papers-class-12-maths-delhi-2009-2-14

Question.8.
cbse-previous-year-solved-papers-class-12-maths-delhi-2009-2-15
Solution.
cbse-previous-year-solved-papers-class-12-maths-delhi-2009-2-16

Question.9.If the binary operation * on the set of integers Z, is defined by a * b = a + 3b2, then find the value of 2*4.
Solution.Given a*b = a + 3b2
... 2*4 = 2 + 3(4)2 = 2 + 3 x 16 = 50.

Question.10.If A is an invertible matrix of order 3 and |A| =5, then find |adj.A|.
Solution.
cbse-previous-year-solved-papers-class-12-maths-delhi-2009-2-17

SECTION – B

Question.11.
cbse-previous-year-solved-papers-class-12-maths-delhi-2009-2-18
Solution.
cbse-previous-year-solved-papers-class-12-maths-delhi-2009-2-19
cbse-previous-year-solved-papers-class-12-maths-delhi-2009-2-20

Question.12.
cbse-previous-year-solved-papers-class-12-maths-delhi-2009-2-21
Solution.
cbse-previous-year-solved-papers-class-12-maths-delhi-2009-2-22
cbse-previous-year-solved-papers-class-12-maths-delhi-2009-2-23
Solution.
cbse-previous-year-solved-papers-class-12-maths-delhi-2009-2-24
cbse-previous-year-solved-papers-class-12-maths-delhi-2009-2-25

Question.13.Find the value of A so that the lines
cbse-previous-year-solved-papers-class-12-maths-delhi-2009-2-26
perpendicular to each other.
Solution: Given equations are
cbse-previous-year-solved-papers-class-12-maths-delhi-2009-2-27
cbse-previous-year-solved-papers-class-12-maths-delhi-2009-2-28

Question.14.Solve the following differential equation:
cbse-previous-year-solved-papers-class-12-maths-delhi-2009-2-29
Solution.
cbse-previous-year-solved-papers-class-12-maths-delhi-2009-2-30
cbse-previous-year-solved-papers-class-12-maths-delhi-2009-2-31

Question.15.Find the particular solution, satisfying the given condition, for the following differential equation:
cbse-previous-year-solved-papers-class-12-maths-delhi-2009-2-32
cbse-previous-year-solved-papers-class-12-maths-delhi-2009-2-33

Question.16.By using properties of determinants, prove the following:
cbse-previous-year-solved-papers-class-12-maths-delhi-2009-2-34
Solution.
cbse-previous-year-solved-papers-class-12-maths-delhi-2009-2-35
cbse-previous-year-solved-papers-class-12-maths-delhi-2009-2-36

Question.17.A die is thrown again and again until three sixes are obtained. Find the probability of obtaining the third six in the sixth throw of the die.
Solution.
cbse-previous-year-solved-papers-class-12-maths-delhi-2009-2-37
Getting a six third time at the 6th throw means outcome at 6th place is a six. Out of first five throws, two outcomes are sixes, three outcome are non sixes.
cbse-previous-year-solved-papers-class-12-maths-delhi-2009-2-38

Question.18.Differentiate the following function w.r.t.
cbse-previous-year-solved-papers-class-12-maths-delhi-2009-2-39
Solution.
cbse-previous-year-solved-papers-class-12-maths-delhi-2009-2-40
cbse-previous-year-solved-papers-class-12-maths-delhi-2009-2-41

Question.19.
cbse-previous-year-solved-papers-class-12-maths-delhi-2009-2-42
Solution.
cbse-previous-year-solved-papers-class-12-maths-delhi-2009-2-43
cbse-previous-year-solved-papers-class-12-maths-delhi-2009-2-44
Solution.
cbse-previous-year-solved-papers-class-12-maths-delhi-2009-2-45

Question.20. Prove that the relation R in the set A = {1,2,3,4,5} given by R={(a, b): \a-b\ is even}, is an equivalence relation.
Solution: A = {1,2,3,4,5}
R = {(a, b): \ a – b \ is even}
For equivalence relation, the relation must be (i) reflexive, (ii) symmetric and (iii) transitive.
Now, R = {(1,3), (2,4), (3,5), (1,5), (1,1), (2,2), (3,3), (4,4), (5,5), (3,1), (4,2), (5,3), (5,1)}
(i) Reflexive: Since 1R1,2R2,3R3,4R4,5R5
...R is reflexive.
(ii)Symmetric: Since
cbse-previous-year-solved-papers-class-12-maths-delhi-2009-2-46

Question.21.
cbse-previous-year-solved-papers-class-12-maths-delhi-2009-2-47
Solution.
cbse-previous-year-solved-papers-class-12-maths-delhi-2009-2-48
cbse-previous-year-solved-papers-class-12-maths-delhi-2009-2-49

Question.22.
cbse-previous-year-solved-papers-class-12-maths-delhi-2009-2-50
Solution.
cbse-previous-year-solved-papers-class-12-maths-delhi-2009-2-51
cbse-previous-year-solved-papers-class-12-maths-delhi-2009-2-52
cbse-previous-year-solved-papers-class-12-maths-delhi-2009-2-53
cbse-previous-year-solved-papers-class-12-maths-delhi-2009-2-54

SECTION – C

Question.23.Find the volume of the largest cylinder that can be inscribed in a sphere of radius r.
Solution: Let the cylinder inscribed in the given sphere of radius r, have height h and base radius a.
In Right angled ΔAOB,
cbse-previous-year-solved-papers-class-12-maths-delhi-2009-2-55
cbse-previous-year-solved-papers-class-12-maths-delhi-2009-2-56
cbse-previous-year-solved-papers-class-12-maths-delhi-2009-2-57
A tank with rectangular base and rectangular sides, open at the top is to be constructed so that its depth is 2 m and volume is 8 m3. If building of tank costs Rs 70 per sq. metre for the base and Rs 45 per sq. metre for sides, what is the cost of least expensive tank ?
Solution:
cbse-previous-year-solved-papers-class-12-maths-delhi-2009-2-58
cbse-previous-year-solved-papers-class-12-maths-delhi-2009-2-59
cbse-previous-year-solved-papers-class-12-maths-delhi-2009-2-60

Question.24.A diet is to contain at least 80 units of vitamin A and 100 units of minerals. Two foods F1 and F2 are available. Food F1 costs Rs. 4 per unit and F2 costs ? 6 per unit. One unit of food F1 contains 3 units of vitamin A and 4 units of minerals. One unit of food F2 contains 6 units of vitamin A and 3 units of minerals. Formulate this as a linear programming problem and find graphically the minimum cost for diet that consists of mixture of these two foods and also meets the minimal nutritional requirements.
Solution:
cbse-previous-year-solved-papers-class-12-maths-delhi-2009-2-61
cbse-previous-year-solved-papers-class-12-maths-delhi-2009-2-62
cbse-previous-year-solved-papers-class-12-maths-delhi-2009-2-63
cbse-previous-year-solved-papers-class-12-maths-delhi-2009-2-64
cbse-previous-year-solved-papers-class-12-maths-delhi-2009-2-65

Question.25.Three bags contain balls as shownin the table below:
cbse-previous-year-solved-papers-class-12-maths-delhi-2009-2-66
A bag is chosen at random and two balls drawn from it. They happen to be white and red. What is the probability that they came from the III bag.
Solution.
cbse-previous-year-solved-papers-class-12-maths-delhi-2009-2-67
cbse-previous-year-solved-papers-class-12-maths-delhi-2009-2-68

Question.26.Usingmatrices, solve the following system of equations:
cbse-previous-year-solved-papers-class-12-maths-delhi-2009-2-69
Solution.
cbse-previous-year-solved-papers-class-12-maths-delhi-2009-2-70
cbse-previous-year-solved-papers-class-12-maths-delhi-2009-2-71
cbse-previous-year-solved-papers-class-12-maths-delhi-2009-2-72

Question.27.
cbse-previous-year-solved-papers-class-12-maths-delhi-2009-2-73
Solution.
cbse-previous-year-solved-papers-class-12-maths-delhi-2009-2-74
cbse-previous-year-solved-papers-class-12-maths-delhi-2009-2-75
cbse-previous-year-solved-papers-class-12-maths-delhi-2009-2-76

Question.28.Using the method of integration, find the area of the region bounded by the lines
2x + y = 4,3x – 2y = 6 and x – 3y + 5 = 0.
Solution.
cbse-previous-year-solved-papers-class-12-maths-delhi-2009-2-77
cbse-previous-year-solved-papers-class-12-maths-delhi-2009-2-78
cbse-previous-year-solved-papers-class-12-maths-delhi-2009-2-79
cbse-previous-year-solved-papers-class-12-maths-delhi-2009-2-80

Question.29. Find the equation of the plane passing through the point (-1,3,2) and perpendicular to each of the planes x + 2y + 3z = 5 and 3x + 3y + z = 0.
Solution : Let the equation of the plane through (-1,3,2) be
a(x + 1)+ b(y-3) + c(z-2) = 0 …(1)
cbse-previous-year-solved-papers-class-12-maths-delhi-2009-2-81

SET II

Note: Except for the following questions, all the remaining questions have been asked in previous sets.

SECTION-A

Question.2.
cbse-previous-year-solved-papers-class-12-maths-delhi-2009-2-83
Solution.
cbse-previous-year-solved-papers-class-12-maths-delhi-2009-2-84

Question.7.
cbse-previous-year-solved-papers-class-12-maths-delhi-2009-2-85
Solution.
cbse-previous-year-solved-papers-class-12-maths-delhi-2009-2-86

Question.11.Differentiate the following function w.r.t:
cbse-previous-year-solved-papers-class-12-maths-delhi-2009-2-87
Solution.
cbse-previous-year-solved-papers-class-12-maths-delhi-2009-2-88
cbse-previous-year-solved-papers-class-12-maths-delhi-2009-2-89
cbse-previous-year-solved-papers-class-12-maths-delhi-2009-2-90

Question.18.Find the value of A. so that the lines are perpendicular to each other.
cbse-previous-year-solved-papers-class-12-maths-delhi-2009-2-91
Solution.
cbse-previous-year-solved-papers-class-12-maths-delhi-2009-2-92
cbse-previous-year-solved-papers-class-12-maths-delhi-2009-2-93

Question.19.Solve the following differential equation:
cbse-previous-year-solved-papers-class-12-maths-delhi-2009-2-94
Solution.The given differential equation is
cbse-previous-year-solved-papers-class-12-maths-delhi-2009-2-95
cbse-previous-year-solved-papers-class-12-maths-delhi-2009-2-96

Question.21.Using properties of determinants, prove the following:
cbse-previous-year-solved-papers-class-12-maths-delhi-2009-2-97
Solution.
cbse-previous-year-solved-papers-class-12-maths-delhi-2009-2-98
cbse-previous-year-solved-papers-class-12-maths-delhi-2009-2-99

SECTION-C

Question.23.Two groups are competing for the position on the Board of Directors of a corporation. The probabilities that the first and the second groups will win are 0.6 and 0.4 respectively. Further, if the first group wins, the probability of introducing a new product is 0.7 and the corresponding probability is 0.3 if the second group wins. Find the probability that the new product was introduced by the second group.
Solution.
cbse-previous-year-solved-papers-class-12-maths-delhi-2009-2-100
cbse-previous-year-solved-papers-class-12-maths-delhi-2009-2-101

Question.26.Prove that the curves y2 = 4x and x2 = 4y divide area of the square bounded by x = 0, x = 4, y = 4 and y = 0 into three equal parts.
Solution.
cbse-previous-year-solved-papers-class-12-maths-delhi-2009-2-102
cbse-previous-year-solved-papers-class-12-maths-delhi-2009-2-103
cbse-previous-year-solved-papers-class-12-maths-delhi-2009-2-104
cbse-previous-year-solved-papers-class-12-maths-delhi-2009-2-105

SET III

Note: Except for the following questions, all the remaining questions have been asked in previous sets.

SECTION – A

Question.4.
cbse-previous-year-solved-papers-class-12-maths-delhi-2009-2-106
Solution.
cbse-previous-year-solved-papers-class-12-maths-delhi-2009-2-107
cbse-previous-year-solved-papers-class-12-maths-delhi-2009-2-108

Question.9.
cbse-previous-year-solved-papers-class-12-maths-delhi-2009-2-109
Solution.
cbse-previous-year-solved-papers-class-12-maths-delhi-2009-2-110

SECTION-B

Question.15.Using properties of determinants, prove the following:
cbse-previous-year-solved-papers-class-12-maths-delhi-2009-2-111
Solution.
cbse-previous-year-solved-papers-class-12-maths-delhi-2009-2-112
cbse-previous-year-solved-papers-class-12-maths-delhi-2009-2-113

Question.17.
cbse-previous-year-solved-papers-class-12-maths-delhi-2009-2-114
Solution.
cbse-previous-year-solved-papers-class-12-maths-delhi-2009-2-115
cbse-previous-year-solved-papers-class-12-maths-delhi-2009-2-116

Question.19.Solve the differential equation:
cbse-previous-year-solved-papers-class-12-maths-delhi-2009-2-117
Solution. Given differential equation is
cbse-previous-year-solved-papers-class-12-maths-delhi-2009-2-118
cbse-previous-year-solved-papers-class-12-maths-delhi-2009-2-119

Question.20.Find the value of X so that the following lines are perpendicular to each other
cbse-previous-year-solved-papers-class-12-maths-delhi-2009-2-120
cbse-previous-year-solved-papers-class-12-maths-delhi-2009-2-121

SECTION-C

Question.24.Find the area of the region enclosed between the two circles x2 + y2 = 9 and (x – 3) 2+ y2 = 9.
Solution. Equations of given circles are
cbse-previous-year-solved-papers-class-12-maths-delhi-2009-2-122
Equation (1) is a circle with centre at origin 0(0,0) and radius 3. Equations (2) is a circle with centre Q(3,0) and radius 3.
cbse-previous-year-solved-papers-class-12-maths-delhi-2009-2-123
Required area of bounded region OPQRO between two circles = area of bounded region OPRO + area of bounded region PQRP = 2 (Area in 1st quadrant).
cbse-previous-year-solved-papers-class-12-maths-delhi-2009-2-124
cbse-previous-year-solved-papers-class-12-maths-delhi-2009-2-125
cbse-previous-year-solved-papers-class-12-maths-delhi-2009-2-126

Question.27.There are three coins. One is a two headed coin (having head on both faces), another is biased coin that comes up tails 25% of the times and the third is an unbiased coin. One of the three coins is chosen at random and tossed, it shows heads, what is the probability that it was the two headed coin ?
Solution.
cbse-previous-year-solved-papers-class-12-maths-delhi-2009-2-127
cbse-previous-year-solved-papers-class-12-maths-delhi-2009-2-128

CBSE previous Year Solved Papers Class 12 Physical Education Outside Delhi 2016

CBSE previous Year Solved  Papers  Class 12 Physical Education Outside Delhi 2016

Time allowed : 3 hours                                                                                           Maximum Marks: 70

General Instructions:

  1.  The question paper consists of 26 questions.
  2. All question are compulsory.
  3. Answer to questions carrying 1 mark should be in approximately 10-20 words.
  4. Answer to questions carrying 3 marks should be in approximately 30-50 words ,
  5.  Answer to questions carrying 5 marks should be in approximately 75-100 words.

SECTION – A

Question.1. Write formula for giving Bye.
Answer : Formula for giving Bye = Next power of 2 – No. of Teams
cbse-previous-year-solved-papers-class-12-physical-education-outside-delhi-2016-1

Question.2.What are Natural Resources in Adventure sports ?
Answer: Natural resources are useful raw materials that we get from the earth. They can be used or utilized by people. Natural Resources in Adventure sports are national parks, Safaries and wildlife Sanctuaries are good places for out-door adventure activities. And so many thinks like light, air, water, stones, minerals, plants, animals, etc. people need these natural resources to live and stay alive.

Question.3.What are the Micro Nutrients ?
Answer: Vitamins, minerals are the micronutrients. These are required in small proportion in our diet like vitamins and minerals. They are considered as supplement food, they act as defensive food and help the proper functioning of various organs such vitamins, minerals.

Question.4.Suggest any two free hand exercises for correcting round shoulder.
Answer: 1. Chakra Asana, Dhanur Asana, Bhujang Asana, Ushtt Asaina, Backward Banding.
2.Pull the shoulders backward and see upward.

Question.5.  Explain the importance of fluid intake during competition.
Answer : During the activity or competition sports drinks helps extend time to exhaustion and improves performance for the competition lasting longer in
extreme heat. Sports drinks provide hydration as well as carbohydrate.

Question.6. What is Oesteoporosis ?
Answer: Osteoporosis is weakening of bone mass due to deficiency of calcium and vitamin D and results in poor bone. This condition declines the performance and leads to bone injuries.

Question.7. Explain the procedure for Eight Foot Up and Go Test.
Answer: This test is designed to measure speed, agility (coordinative ability) and balance while moving. The equipments required is stopwatch, straight back chair (approximately 45 cm high) cone markers, measuring tape, safe and clear area.

  1.  Place the chair against wall and mark a cone eight feet away in front of chair.
  2.  Clear the path between the chair and the marker.
  3.  The person sits on chair, hands resting on knees and feet flat on chair.
  4.  On the command ‘GO’ timing starts and person is instructed to walk (on running) around the cone as quickly as possible and sit down on the chair.
  5. Scoring is counted as time taken to perform this. Two trials are conducted and lowest time in seconds is considered for scoring.

Question.8. What is “Stroke Volume”?
Answer: Amount of blood pumped by left ventricle in per beat.
At rest period-50 to 70 ml/beat.
During Exercise-110 to 130 ml/beat OR
Amount of blood ejected by heart in one stroke.

Question.9. What kind of sports injury can be termed as “Abrasion” ?
Answer: Abrasion is injury on the surface of the skin. In this injury skin is scrapped or rubbed by friction.
It causes severe pain and sometimes bleeding over affected part. Abrasion are very common sports injuries caused by fall on hard surfaces..

Question.10. Explain, what is “Dynamic Friction” ?
Answer: Dynamic friction is the force of friction which come into play when a body moves over the sufrace of anotherbody.
(1) Sliding Friction (2) Rolling Friction.

Question.11. How participation in physical activities enhances self esteem among children ?
Answer: Self-esteem is based on our ability to assess ourselves accurately and still be accepting of who we are. This means being able to acknowledge our strengths and weakness and at the same time recognize that we are worthy and worthwhile.

Question.12. Explain, what is strength and write the methods of improving strength?
Answer: Muscular strength is the highest amount of effort exerted by the muscles of the body in order to overcome the most resistance in a single effort. Strength refers to a muscle’s ability to generate force against physical objects, Training methods for strength development.

  1. Isometric Exercises.
  2. Isotonic Exercises.
  3. Isokinetic Exercises.

Question.13. Write about the deformities of spinal curvature.
Answer : There are three types of spinal curvature deformities like kyphosis, lordosis and scolosis.

  1. Kyphosis ’round shoulders’: It is the postural defect in which shoulder are projected forward (shoulders are bent forward, the chin is downward and head is bent forward)
  2. Lordosis: Lordosis is the problem of lumbar-spine, lordosis is an inward curvatureo£a portion of the vertebral column. Here the vertebral column is curved backward and the individual leans backward while standing.
  3.  Scoliosis : It is the problem of spine in which vertebral column bends to sideward. Scoliosis causes one shoulder down and other is raised up.

Question.14. What safety measures children should be taught while participating in River Rafting?
Answer: Safety measures are safe raft swim suit, life jacket, raft push, knowledge of swimming and life guard instructions, knowledge about river flow and its curver, safe destination point.

Question.15. Draw a fixture of 11 Football teams participating in a Tournament on the basis of knock out.
Answer: Total number of teams =11
cbse-previous-year-solved-papers-class-12-physical-education-outside-delhi-2016-2
cbse-previous-year-solved-papers-class-12-physical-education-outside-delhi-2016-3

Question.16. How various committies are formed for Tournaments ? Write briefly.
Answer: Arrangement committee, Technical committee, Discipline committee, Reception committee, Boarding and lodging committee, Ceremonial committee, Press and media committee, Certificate writing committee, Medical committee, Announcement committee, Recorders and bulletin board committee.

Question.17. Sports are good for all age groups. Competitive sport
is a showcase of power at the international arena. Supremacy over the other country is indirectly shown by standing at the top of medal tally. To achieve their target few countries are imparting very strict training to growing children. Over the years many organisations have raised their voice against the intensity with which training is scheduled for making an international athletes.
(i) Elaborate any two physiological benefits of exercise in children.
(ii) What are the disadvantages of giving high intensity training to the growing children ?
Answer :
(i).Improves cardiovascular system and Health.
(ii).

  1. Stress/fatigue.
  2.  Chances of overload.
  3. Early maturity.
  4. Can reach too early top form.
  5. Less flexibility.
  6.  Chances of injury.

Question.18. Briefly explain different types of co-ordinative abilities.
Answer:

  1. Adaptive ability: According to predictable changes in circumstances, it is the capability of a sports person to result in an effectual alteration in the movement.
  2. Balance ability: It is the capability of a sports person to sustain balance of the body together in static and dynamic conditions.
  3.  Combinatory ability: This ability depends upon the functional capacity of kinesthetic and optic sense organs as it has particular significance of gymnastics, combative sports and team games.

Question.19. Write briefly about protein as an essential component of diet.
Answer: Protein is needed for growth and development of body. It helps to repair or replace the worn out tissues, it does not provide energy in normal routine where as it acts as energy source only Under extreme starvation (hunger) There are two types of proteins.

  1. Essential protein.
  2. Non – essential protein.

Question.20. Write in detail about the physical and physiological advantages of physical exercises during the childhood stage.
Answer :

  1. Exercise helps in healthy growth and development: Exercise is an important part of keeping children healthy. Encouraging healthy lifestyles in children and adolescents is important for when they grow older. Participating in organized sports and games is not only of great fun but is very essential for healthy growth and development.
  2.  Exercise improve self-esteem: Exercise is necessary for your physical and mental health. Self-esteem can play a great role in how children feel about themselves and also how much they enjoy things or worry about things. Exercise reduces depressive symptoms and improves self-esteem in children.
  3. Exercise makes heal their weight range: These days a lot of children not only in India but also throughout the world are getting overweight which is the root cause of various physiological problems. Regular exercise is important to control weight as it helps in burning extra calories and lowers the weight.
  4. Exercise makes stronger bones, muscles and joints : Exercise is vital for strong muscles, bones and joints. Exercise may help children lower their risk of chronic pain related to muscles, bones and joints in the future. Exercise increase bone density which helps to make bones stronger. Exercise is beneficial in building strong and health muscles. Joints require motion to stay healthy.
  5.  Exercise strengthens the heart: Exercise helps to improve heart health, and can even reverse some heart disease risk factors. The resting heart rate of those who exercise is also slower, because less effort is needed to pump blood. A child who exercises often has the lowest risk for heart disease, but any amount of exercise is beneficial’.

Question.21. How the Cardiovascular Fitness is measured with the help of “Harvard Step Test” ? Write in detail about its administrative procedure.
Answer: This test was developed in 1943 by Brouha to measure the cardiovascular fitness of the individual. Requirements of test:
1. Gym Bench (45cm high)    2. Stopwatch       3. Assistant /Helper.
In this test, the student step-up and step-down on the gym bench for 5 minutes or until exhaustion at a rate of 30 steps/minute.

  1.  Firstly, the student performs warm-up and comes near the gym bench.
  2.  At the command of ‘GO’ the student starts to step- up and step-down on the gym bench whereas assistant starts the stopwatch at same time.
  3.  This goes on for five minutes.
  4. The assistant measures the heart rate (beats per minute) after one minute of test finish as pulse 1.
  5. The assistant measures the heart rate (beats per minute) after two minutes of test finish as pulse 2.
  6.  The assistant again measures the heart rate (bpm) after three minutes of test finish as pulse 3.
    RUSELT = 30000/(pulse 1 + pulse 2 +pulse 3)

Question.22. Keeping in view of the Indian Ideology, critically analyse the sociological aspect of Women Athletes in sports participation.
Answer: It has been decades that gender ideology has had a tremendous impact on the way people view the game and its athletes. Over the generations, it has been considened that sports is predominantly for man, and woman are not that adept at such plysical acts. In India till recent year, woman has been threated and accorded only for their roles of being wife, mother and home-maker. It has only in 1930, that woman began to participate in sports.
Time and again, the government policies has helped rural women of different regions but then several drawbacks still occurred. The belief that men are much stonger, faster and more capable than women, and more entertain to watch has been a major provoking influence to the mindset of people over generations. Some of the ways through which the sociological aspect of women Athletes in sports participation has highlighted are: Equality in society, motivation from parents, encouragement for institutions, respect in society, develop confidence, safe playing environment, good coaching and training facility, financial Aid, government policies.

Question.23. Participation in physical activity for a longer duration maintain functional fitness among aged population. . Justify.
Answer: On the ageing process, physical activity can play very effective role, however, it should be implemented in a suitable manner. It is a well known fact that regular physical activity is the supplement to delay the ageing process. It can in some cases, overturn the most common signs of ageing viz., loss of muscles and bones, increased body fats, decline of memory, decreased metabolism, flexibility and blood flow etc. Participation in physical activity for a longer duration maintain functional fitness among aged population in following manner:

  1. Reduces loss of muscles mass : With advancing age the muscle mass decreases. The accumulation of fats reduces due to regular exercise. Ageing has a depressing consequence on metabolism. Regular physical activity results in dropping the metabolic rate and decreases the loss of lean body mass.
  2. Maintains bone density: With advancement of age, bone density decreases as well generally leading to the rupture or osteoporosis. Phycical activity assist in mainting bone mass and prevent osteoporosis. Bone growth gets stimulated with resistance exercises.
  3. Improves lungs capacity : Oxygen uptake and exchange increases and enhances the lungs capacity due to regular exercise. In maintenance of strong lungs, it plays prominent role and reduces the loss of elasticity of the lungs and chest wall.
  4. Lessens stress and tension : Regular physical activity has a distinctive capability to slow down the depression process by reducing stress and tension. Actually, regular physical activity lessens the levels of body as stress hormones like adrenaline and cortisol. The body’s natural painkillers and mood elevators like the endorphins are produced due to regular physical activity.

Question.24. Differentiate between Intrinsic and Extrinsic motivation. Explain in detail Goal setting and Rein-forcement as technique of motivation.
Answer:
cbse-previous-year-solved-papers-class-12-physical-education-outside-delhi-2016-4
cbse-previous-year-solved-papers-class-12-physical-education-outside-delhi-2016-5
Goal setting: Achieving performance goals is a symbol of competency that affects motivation positively, hence it is essential to set realistic goals based on individual’s own abilities. The level of the motivation gets adversely affected when goals are set up too high or too low. The goal should be realistic, precise and within reach but should at the same time be a difficult one. Goal settings have been acknowledged as an influential motivational technique as it mobilizes an athlete’s hard work and extends his determination.
Reinforcement: Reinforcement is a vital motivational means. It refers to some kind of occurrence that increases or decreases the posibility of a similar reaction taking place in the future. Positive reinforcements enlighten the athlete at what time he is doing something accurate and support the continuance of the activity in the precise direction. Negative reinforcement is in general, of slight importance since it simply indicates that the actions are inaccurate devoid of providing information with respect to the accurate reaction or behaviour. It has been established that positive reinforcement has to a large extent, more motivational value than negative reinforcement.

Question.25. Write in detail about the Dislocation and Fractures among the bones and joint injuries.
Answer: Dislocation: It is an injury of joint in which , the adjoining bones are displaced from their original position. Dislocations mainly occurs in contact sports, such as football, hockey, and in sports that may involve falls, such as downhill skiing, gymnastics, volleyball and also during a motor vehicle accidents.
Dislocation takes place in different location of the joints like:

  1.  Dislocation of shoulder joint.
  2. Dislocation of lower jaw.
  3.  Dislocation of hip joint.

Fracture: Bone injuries occur in the bone due to some impact or minimal trauma injury as a result of certain medical conditions that weaken the bone. There are many types of bone injuries which are as follows:

  1.  Simpe fracture
  2.  Compound fracture
  3. Greenstick fracture
  4.  Comminuted fracture
  5.  Transverse fracture
  6. Oblique fracture
  7. Spiral fracture
  8. Pathologic fracture
  9.  Impacted fracture
  10.  Stress fracture.

Question.26. Differentiate mechanically between walking and running. Explain in detail mechanical analysis of walking.
Answer: Walking: Walking is rolling down from heel to toe action. The first phase is heel strikes the ground followed by the transfer of weight and balance making. Afterwards the heel is rolled forward and push is given by toe and swing comes after, During walking the head should be straight without tension in shoulder. In the torso region the abdomen muscles should be slightly tightened, loose abdomen muscles cause back pain. The arm swing helps to gain speed in walking.
Running : running is a toe action movement. For running the focus is on the three major producing action- push off, knee-drive and paw-back. These three phases are done in a sequential manner to provide efficient running action, push off is the primary toe action movement heeded to supply the driving force in forward direction. The force comes from the powerful contraction of calf muscles by extension of ankle joint. The slight flexion of knee joint provides horizontal force instead of vertical force. Body is also bent forward.

CBSE previous Year Solved Papers Class 12 English Outside Delhi 2010

CBSE previous Year Solved  Papers  Class 12 English Outside Delhi 2010

Time allowed : 3 hours                                                                                           Maximum Marks: 100
General Instructions :

  1. This paper is divided into three sections : A, B and C. All the sections are compulsory.
  2. Separate instructions are given with each section and question, wherever necessary. Read these instructions very carefully and follow them faithfully.
  3. Do not exceed the prescribed word limit while answering the questions.

SET I

SECTION —A
(READING)

Question.1. Read the passage given below and answer the questions that follow:

  1.  Today’s woman is a highly self-directed person, alive to the sense of her dignity and the importance of her functions in the private domestic domain and the public domain of the world of work. Women are rational in approach, careful in handling situations and want to do things as best as possible. The Fourth World Conference of Women, held in Beijing in September 1995, had emphasized that no enduring solution of society’s most threatening social, economic and political problems could be found without the participation and empowerment of the women. The 1995, World Summit for Social Development had also emphasized the pivotal role of women in eradicating poverty and mending the social fabric.
  2.  The Constitution of India had conferred on women equal rights and opportunities – political, social, educational and of employment – with men. Because of the oppressive traditions, superstitions, exploitation and corruption, a majority of women are not allowed to enjoy the rights and opportunities, bestowed on them. One of the major reasons for this state of affairs is the lack of literacy and awareness among women. Education is the main instrument through which we can narrow down, the prevailing inequality and accelerate the process of economic and political change in the status of women. ‘
  3.  The role of women in a society is very important. Women’s education is the key to a better life in the future. A recent World Bank study says that educating girls is not a charity, it is good economics and if developing nations are to eradicate poverty, they must educate the girls. The report says, that the economic and social returns on investments in education of the girls considerably affect the Human Development Index of the nation. Society would progress only if the status of women is respected and the presence of an educated woman in the family would ensure education of the family itself. Education and empowerment of women are closely related.
  4. Women’s education has not received due care and attention from the planners and policy makers. The National Commission for women has rightly pointed out that even after 50 years of Independence, women continue to be treated as .the single-largest group of backward citizens of India. The role of women in overall development has not been fully understood nor has it been given its full weight
    in the struggle to eliminate poverty, hunger, injustice and inequality at the national level. Even, when we are at the threshold of the 21st century, our society still discriminates against women in matters of their rights and privileges and prevents them from participating in the process of national and societal progress. Various committees and commissions have been constituted before and after Independence to evaluate the progress in women’s education and to suggest ways and means to enhance the status of women. The female literacy rate has gone up in the 20th century from 0.6 percent in 1901 to 39.29 percent in 1991 but India still possesses the largest number of illiterate women in the world. The Female Literacy Index for the year 1991 shows that there are eight states which fall below the national average. The most populous states of the country, UP, MP, Bihar and Rajasthan fall in the category of most backward states as far as female literacy is concerned. .
  5.  The prevailing cultural norms of gender behaviour and . the perceived domestic and reproductive roles of women tend to affect the education of the.girls. Negative attitude towards sending girls to schools, restrictions on their mobility, early marriage, poverty and illiteracy of parents affect the girls’ participation in education.
  6.  Women’s political empowerment got a big boost with the Panchayati Raj Act of 1993 which gave them 30 percent reservation in Village Panchayats, Block samities and Zila Parishads throughout the country. The National Commission for women was also set up in 1992, to act as a lobby for women’s issues.
  7.  The educational system is the only institution which can counteract the deep foundations of inequality of sexes that are built in the minds of people through the socialisation process. Education, is the most important instrument of human resource development. Educational system should be used to revolution is  the traditional attitudes and inculcate new values of equality.

(a) (i) Mention any two attributes of a modem woman.
Answer : (i) Women are careful in handling situations.
(ii) They are rational in their approach.
(ii) Why is women’s participation and empowerment
considered necessary ?
Answer : Society’s most threatening social, economic and political problem’s solution could not be found without the participation and empowerment of women.
(iii) Which factors adversely affect the education of
girls ?
Answer : The prevailing cultural norms of gender bias ness and the perceived domestic and reproductive roles of women tend to affect the education of the girls.
(iv) What benefits did the women get with the enactment of the Panchayat Raj Act of 1993 ?
Answer: The women got 30% reservation in village Panchayats, Block samities and Zila parishads, throughout the country with the enactment of the Panchayat Raj Act of 1993.
(v) By what process can we remove the sense of inequality of sexes from the minds of the people ?
Answer : The educational system is the only institution which can remove the sense of inequality of sexes from the minds of the people through the socialisation process.
(b) Pick out words from the passage which mean the same as each of the following:
(i) Gruel and unfair (para 2)
Answer: oppressive
(ii) Remove (para 3)
Answer: eradicate
(iii) full of people (para 4)
Answer: populous

Question.2. Read the passage given below and answer the questions that follow:
Despite all the research, each one of us catches cold and most of us catch it frequently. Our failure to control one of the commonest of all ailments, sometimes seems ridiculous. Medical Science regularly practices transplant surgery and has rid many a countries of such killing diseases as Typhus and the Plague. But the problem of common cold, is unusually difficult and much is yet to be done to solve it.
It is known that a cold is caused by one of a number of viral infections that affect the lining of the nose and other passage leading to the lungs but the confusing variety of viruses makes study and remedy very difficult. It was shown in 1960 that many typical colds in adults are caused by one or the other of a family of viruses known as rhinoviruses, yet there still remain many colds for which no virus has yet been isolated. There is also the difficulty that because they are so much smaller, than the bacteria, which causes many other infections, viruses cannot be seen with ordinary microscopes. Nor can they be cultivated easily in the bacteriologists laboratory, since they only grow within the living cells of animals or plants. An important recent step forward, however, is the development of the technique of tissue culture, in which bits of animal tissue are enabled and to multiply independently of the body. This has greatly aided virus research and has led to the discovery of a large number of viruses. Their ^existence had previously been not only unknown but even unsuspected. .
The fact that we can catch cold repeatedly creates another difficulty. Usually a virus strikes only once and leaves the victim immune to further attacks. Still we do not gain immunity from colds. Why ? It may possibly be due to the
fact that while other viruses get into the bloodstream where antibodies can oppose them — the viruses causing cold attack cells only on the surface. Or it may be that immunity from one of the many different viruses does not guarantee protection from all others. It seems, therefore, that we are likely to have to suffer colds for some time yet.
(a) On the basis of your reading of the above passage, make notes on it in points only, using abbreviations wherever necessary. Also suggest a suitable title.
Answer:
Title: Difficulties of Common Cold.
1. Notes
(i) Common cold
(a) everyone catches
(b) Med Sc. failed to control
(c) much has to be done in order to cure it.
(ii)causes
(a) viral infections
(b) variety of viruses
(c) family virus known as Rhino.
(iii)other problems
(a) too small to be seen under microscp
(b) leaves victim immune to further attacks
(c) can’t be cured by anti bods.
cbse-previous-year-solved-papers-class-12-english-outside-delhi-2010-1
(b) Write a summary of the above passage in about 80 words using the notes made.

SUMMARY

Everyone catches cold and the cure of it has not been found yet. Medical Science has failed to control it. It is caused by viral infections that affect the lining of the nose and other passages. The confusing variety of viruses, makes the study difficult. Family virus rhinovirus is also responsible for it. The viruses are so small that they can neither be seen with microscope nor cultivated in laboratory. This virus leaves the. victim immune to further attacks. They do not get into the bloodstream where antibodies can oppose them. Therefore, we have to suffer from cold for some time yet.

SECTION – B
(ADVANCED WRITING SKILLS)

Question.3. You are the Secretary of your School Literary Association. Write a notice in not more than 50 words for your school notice board, giving details of the proposed inauguration of the Literary Association of your school. You are ‘XYZ’ of Jain Vidyashram, Cuddalore. 
OR
You are the General Manager of E.V.L. Company which requires posh bungalows on company lease, as guest houses. Draft an advertisement in not more than 50 words under classified columns to be published in “The New Indian Express”1.
Answer:
cbse-previous-year-solved-papers-class-12-english-outside-delhi-2010-2
OR
cbse-previous-year-solved-papers-class-12-english-outside-delhi-2010-3

Question.4.Your school organized an exhibition-cum-sale of the items prepared under Work Experience by your school students. There was an overwhelming response from the public. Prepare a report in 100-125 words for a local daily. You are the Co-ordinator, S.U.P.W activities, Nita School, Gurgaon.
OR
Your school organized a seminar on Swine Flu for creating awareness among students of your school, and then- parents. As Secretary of the Health Club of Oasis Public School, Hyderabad, write a report in 100-125 words for ‘The Deccan Herald’.
Answer:
Exhibition-Cum-Sale
Report: Co-Ordinator (S.U.PW. Activities)
Nita School, Gurgaon
12th February, 20XX .
An Exhibition Cum Sale of the items prepared by students of our school under Work Experience was organized in the school premises on 3rd and 4th February. The exhibition was inaugurated by honorable Principal, Mr. N.M. Pareek. Parents as well as public were cordinally invited to watch the efforts of the children. Students showed their skills with exuberance. The items made by the students included models and products made with discarded material which has some utility. As the name itself suggests, Socially Useful Productive Work, the exhibition proved to be truly useful and the sale was also high. The exhibition showcased the creativity and talents of the students’ right from the junior section to the senior ones. It was applauded by one and all. The effort of the students was commendable and the response from the public was overwhelming which encouraged more and more students to be a part of S.U.P.W.
OR
A Seminar on Swine Flu
Report: Secretary (Health Club)
Oasis Public School, Hyderabad
25th March, 20XX
A seminar on ‘Swine Flu’ was organized on 20th March, 20XX in Oasis Public School, Hyderabad to create awareness among students and their parents. School chairman Mr. L.N. Sharma was the chief guest. In his welcome speech, he told that swine influenza, is a contagious respiratory disease that normally effects pigs but has harmful effects on human beings also, if they come in contact with infected pigs. The seminar was attended by several eminent doctors and medical officers who suggested ways to cure this disease. Students from “* other schools were also invited to present their views on the symptoms of the Flu and the precautions to be taken. It was told by the doctors, that although the vaccines for swine flu are available in medical stores but if not cured in time, it may • prove to be fatal. The parents found this seminar extremely beneficial as they also became aware of the causes and cures of this disease. A team of medical experts told that it is just like seasonal flu and can be reduced by maintaining good hygiene.

Question.5.Write a letter to the Editor, ‘The Hindu’, Chennai about rash and reckless driving by the people in your city, suggesting preventive measures. You are Kamal/Kanwar of 10, Mount Road, Velacherry.
OR
You have shifted your residence froih 10, Lajpat Road to House No. 232, Aurobindo Marg, Delhi. Write a letter to the General Manager, MTNL, requesting him for an early transfer of your telephone line. You are Rohit /Radhika of 15, The Mall, Amritsar.
Answer:
10, Mount Road,
Velacherry 10th April, 20XX
The Editor The Hindu
Chennai
Subject: Rash and Reckless driving
Dear Sir,
Through the columns of your esteemed newspaper, I would like to share my views on the increasing cases of rash and reckless driving in the city. This has created havoc in the city as it has become a routine habit of the people to drive at a very fast speed even in the colonies which results into one or two cases of accidents. Due to the negligence of the people, sometimes small children playing in the lanes become victims. The people who are responsible for these accidents are generally teenagers who are given bikes by their parents or they borrow it from their friends tp show off. I have noticed many times that they do stunts on the busy roads which is not only risky for others’ lives but their lives too. Number of accidents have been increasing daily which sometimes results in the loss of life. This should be controlled and traffic police should take strict action against it. Teenagers should not be given the permission to drive until they are perfecdy trained to drive on the roads. If the driver is found, the registration of the vehicle should be cancelled or the driving license should be seized.
Hope this letter would leave an impact on the public. Thanking you Yours Sincerely,
Kamal (A resident)
OR
Answer:
15, The Mall Amritsar .
18th June, 20XX
The General Manager MTNL •
Subject: Early transfer of telephone line
Dear Sir,
This is to inform you that I, Radhika of 15, The Mall, Amritsar, have shifted my residence from 10, Lajpat Road to House No. 232, Aurobindo Marg, Delhi for a minimum period of three years. You know that Delhi is a vast city and the distances are too much that it is impossible to reach
anywhere within a short time. In such a situation the only means of communication left with us is the telephone. But getting the telephone line transferred immediately after shifting will take a lot of time. So I request you for an early transfer of telephone line to avoid any inconvenience. I am submitting all the papers required for the transfer of line. Thanking You Yours Sincerely,
Radhika

Question.6. As compared to the older generation the youth of today are gteady inclined to pursue adventurous activities either for money or for fun. There is a latest craze for joining reality shows, rafting, rock climbing, mountaineering etc. Write an article in 150-200 words on which life would you prefer- safe or adventurous.
OR
Science attempts to explore the secrets of nature while religion wants to reveal the very purpose of existence. The aim of both Science and Religion is to discover the truth. Write an article in 150-200 words on ‘Science and Religion’.
Answer:
‘Which Life Would You Prefer—Safe or Adventurous ?’
Adventurous life has attracted youth a lot and it has become the fastest growing charm. It result due to either fun or money. That is why adventure tourism has grown in decades. Adventure life excites and rejuvenates a man; so more and more youth are fascinated by it. Experiencing the life in the lap of nature rock climbing, river rafting, jungle safari, mountaineering, skiing etc. is totally a different experience. Now-a-day’s adventure tour operators are flourishing only because of the increasing popularity of adventurous trips by people. They enjoy new experience and seek out thrills, but, it involves risk equally. These activities signify danger and attract people who are ready to gamble their personal safety. They risk their life in search of a rush of excitement or an unusual accomplishment. The reality shows have also prompted our youth to take part in risky adventurous games by promising fabulous prizes and stardom. Some people try these stunts at home putting their life in extreme danger. It is impossible to understand why people engage in such activities.
OR
Science And Religion
Science and Religion, we can say are the two sides of a coin. The relationship between Science and Religion has been a subject of study since ages. It has been a subject of conflict and harmony among different regions, cultures and Historians. Both science and religion pursue knowledge of the universe and search truth using different7 methods. The only difference is that Science needs reasons and evidence whereas Religion relies on faith and revelation. Although Scientific innovations are all based on the religious findings, they claim which to be their own researches. Science tries to find the secrets of nature while Religion tells the reason of our existence. Religion is based upon trust quite away from the arguments presented by Science. That is why they are termed as two sides to a coin. Scientific facts may be influenced by religion but many people reject this idea. Religion stands upon tradition whereas science is factual but the objectives of both Science and Religion is human welfare. In fact, both are complementary to each other

SECTION – C
(TEXT BOOK)

Question.7. Read the extract given below and answer the questions that follow:
… on their slag heap, these children
Wear skins peeped through by bones and spectacles of steel
With mended glass, like bottle bits on stones.
All of their time and space are foggy slum So blot their maps with slums as big as doom.
(a) Which two images are used to describe these slums ?
Answer : The two images used to describe these slums are :
(i) Foggy slums
(ii) Slums as big as’deom.
(b) What sort of life do these children lead ?
Answer : These children lead a miserable life confined to their dark and damp slums. Their childhood is lost on slag and heaps of garbage.
(c) Which figure of speech is used in the last line ?
Answer: Simile

Question.8. Answer any three of the following in 30-40 words each :
(a) What is the significance of the parting words of the poetess and her smile in ‘My Mother at Sixty-Six’ ?
Answer: ‘The parting words’…. See you soon Amma’ means that the poetess is doubtful about meeting her mother again as she is frail and too old.
(b) According to the poet, what is it that human beings can learn from nature ? (Keeping Quiet)
Answer : Human beings can learn to construct rather than destruct, from nature. It teaches us the art of preserving life, beneath the apparent dark surfaces, there is life.
(c) What makes human beings love life in spite of troubles and sufferings ? (A Thing of Beauty)
Answer : The beauty of nature, rivers, mountains, fresh air, flowers, the sun, the moon, the green pastures and all beautiful things make human beings love life.
(d) What will happen to Aunt Jennifer’s tigers when she is dead ?
Answer : The tigers created by Aunt Jennifer will keep on prancing in the panel after her death!

Question.9. Answer the following in 30-40 words each :
(a) What changes came over little Franz after he heard M. Hamel’s announcement ? (The Last Lesson)
Answer : When Franz came to know about M. Hamel’s announcement, he was shocked. He felt sad and regretted for not being attentive in the class and serious about his studies.
(b) Why does the author say that the Bangle makers are caught in a vicious web ? (Lost Spring)
Answer : The Bangle makers are said to be caught in a vicious web because they are exploited by sahukars, middlemen, policemen, bureaucrats and politicians. They are not able to make their co-operative societies for their benefit.
(c) Why was Edla happy to see the gift left by the peddler ? (The Rattrap)
Answer:When Edla heard about the theft at the Crofters house, she became sad. On reaching home she discovered that the peddler had not taken anything from her house and left a small rattrap as a gift and three, ten stolen kronors, because of which she became extremely happy.
(d) Why did Sophie like her brother Geoff more than any other person ?
Answer : Sophie liked her brother Geoff more than any other person because he was the only one who took interest in her imaginary wild stories. Moreover, he never revealed her secret to anyone and being a motor mechanic, she used to roam with him everywhere.

Question.10. Answer the following in 125-150 words :
“… there was terror in my heart at the overpowering force of the waves”. When did Douglas start fearing water ? “Which experience had further strengthened its hold on his mind and personality ?
Answer: Douglas’ first experience of water was horrific. He was with his father on a beach in California when a strong wave swept him over and terrified him completely. So Douglas was eleven years old when he started learning swimming in Y.M.C.A. pool. It was safe, as its depth at the end was only two feet. However the further end was nine feet deep. One day he was sitting alone on the edge of the pool when a strong young man threw him into the deep end of the pool for fun. He was nearly drowned. He could not come out of water and thus left the idea of swimming. The fear of water persisted with him for many years.
Then he decided to overcome his fear of water because he could no more enjoy water sports like canoeing, boating, swimming, etc. Moreover, he could not enjoy river bath. He had faith in the words spoken by Roosevelt, “All we have to fear is the fear itself”, .which gave him strength. He hired
an instructor to learn swimming. The instructor put all his efforts to turn him into a good swimmer.

Question.11. Answer the following in 125-150 words :
Why did Mr. Lamb try to give courage and confidence to Derry ?
Answer : Derry and Mr. Lamb shared the same problem of loneliness as they were physically disabled. They both lived a secluded life. Mr. Lamb had an Apple garden and he kept his gates always open. When Derry came to his garden, he welcomed him. He found a companion in him as Derry was too suffering the same pain as Mr. Lamb was. Mr. Lamb was disabled but was calm and maintained peace with people around him and he infused the same feeling in Derry. He encouraged Derry . also to face his disability boldly. Before coming in contact with Mr. Lamb, Derry felt lonely due to the behaviour of the people- but Lamb inspired him and filled him with self-confidence and determination. He taught him to move ahead in life as his brain and senses were working fully. He imbibed positive thinking in him and developed a kind of sympathy with him, thus showing him the way to live in this world in spite of his disability.

Question.12. Answer the following in 30-40 words each :
(a) What did the British Officer’s Secretary tell the Maharaja ? Why did the Maharaja refuse permission ?
Answer : The British Officers Secretary revealed his desire to kill the tigers but the Maharaja refused the permission as according, to an astrologer, his death would be caused by a tiger and if he allowed him to kill the tiger, the number of tigers would be reduced.
(b) Why did the messenger come to Dr. Sadao ? What did
Hana think about it ?
Answer : Since The General was extremely ill and would require 2011 Dr. Sadao’s service any moment for an operation, therefore a messenger was sent to call him. When he came to his house, Hana thought that he had come to arrest him on account of giving shelter to an enemy.
(c) Why did Evans not take off his hat when Jackson ordered him to do so ?
Answer : Evans had curly hair but he had shaved his head to remain in disguise. So he did not take off his hat when Jackson ordered him to do so. Instead he told Jackson that, the hat was lucky for him.
(d) Why was Zitkala-Sa terrified when Judewin told her that her hair would be cut short ?
Answer : Zitkala-Sa was terrified when Judewin told her that her hair would be cut short because in her community, keeping long hair was the symbol of bravery and only those warriors who were punished, had their hair shingled.

SET II

Note : Except for the following questions, all the remaining questions have been asked in previous sets.

SECTION – B
(ADVANCED WRITING SKILLS)

Question.5. Write a letter to the Secretary, Youth Hostels, Jaipur, requesting him to provide accommodation for 3 days to 4 members of your family going on a tour of Rajasthan. You are Kamini / Kamlesh, Gandhinagar, Chennai.
Answer:
Gandhinagar,
Chennai
20th June, 20XX
The Secretary
Youth Hostel, Jaipur
Subject: Accommodation for 4 members for three days
Dear Sir,
This is to inform you that a tour to Rajasthan has been planned by my family. You, are requested to arrange for an accommodation for four members for three days in your hostel. Kindly arrange for; food also. As it’s a new place for
us, we would be obliged if you could arrange a taxi from 10 a.m. to 5.p.m. at reasonable rates. Please ensure that the cab driver has been registered with the government authority and should be honest as we come across few cases of unscrupulous ways of drivers on a daily basis.
Payment will be done only after checking out and it will be according to your terms and conditions provided that all the facilities will be given to us on time.
Thanking You Yours Sincerely,
Kamini

Question.6.It is due to cable television that programmes are brought to our homes day and night for all the seven days in a week. It has brought a lot of change in the lifestyle of the people. Write an article in 150-200 words on ‘Cable TV-A Boon or A Bane’.
OR
India is standing at the threshold of joining the developed nations but that is not possible till we achieve complete literacy in the country. The contribution of students may be very significant in achieving our goals. Write an article in 150-200 words on “The Role of the Students in Removing Illiteracy”.
Answer:
‘Cable TV-A Boon or A Bane’
On an average, a person spends 40% of his time watching TV. In the last two decades, television has become the most powerful means of entertainment. On top of it, cable TV has brought a revolution in the lifestyle of people. The cable TV has invaded our rooms and we have become addicted to it. In case there is power failure or fault in cable, we feel paralyzed and it seems as if something is missing from our daily routine.
Nobody, has remained unaffected by its influence, be it children or the aged. From films to cartoons, news channels to sports channels, serials to music channels, every channel has its unlimited viewers. Advertisements have their own positive as well as negative effects both on youth and elders. Many times we shirk away our important work for the sake of watching a movie or a daily soap. There is repeated telecast also of the programs and that is not only once, but twice or thrice. Our outdoor activities, reading habits, social life has also not remained unaffected by watching TV. We prefer staying at home, confined to our rooms, watching television instead of going out. That is why it is called the “Idiot Box”. We should find better alternatives like going for picnic, playing outdoor games, visiting relatives etc. instead of merely watching television.
OR
The Role of the Students
in Removing Illiteracy
Any country’s progress depends on its youth. One cannot deny their importance in building and upliftment of a nation. They play a vital role in the development of a country. It is a known fact that in India, there are mostly villages, and the people living in these villages are mostly illiterate. They don’t even know to write their names and recognition of letters is far beyond from their reach. So it, becomes the duty of the youth to dedicate their services to teach the, illiterate but also the most important community of our country. The villagers are bound by the superstitions and it is necessary to remove these beliefs from their minds. The students are the only ones who can change their mindset and transform them. They can easily teach them after their school hours and tell them the new techniques which would be helpful in their farming process. The students should divide themselves in groups of four or five and start giving evening classes to the illiterate people of their respective areas. With the increase in the population, the responsibilities of students have also increased. Their role is full of challenges and they, can only bring revolutionary changes in making India a progressive and prosperous country.

SECTION – C
(TEXT BOOK)

Question.12. Answer the following in 30-40 words each :
(a) Why do you think, was the Maharaja in danger of losing his throne ?
Answer : According, to an astrologer’s prediction, the Maharaja started to kill tigers, but when a high ranked British Official wished to hunt tigers, the Maharaja did not allow him and even refused to get him photographed near the carcass of tiger. Thus, he was in danger of losing his kingdom.
(b) What forced Dr. Sadao to be impatient and irritated with his patient ?
Answer : Dr. Sadao was sheltering an enemy by putting his position and life in high risk. This secret was soon aired and he became worried. Therefore he thought of a plan to get rid of his patient.

SET III

Note : Except for the following questions, all the remaining questions have been asked in previous sets.

SECTION – B
(ADVANCED WRITING SKILLS)

Question.5.There is a flood of advertisements on television channels these days. Useless commodities and even superstitious beliefs are promoted through glamorous and exaggerated* presentations. Write a letter to the Editor, ‘New Indian Express’ about the negative influence such advertisements have on the minds of the people. You are Radha/Ramesh of MayurVihar, Lucknow.
OR
Write a letter to the Manager, Haldirams, Nagpur, requesting him to supply 60 lunch packets for your excursion party which would be travelling by G.T. Express from Delhi to Chennai. Your train would reach Nagpur at 12.15 p.m. Mention your coach number and the other relevant details. You are Priti/Pritam, Co-ordinator of the Excursion Party of Sriram Vidya Niketan, Delhi.
Answer:
Mayur Vihar,
Lucknow
23rd October, 20XX
The Editor
New Indian Express
Subject: Negative Influence of Advertisements .
Dear Sir,
Through the esteemed columns of your newspaper, I would like to bring to your notice the influence that the advertisements have on the minds of the public. The aim of the advertisements is to present a commodity in the best way and promote its selling. This dramatization is done by the celebrities from the field of film line, sports etc. They have positive and negative impact both, on our minds. But most of time these ads lure people by representing their product , falsely. Based on the images of fairness creams, women often feel attracted towards them and they spend a lot of money buying them. These ads communicate false images of men also which are unrealistic. So, these ads should not be promoted and action should be taken against the companies for promoting their false product.
Hope my letter would drive a positive response.
Thanking You Yours Sincerely,
Radha (Mayur Vihar)
OR
Sriram Vidya Niketan
Delhi ,
14th April, 20XX The Manager Haldirams, Nagpur
Subject: Supply of 60 lunch packets for excursion party.
Dear Sir,
This is to inform you that an excursion party to Chennai is being organized for 60 members. These people will be travelling by G.T. Express from Delhi to Chennai via Nagpur. The train will reach Nagpur station at 12.15 p.m. and stop for about half an hour. The party will be requiring lunch there as after that the train will not halt at any big station.
Therefore, I request you to arrange for 60 lunch packets for the party. It must include Chapati, dal, mix vegetable, rice and fresh salad. Please do not pack raita as there are chances of spilling. I hope that all the items would be fresh and as per our requirement. Kindly ensure that the packets should reach coach No. S-7. You are requested to be ready with packets at platform No. 1 before the arrival of the train. Payment will be done only after getting the packets.
Thanking you Yours Sincerely,
Pritam
(Co-ordinator, Sriram Vidya Niketan)

Question.6. The other day you visited the grain market at Azamgarh. There you saw sacks of wheat being carried in over loaded * bullock carts by emaciated oxen. You felt disturbed by the cruelty meted out to harmless animals. Write an article in 150-200 words suggesting ways to prevent cruelty towards animals.
OR
Compulsory value-based education is the only remedy for inculcating values among the future citizens of the country. This will also help in the prevention of crime in the country. Write an article in 150-200 words on the needs of compulsory value-based education.
Answer:
Cruelty to Animals
The sight I saw in the grain market was really heart rending. The sacks of wheat were being carried in over loaded bullock carts and the lean and weak oxen hanged in the air as it is was lifted up by the overloaded cart. The ‘PETA’, i.e. People for Ethical Treatment of Animals get awards daily but in reality the situation is entirely different. The prevention of cruelty to animals appears only in news. In fact, the innocent animals are ill-treated and neglected every day. Nobody, cares about the agony of the animals. On a daily basis we come across a sight or two where the horses, donkeys, camels, oxen, etc. are subjected to utter cruelty by heartless human beings.
The owners overload the carts carrying goods and the police turn their eyes blind by taking bribes or on the pretext that the tradesman is poor. I don’t understand why these people refuse to see that the animals cannot narrate their sufferings but they do feel. They have equal right on earth as we have.
It’s high time, to realize that we have a responsibility towards these meek creatures of God which demand our care and love too.
OR
Value Education—The need of the HourIn this fast changing and competitive world, the values and ethics are deteriorating. The moral dilemmas andconflicting values have become too common. Therefore, it is necessary to ensure a curriculum which enables a child to revive his lost values and think and feel right. Only then, a stable and morally integrated future generation will be developed. This can be done through value education which has rightly been projected as a national priority in the National Educational Policy. The children can thus attain self-esteem, dignity and enhance their ability to decide between the good, and bad, right and wrong. Till now the whole emphasis is on academic progress but now it’s time for value education to find its place in the curriculum. The field of value education is as vast as life itself. It touches every aspect of human life and personality. There is sharp decline in the moral standards of the people due to diplomatic attitude and changing interests. The situation is alarming as the ethics and values are in a state of constant flux and this is leading to deterioration of civic sense as well as the country. The main aim of people is only the accumulation of money by any  means for which the crime graph is consistently rising high. They have forgotten their moral duties and consideration for  the fellowmen. Thus, value education should be taught in the  schools on a regular.

SECTION – C
(TEXT BOOK)

Question.8. Answer any three of the following in 30-40 words each :
(d) Why do you think Aunt Jennifer created animals that are so different from her own character ?
Answer : Aunt Jennifer had experienced constraints in her married life. The ring around her finger symbolizes, the weights of her marriage so she created tigers which are representative of the freedom that she aspires for.

Question.9.Answer the following in 30-40 words each :
(e) What did Sophie tell Geoff about her meeting with Danny Casey ?
Answer : Sophie told her brother Geoff that she had met Danny Casey in the arcade and that he was not very tall. He had green eyes and was gentle.

12. Answer the following in 30-40 words each:
(a) What led the Maharaja to start out on a tiger hunt ?
Answer : The Maharaja was told by an astrologer that a tiger would become the cause of his death at the time his birth born. So, he began his tiger hunt as soon as he was entitled to the throne.
(b) In what context does Hana remember General Takima ? What does she infer ?
Answer : Dr. Sadao was sheltering a prisoner of war from .U.S.A., the country with which they were involved in a war. They were always in terror of getting labelled with the tag of terrorist. Thus, she remembered the General in the content that he would support-them in case of any questioning by the authority.

 

CBSE Previous Year Solved Papers Class 12 Computer Science Outside Delhi 2012

CBSE Previous Year Solved  Papers  Class 12 Computer Science Outside Delhi 2012

Time allowed : 3 hours                                                                                           Maximum Marks: 70

General Instructions :

  1.  There are a total of 26 questions and five sections in the question paper, All questions are compulsory.
  2. Section A contains question number 1 to 5, Very Short Answer type questions of one mark each.
  3.  Section B contains question number 6 to 10, Short Answer type I questions of two marks each.
  4.  Section C contains question number 11 to 22, Short Answer type II questions of three marks each.
  5.  Section D contains question number 23, Value Based Question of four marks.
  6. Section E contains question number 24 to 26, Long Answer type questions of five marks each.
  7. There is no overall choice in the question paper, however, an internal choice is provided in one question of two marks, one question of three marks and all three questions of five marks. An examined is to attempt any one of the questions out of two given in the question paper with the same question number.

Question.1.(a) Give the difference between the type casting and automatic type conversion. Also, give a suitable C++ code to illustrate both.
Answer:
cbse-previous-year-solved-papers-class-12-computer-science-outside-delhi-2012-1
1(b)Which C++ header file(s) are essentially required to be included to run/execute the following C++ source code(Note: Do not include any header file, which is/are not required) :
cbse-previous-year-solved-papers-class-12-computer-science-outside-delhi-2012-2
Answer:
(iv) iostream.h
(v) string.h
1(c) Rewrite the following program after removing the syntactical error(s) (if any). Underline each correction.
cbse-previous-year-solved-papers-class-12-computer-science-outside-delhi-2012-3
cbse-previous-year-solved-papers-class-12-computer-science-outside-delhi-2012-4
Answer:
cbse-previous-year-solved-papers-class-12-computer-science-outside-delhi-2012-5
cbse-previous-year-solved-papers-class-12-computer-science-outside-delhi-2012-6
1 (d) Find the output of the following program :
cbse-previous-year-solved-papers-class-12-computer-science-outside-delhi-2012-7
cbse-previous-year-solved-papers-class-12-computer-science-outside-delhi-2012-8
Answer:
cbse-previous-year-solved-papers-class-12-computer-science-outside-delhi-2012-9
1(e) Find the output of the following program :
cbse-previous-year-solved-papers-class-12-computer-science-outside-delhi-2012-10
cbse-previous-year-solved-papers-class-12-computer-science-outside-delhi-2012-11
Answer:
cbse-previous-year-solved-papers-class-12-computer-science-outside-delhi-2012-12
1(f) Observe the following program and find out, which output(s) out of(i) to(iv) will not be expected from the program? What will be the minimum and the maximum value assigned to the variable chance?
cbse-previous-year-solved-papers-class-12-computer-science-outside-delhi-2012-13
cbse-previous-year-solved-papers-class-12-computer-science-outside-delhi-2012-14

Question.2.(a) What is the difference between the members in private visibility mode and the members in protected visibility mode inside a class ? Also, give a suitable C++ code to illustrate both.
Answer:
cbse-previous-year-solved-papers-class-12-computer-science-outside-delhi-2012-15
cbse-previous-year-solved-papers-class-12-computer-science-outside-delhi-2012-16
2. (b) Answer the question (i) and (ii) after going through the following class.
cbse-previous-year-solved-papers-class-12-computer-science-outside-delhi-2012-17
cbse-previous-year-solved-papers-class-12-computer-science-outside-delhi-2012-18

  1.  In Object Oriented Programming, what are Function 1 and Function 4 combined together referred as?
  2.  In Object Oriented Programming, which concept is illustrated by Function 3? When is this function called/ invoked?

Answer:

  1.  Polymorphism OR Constructor Overloading
  2.  Function 3: Destructor ‘
    A destructor called/invoked when an object of that class is destroyed. When a variable goes out of scope, or a dynamically allocated variable is explicitly deleted using the delete keyword, the class destructor is called to help clean up the class before it is removed from memory.

2. (c) Define a class RESTRA in C++ with following description: Private Members:

  • FoodCode of type int
  •  Food of type string
  •  FType of type string
  •  Sticker of type string
  • A member function GetSticker() to assign the following values for Sticker as per the given Ftype
    cbse-previous-year-solved-papers-class-12-computer-science-outside-delhi-2012-19

Public Members:

  •  A function GetFoodO to allow user to enter values for FoodCode, Food,Ftype and call function „ GetSticker( ) to assign Sticker.
  •  A function ShowFoodO to allow user to view the concept of all the data members.

Answer:
cbse-previous-year-solved-papers-class-12-computer-science-outside-delhi-2012-20
cbse-previous-year-solved-papers-class-12-computer-science-outside-delhi-2012-21
cbse-previous-year-solved-papers-class-12-computer-science-outside-delhi-2012-22
2. (d) Answer the questions (i) to (iv) based on the following :
class COMPANY
cbse-previous-year-solved-papers-class-12-computer-science-outside-delhi-2012-23
cbse-previous-year-solved-papers-class-12-computer-science-outside-delhi-2012-24
cbse-previous-year-solved-papers-class-12-computer-science-outside-delhi-2012-25

  1.  Name the type of inheritance illustrated in the above C++ code.
  2.  Write the names of data members, which are accessible from member functions of class SHOP.
  3.  Write the names of all the member functions, which are accessible from objects belonging to class FACTORY.
  4.  Write the names of all the members, which are accessible from objects of class SHOP.

Answer:

  1. Hierarchical Inheritance
  2.  None of the data members can be accessible except SHOP class data members.
  3.  Register!),Enter!) and Show( ) of Factory class.
  4.  Input! ( ) Output!( )

Question.3. (a) Write a function SWAP2BEST (int ARR[],int Size) in C++ to modify the content of the array in such a way that the elements, which are multiples of 10 swap with the value present in the very next position in the array.
For example
cbse-previous-year-solved-papers-class-12-computer-science-outside-delhi-2012-26
Answer:
cbse-previous-year-solved-papers-class-12-computer-science-outside-delhi-2012-27
cbse-previous-year-solved-papers-class-12-computer-science-outside-delhi-2012-28
cbse-previous-year-solved-papers-class-12-computer-science-outside-delhi-2012-29
3. (b) An arrayT[20][10] is stored in the memory along the column with each of the element occupying 2 bytes, find out the memory location of T[10] [5],if an element T[2] [9] is stored at location7600.
Answer:
cbse-previous-year-solved-papers-class-12-computer-science-outside-delhi-2012-30
3. (c) Write a function in C++ to perform insert operation in a static circular Queue containing Book’s information (represented with the help of an array of structure BOOK).
cbse-previous-year-solved-papers-class-12-computer-science-outside-delhi-2012-31
Answer:
cbse-previous-year-solved-papers-class-12-computer-science-outside-delhi-2012-32
3. (d) Write a function ALTERNATE (int A[][3],int N,int M) in C++ to display all alternate element from two¬dimensional array A (starting from A[0] [0]).
For example:
cbse-previous-year-solved-papers-class-12-computer-science-outside-delhi-2012-33
cbse-previous-year-solved-papers-class-12-computer-science-outside-delhi-2012-34
cbse-previous-year-solved-papers-class-12-computer-science-outside-delhi-2012-35
cbse-previous-year-solved-papers-class-12-computer-science-outside-delhi-2012-36
(e) Evaluate the following POSTFIX notation. Show status of stack after every step of evaluation (i.e. after each operator):
True, False, NOT, AND, False, True, OR, AND
Answer:
cbse-previous-year-solved-papers-class-12-computer-science-outside-delhi-2012-37
cbse-previous-year-solved-papers-class-12-computer-science-outside-delhi-2012-38
cbse-previous-year-solved-papers-class-12-computer-science-outside-delhi-2012-39

Question.4. (a)Student try to answer this question Observe the program segment given below carefully and the question that follow:
cbse-previous-year-solved-papers-class-12-computer-science-outside-delhi-2012-40
cbse-previous-year-solved-papers-class-12-computer-science-outside-delhi-2012-41

  1. Write statement 1 to position the file pointer to the appropriate place, so that the data updation is done for the required item.
  2.  Write statement 2 to perform the write operation so that the updation is done in the binary file.

Answer :
Statement 1 – File.seekp(Success);
Statement 2 – File.write((char*) &S, sizeof(S));
(b) Write a function in C++ to read the content of a text file “DELHI.TXT” and display all those lines on screen, which are either starting with ‘D’ or starting With ‘M’
Answer.
cbse-previous-year-solved-papers-class-12-computer-science-outside-delhi-2012-42
cbse-previous-year-solved-papers-class-12-computer-science-outside-delhi-2012-43
cbse-previous-year-solved-papers-class-12-computer-science-outside-delhi-2012-44
(c) Write a function in C++ to search for the details (Phoneno and Calls) of those Phones, which have more , than 800 calls from a binary file “phones.dat”. Assuming that this binary file contains records/objects of class Phone, which is defined below.
cbse-previous-year-solved-papers-class-12-computer-science-outside-delhi-2012-45
Answer:
cbse-previous-year-solved-papers-class-12-computer-science-outside-delhi-2012-46

Question.5. (a) Give a suitable example of a table with sample data and illustrate Primary and Alternate Keys in it.
Answer : Primary Key : Primary key is a set of one or more fields/columns of a table that uniquely identify a record in database table. It cannot accept null, duplicate values. Only one candidate key can be primary key.
Alternate key : Alternate key is a key that can be work as a primary key. Basically it is a Candidate key that currently is not primary key.
Example : In below table Admission No. becomes Alternate Keys when we define Registration No. as Primary Key.
cbse-previous-year-solved-papers-class-12-computer-science-outside-delhi-2012-47
cbse-previous-year-solved-papers-class-12-computer-science-outside-delhi-2012-48
(b) Write SQL commands for the following statements :

  1.  To display the names of all the silver colored Cars.
  2. To display name of car, make and capacity of cars in
    descending order of their sitting capacity.
  3. To display the highest charges at which a vehicle can be hired from CARDEN.
  4. To display the customer name and the corresponding name of the cars hired by them.

Answer:

  1. SELECT CarName FROM carden WHERE Color = ‘Silver’;
  2.  SELECT CarName,Make,Capacity FROM carden ORDER BY Capacity DESC;
  3. SELECT MAX(Charges) FROM carden;
  4. SELECT Cname,CarName FROM carden,customer WHERE

carden. Ccode=customer.Ccode;

(c) Give the output of the following SQL queries :
(i) SELECT COUNT (DISTINCT Make) FROM CARDEN;
(ii) SELECT MAX(Charges),MIN(Charges) FROM CARDEN;
(iii) SELECT COUNT (*) ,Make FROM CARDEN;
(iv) SELECT CarName FROM CARDEN WHERE Capacity=4;
Answer:
cbse-previous-year-solved-papers-class-12-computer-science-outside-delhi-2012-49

Question.6.(a) Verify the following using truth table :
(i) X.XN0
(ii)X+l=l
Answer:
cbse-previous-year-solved-papers-class-12-computer-science-outside-delhi-2012-50
(b) Write the equivalent Boolean Expression for the following Logic Circuit:
cbse-previous-year-solved-papers-class-12-computer-science-outside-delhi-2012-51
(c)Write the SOP form of a Boolean .Function F, which is represented in a truth table as follows :
cbse-previous-year-solved-papers-class-12-computer-science-outside-delhi-2012-52
(d) Reduce the following Boolean Expression using K-Map :
F(A,B,C,D) =∑(2,3,4,5,6,7,8,10,11)
Answer:
cbse-previous-year-solved-papers-class-12-computer-science-outside-delhi-2012-53

Question.7.(a) What out of the following, Svill you use to have an audio-visual chat with an expert sitting in a far-away place to fix-up a technical issue?
(i) VoIP
(ii)email
(iii)FTP
Answer: (i) VoIP
(b) Name one server side scripting language and one client side scripting language.
Answer:

  •  Client side script:
    (a) Javascript
  • server side script:
    (a) ASP

(c)Which out of the following comes under Cyber Crime?

  1. Operating someone’s internet banking account, without his knowledge.
  2. Stealing a keyboard from someone’s computer.
  3. Working on someone’s computer with his/her permission.

Answer:

  1. Operating someone’s internet banking account, without his knowledge.
    (d) Write one advantage of bus topology of network. Also, illustrate how 4 computers can be connected with each other using star topology of network.
    Answer:
    cbse-previous-year-solved-papers-class-12-computer-science-outside-delhi-2012-54

(e) Workalot consultants are setting up a secured network for their office campus at Gurgaon for their day- to-day office and web-based activities. They are planning to have connectivity between 3 buildings and the head office situated in Mumbai. Answer the questions (i) to(iv) after going through the building positions in the campus and other details, which are given below :
cbse-previous-year-solved-papers-class-12-computer-science-outside-delhi-2012-55
cbse-previous-year-solved-papers-class-12-computer-science-outside-delhi-2012-56

  1. Suggest the most suitable place (i.e. building) to house the server of this organization. Also give a reason to justify your suggested location.
  2. Suggest a cable layout of connection between the buildings inside the campus.
  3. Suggest the placement of the following device with justification:
    (1) Switch
    (2) Repeater
  4. The organization is planning to provide a high speed link with its head office situated in MUMBAI using a wired connection. Which of the following cables will be most suitable for this job?
    (1) Optical Fiber
    (2) Co-axial Cable
    (3) Ethernet Cable

Answer: (i)
RED building because maximum number of computers are there
OR
BLUE building because closest to all other building (minimum cable length required)
Answer: (ii)
cbse-previous-year-solved-papers-class-12-computer-science-outside-delhi-2012-57
Answer: (iii)
Switch. By using 1 switch per building we can use maximum numbers of computers to connect them in network.
Answer : (iv) Optical Fiber
(f) Give one suitable example of each URL and Domain Name.
Answer: URL – http://www.cbsecsnip.in/index.php Domain – cbsecsnip
(g) Name two Proprietary software along with their application.
Answer:
(i) Microsoft Office : Microsoft Office belongs to Microsoft Corporation. This software is used for office automation and also can be used other than office productivity at personal level. Microsoft Office contains following other applications like Microsoft Word, Microsoft Excel, Microsoft PowerPoint, etc.
(ii)Oracle : Oracle Corporation is the owner of Oracle software. Oracle is one of most popular RDBMS software in world.